PDA

نسخه کامل مشاهده نسخه کامل : اتاق ریاضیات(طرح سؤالات)



صفحه ها : 1 2 3 4 5 6 7 8 9 10 11 12 13 14 [15] 16 17 18 19 20

mofidy1
19-06-2011, 13:13
لطفا ببينيد جواب اين معادله ديفرانسيل چي ميشه؟راه حلش منظورمه


[ برای مشاهده لینک ، لطفا با نام کاربری خود وارد شوید یا ثبت نام کنید ]

با سلام


[ برای مشاهده لینک ، لطفا با نام کاربری خود وارد شوید یا ثبت نام کنید ]{y''}{y'}=ln(y')\\\left ( ln(y')\right )'=ln(y')\\ln(y')=ae^x\\y'=e^{ae^x}\\y=\int e^{ae^x}\;dx

انتگرال آخر را می توان بر حسب تابع گاما و توابع فوق هندسی نمایش داد که این جا جای بحث آن نیست.

موفق باشید.

29 خرداد 1390

mofidy1
19-06-2011, 13:47
سلام ، خوبین ؟ آقا من توو معادله زیر نمیتونم C رو حذف کنم کمک کنید 3 روز دیگه امتحان دارم

مسیر های متعامد خانواده منحنی های زیر را بیابید

x^2 - y^2 = cx


با سلام

پس از مشتق ضمنی از طرفین به معادله ی زیر می رسید:


[ برای مشاهده لینک ، لطفا با نام کاربری خود وارد شوید یا ثبت نام کنید ]'=\frac{c}{2}
پارامتر c را هم از معادله ی اصلی پیدا کنید:


[ برای مشاهده لینک ، لطفا با نام کاربری خود وارد شوید یا ثبت نام کنید ]{x^2-y^2}{x}
و آن را در معادله ی اول قرار دهید:


[ برای مشاهده لینک ، لطفا با نام کاربری خود وارد شوید یا ثبت نام کنید ]'=\frac{x^2-y^2}{2x}
که نتیجه می دهد:

[ برای مشاهده لینک ، لطفا با نام کاربری خود وارد شوید یا ثبت نام کنید ]'=\frac{x^2+y^2}{2xy}

حال به جای 'y قرینه ی معکوس آن را قرار داده، معادله ی دیفرانسیل به دست آمده را حل می کنیم تا مسیر های متعامد به دست آید:


[ برای مشاهده لینک ، لطفا با نام کاربری خود وارد شوید یا ثبت نام کنید ]{-1}{y'}=\frac{x^2+y^2}{2xy}

با تغییر شکل دیفرانسیلی معادله ی بالا خواهیم داشت:


[ برای مشاهده لینک ، لطفا با نام کاربری خود وارد شوید یا ثبت نام کنید ](x^2+y^2)\;dy=0

معادله ی بالا یک معادله ی کامل است، یعنی مشتق عبارت سمت چپ نسبت به y با مشتق عبارت سمت راست نسبت به x برابر است. بنابر قضیه، تابع دو متغیره ی f چنان موجود است که:


[ برای مشاهده لینک ، لطفا با نام کاربری خود وارد شوید یا ثبت نام کنید ]{\partial f}{\partial x}=-2xy,\;\;\frac{\partial f}{\partial y}=-x^2-y^2

همچنین تابع g چنان موجود است که:


[ برای مشاهده لینک ، لطفا با نام کاربری خود وارد شوید یا ثبت نام کنید ] -2xy\;dx+g(y)=-x^2y+g(y)

بنابراین می توان نوشت:


[ برای مشاهده لینک ، لطفا با نام کاربری خود وارد شوید یا ثبت نام کنید ]^2-y^2=\frac{\partial f}{\partial y}=-x^2+g'(y)

بنابراین


[ برای مشاهده لینک ، لطفا با نام کاربری خود وارد شوید یا ثبت نام کنید ](y)=\frac{-1}{3}y^3

در نتیجه


[ برای مشاهده لینک ، لطفا با نام کاربری خود وارد شوید یا ثبت نام کنید ](x,y)=-x^2y-\frac{1}{3}y^3

لذا جواب مساله عبارت است از:


[ برای مشاهده لینک ، لطفا با نام کاربری خود وارد شوید یا ثبت نام کنید ]^2y-\frac{1}{3}y^3=c

که مسیرهای متعامد را به شما می دهند.

تذکر: اگر نیاز به قضیه ای درباره ی مساله ی بالا داشتید، به ترجمه ی کتاب معادلات دیفرانسیل و کاربردهای آن تالیف جرج سیمونز صفحه ی 42 تا 45 مراجعه فرمایید.

موفق باشید.

29 خرداد 1390

sokote mordab
20-06-2011, 19:51
سلام

استاد می تونید این سه تا سوال را حل کنید برام.ممنونم
[ برای مشاهده لینک ، لطفا با نام کاربری خود وارد شوید یا ثبت نام کنید ]

..:: HoRaTo ::..
21-06-2011, 08:54
سلام
امکان داره بهم بگید چطور ممکن هست این حد رو حل کنم ؟ خیلی کلافه شدم !


[ برای مشاهده لینک ، لطفا با نام کاربری خود وارد شوید یا ثبت نام کنید ]


دوستان اینم یه راه دیگه برا حل این مسئله

[ برای مشاهده لینک ، لطفا با نام کاربری خود وارد شوید یا ثبت نام کنید ]

mansour_ir2000
21-06-2011, 13:18
سلام
دوستان کسی می تونه مساحت ذوزنقه شکل پایین رو برام محاسبه کنه + فروملش رو که استفاده می کنه هم بنویسه؟ یا نرم افزاری برای این کار وجود داره که با کامپیوتر بتونم محاسبه اینجوری مساحت رو محاسبه کنم:19:
(ذوزنقه 2 ضلع بزرگ 870 متر و ضلع های کوچیک 60 و 48 متر )
تشکر:31:

برای مشاهده محتوا ، لطفا وارد شوید یا ثبت نام کنید

mofidy1
21-06-2011, 16:29
سلام
دوستان کسی می تونه مساحت ذوزنقه شکل پایین رو برام محاسبه کنه + فروملش رو که استفاده می کنه هم بنویسه؟ یا نرم افزاری برای این کار وجود داره که با کامپیوتر بتونم محاسبه اینجوری مساحت رو محاسبه کنم:19:
(ذوزنقه 2 ضلع بزرگ 870 متر و ضلع های کوچیک 60 و 48 متر )
تشکر:31:

برای مشاهده محتوا ، لطفا وارد شوید یا ثبت نام کنید

با سلام

اگر A را مساحت ذورنقه و c، d ، b، a را اضلاع آن فرض کنید، فرمول زیر را داریم (a و b اضلاع موازی هستند که b بزرگ تر یا مساوی a است):

[ برای مشاهده لینک ، لطفا با نام کاربری خود وارد شوید یا ثبت نام کنید ]

که در این فرمول

[ برای مشاهده لینک ، لطفا با نام کاربری خود وارد شوید یا ثبت نام کنید ]

در مساله ی شما تمام اضلاع داده شده است، ابتدا s را به دست آورید و سپس A را محاسبه کنید.

منبع:

برای مشاهده محتوا ، لطفا وارد شوید یا ثبت نام کنید

موفق باشید.

31 خرداد 1390

mofidy1
21-06-2011, 16:59
سلام

استاد می تونید این سه تا سوال را حل کنید برام.ممنونم


1- معادله ی برنولی:

[ برای مشاهده لینک ، لطفا با نام کاربری خود وارد شوید یا ثبت نام کنید ]'+y=x^4y^3

حل مساله:

معادله را به صورت زیر درآورید:

[ برای مشاهده لینک ، لطفا با نام کاربری خود وارد شوید یا ثبت نام کنید ]'+\frac{1}{x}y=x^3y^3

تغییر متغیر زیر را اعمال کنید:


[ برای مشاهده لینک ، لطفا با نام کاربری خود وارد شوید یا ثبت نام کنید ]^{-2}

با این تغییر متغیر، معادله به یک معادله ی خطی تبدیل می شود که آن را حل کنید.


2- تبدیل لاپلاس:

[ برای مشاهده لینک ، لطفا با نام کاربری خود وارد شوید یا ثبت نام کنید ](t)=t^3-cos(3t)+sin(2t)\\L(F(t))=L(t^3)-L(cos(3t))+L(sin(2t))\\L(F(t))=\frac{6}{p^4}-\frac{p}{p^2+9}+\frac{2}{p^2+9}

[ برای مشاهده لینک ، لطفا با نام کاربری خود وارد شوید یا ثبت نام کنید ](t)=e^{3t}+t-1\\L(F(t))=\frac{1}{p-3}+\frac{1}{p^2}-\frac{1}{p}

3 - تبدیل لاپلاس معکوس


[ برای مشاهده لینک ، لطفا با نام کاربری خود وارد شوید یا ثبت نام کنید ](F(x))=\frac{2}{s-3}-\frac{s}{s^2-16}=2L(e^{3x})-L(sinh(4x))\\ \\F(x)=2e^{3x}-sinh(4x)

موفق باشید.

majj
21-06-2011, 17:52
[ برای مشاهده لینک ، لطفا با نام کاربری خود وارد شوید یا ثبت نام کنید ] ([ برای مشاهده لینک ، لطفا با نام کاربری خود وارد شوید یا ثبت نام کنید ])


اين رو ببينيد درست حل كردم[COLOR="Silver"]

arash7960
21-06-2011, 20:02
لطفآ اثبات کنید :
برای اعداد حقیقی و مثبت x,y
اگر x^3+y^3 = x-y
نشان دهید : x^2+y^2<1

sokote mordab
21-06-2011, 21:35
با سلام

پس از مشتق ضمنی از طرفین به معادله ی زیر می رسید:


[ برای مشاهده لینک ، لطفا با نام کاربری خود وارد شوید یا ثبت نام کنید ]'=\frac{c}{2}
پارامتر c را هم از معادله ی اصلی پیدا کنید:


[ برای مشاهده لینک ، لطفا با نام کاربری خود وارد شوید یا ثبت نام کنید ]{x^2-y^2}{x}
و آن را در معادله ی اول قرار دهید:


[ برای مشاهده لینک ، لطفا با نام کاربری خود وارد شوید یا ثبت نام کنید ]'=\frac{x^2-y^2}{2x}
که نتیجه می دهد:

[ برای مشاهده لینک ، لطفا با نام کاربری خود وارد شوید یا ثبت نام کنید ]'=\frac{x^2&plus;y^2}{2xy}

حال به جای 'y قرینه ی معکوس آن را قرار داده، معادله ی دیفرانسیل به دست آمده را حل کنید تا مسیر های متعامد به دست آید.

موفق باشید.

29 خرداد 1390

ممنون از حل سه تمرین گذشته اگر میشه این تمرین را کامل حل کنید.

lebesgue
21-06-2011, 23:36
لطفآ اثبات کنید :
برای اعداد حقیقی و مثبت x,y
اگر x^3+y^3 = x-y
نشان دهید : x^2+y^2<1


[ برای مشاهده لینک ، لطفا با نام کاربری خود وارد شوید یا ثبت نام کنید ]{y}{x}=a%5C%5C%5C%5C%20x^3+y^3%20 =%20x-y%20%5CRightarrow%20x^3+(ax)^3%20=%20x-ax%5C%5C%5C%5C%20%5CRightarrow%20x^3(1+a^3)%20=%20 x(1-a)%5C%5C%5C%5C%20%5CRightarrow%20x^2%20=%20%5Cfrac {(1-a)}{(1+a^3)}%5C%5C%5C%5C%20%5CRightarrow%20x^2+y^2 =x^2+(ax)^2=%5Cfrac{(1-a)(1+a^2)}{1+a^3}

چون x و y و در نتیجه x^3+y^3 مثبت است، a بین 0 و 1 خواهد بود و به سادگی نتیجه میشود عبارت بدست آمده کوچکتر از 1 است.

mofidy1
22-06-2011, 07:39
لطفآ اثبات کنید :
برای اعداد حقیقی و مثبت x,y
اگر x^3+y^3 = x-y
نشان دهید : x^2+y^2<1

با سلام

روش اول در بالا توسط 1233445566 ارائه شد. با تشکر از ایشان روش دوم را به برهان خلف ارائه می کنیم که شاید کمی ساده تر باشد:

فرض کنیم نتیجه ی مورد نظر درست نباشد، یعنی:


[ برای مشاهده لینک ، لطفا با نام کاربری خود وارد شوید یا ثبت نام کنید ] x^2+y^2\geq 1

به طرفین xy- را اضافه کنید. با استفاده از فرض مساله و اتحاد چاق و لاغر!!، داریم:


[ برای مشاهده لینک ، لطفا با نام کاربری خود وارد شوید یا ثبت نام کنید ] \frac{x-y}{x+y}\geq 1-xy

پس از طرفین وسطین و نیز ساده کردن عبارت، به دست می آید:


[ برای مشاهده لینک ، لطفا با نام کاربری خود وارد شوید یا ثبت نام کنید ] x^2+xy\geq 2

حال اگر به طرفین y^2 را اضافه و باز هم از اتحاد چاق و لاغر!! استفاده کنید، خواهیم داشت:


[ برای مشاهده لینک ، لطفا با نام کاربری خود وارد شوید یا ثبت نام کنید ] 1\geq \frac{x^3-y^3}{x^3+y^3}=\frac{x^3-y^3}{x-y}\geq 2+y^2

که تناقض است، زیرا سمت راست، از 2 بزرگ تر است.

موفق باشید.
1 تیر 1390

mofidy1
22-06-2011, 09:01
ممنون از حل سه تمرین گذشته اگر میشه این تمرین را کامل حل کنید.

با سلام

حل مساله در همان جا کامل شد.

موفق باشید.

lebesgue
22-06-2011, 09:56
لطفآ اثبات کنید :
برای اعداد حقیقی و مثبت x,y
اگر x^3+y^3 = x-y
نشان دهید : x^2+y^2<1


دوست عزیزی به یک اثبات دیگر نیز اشاره کرد که برای تنوع بیشتر، آنرا در اینجا می گذارم.

از معادله [ برای مشاهده لینک ، لطفا با نام کاربری خود وارد شوید یا ثبت نام کنید ]^3+y^3%20=%20x-y و بازنویسی آن به صورت [ برای مشاهده لینک ، لطفا با نام کاربری خود وارد شوید یا ثبت نام کنید ]^3+y%20=%20x(1-x^2) نتیجه میشود که [ برای مشاهده لینک ، لطفا با نام کاربری خود وارد شوید یا ثبت نام کنید ]

حال معادله را به صورت زیر بازنویسی می کنیم:

[ برای مشاهده لینک ، لطفا با نام کاربری خود وارد شوید یا ثبت نام کنید ][2-x(x+y)]=(x+y)(1-x^2-y^2)

[ برای مشاهده لینک ، لطفا با نام کاربری خود وارد شوید یا ثبت نام کنید ] نتیجه میدهد که [ برای مشاهده لینک ، لطفا با نام کاربری خود وارد شوید یا ثبت نام کنید ](x+y)%3C2 و [ برای مشاهده لینک ، لطفا با نام کاربری خود وارد شوید یا ثبت نام کنید ](x+y)%3C2. پس سمت چپ معادله بالا مثبت بوده و بنابراین سمت راست هم مثبت خواهد بود.

sokote mordab
22-06-2011, 11:05
سلام

استاد می تونید این سه تا سوال را حل کنید برام.ممنونم

برای مشاهده محتوا ، لطفا وارد شوید یا ثبت نام کنید


با تشکر ویژه از حل مسائل قبلی اگر یک بزرگواری بکنید این معادله برنولی را کامل حل کنید اخرین درخواست من هست.

mostafabad2007
24-06-2011, 10:48
با سلام اگه ممکنه این سوالات انتگرالو برام حل کند چون امتحال ریاضی دارم مال دانشگاهه


برای مشاهده محتوا ، لطفا وارد شوید یا ثبت نام کنید

------------------------------------------------
با سلام

آدرس عکس به علت حجیم بودن آن ویرایش شد. کاربران عزیز رعایت فرمایند.

مدیر انجمن ریاضیات

mofidy1
24-06-2011, 20:31
با سلام اگه ممکنه این سوالات انتگرالو برام حل کند چون امتحال ریاضی دارم مال دانشگاهه


برای مشاهده محتوا ، لطفا وارد شوید یا ثبت نام کنید



با سلام

دوست من، انتظار ندارید که تمام این 41 سوال در این انجمن حل شود؟!! روی چند تا از این مسائل خودتان کار کرده اید و به نتیجه نرسیده اید؟! بعضی از این مسائل به قدری ساده هستند، که به راحتی می توانید با کمی فکر کردن حلشان کنید. چرا انتظار دارید دیگران به جای شما فکر کنند و در نتیجه زحمات یک ترم استادتان بر باد رود؟

از بین این ها مسائلی را که به هیچ عنوان نتوانسته اید حل کنید، جدا و در انجمن مطرح کنید. در این صورت مطمئن باشید بنده و دوستان دیگر کمکتان خواهیم کرد.

موفق باشید.

3 تیر 1390

Smartie7
25-06-2011, 07:51
سلام

لطف کنید توضیح بدید برای تشخیص زوج یا فرد بودن این توابع باید چیکار کرد؟!



[ برای مشاهده لینک ، لطفا با نام کاربری خود وارد شوید یا ثبت نام کنید ]

mofidy1
25-06-2011, 20:57
سلام

لطف کنید توضیح بدید برای تشخیص زوج یا فرد بودن این توابع باید چیکار کرد؟!



[ برای مشاهده لینک ، لطفا با نام کاربری خود وارد شوید یا ثبت نام کنید ]


با سلام

دامنه ی هر سه تابع نسبت به مبدأ متقارن است. x را به x- تبدیل کنید، اگر تغییری در ضابطه ایجاد نشد، تابع زوج و اگر به قرینه ی خود تبدیل شد، تابع فرد است. تابع اول و آخر زوج و تابع وسطی فرد است.

موفق باشید.

4 تیر 1390

Smartie7
25-06-2011, 21:19
پس sin همیشه فرد هست و cos همیشه زوج، درسته؟!

حالا Lnx رو چطوری زوج و فرد بودنش رو مشخص کنیم؟!

kavehj4488
26-06-2011, 09:06
ببخشيد يه سؤال داشتم درباره تركيبيات:
با ارقام 2 و 0 و 0 و 0 و 3 چند عدد ژنج رقمي مي توان ساخت؟ (عدد بايد سه رقم صفر داشته باشد.)

..:: HoRaTo ::..
26-06-2011, 11:45
سلام اگه میشه لاپلاس معکوس تابع زیر را به دست آورید(با راه حل).
G(s)=stg(1/s)

lebesgue
26-06-2011, 14:33
پس sin همیشه فرد هست و cos همیشه زوج، درسته؟!

حالا Lnx رو چطوری زوج و فرد بودنش رو مشخص کنیم؟!


خب دامنه تابع ln اعداد حقیقی مثبت است و در نتیجه نامتقارن نسبت به مبدا. بنابراین نه زوج است و نه فرد.

ببخشيد يه سؤال داشتم درباره تركيبيات:
با ارقام 2 و 0 و 0 و 0 و 3 چند عدد ژنج رقمي مي توان ساخت؟ (عدد بايد سه رقم صفر داشته باشد.)
رقم اول یا 2 است یا 3، اگر 2 باشد، برای 3، چهار جایگاه [ برای مشاهده لینک ، لطفا با نام کاربری خود وارد شوید یا ثبت نام کنید ] uare0%5Csquare موجود است و اگر 3 باشد به طور مشابه. در نتیجه پاسخ 8 است.

سلام اگه میشه لاپلاس معکوس تابع زیر را به دست آورید(با راه حل).
G(s)=stg(1/s)

احتمالاً منظور شما این نبوده؟ :

[ برای مشاهده لینک ، لطفا با نام کاربری خود وارد شوید یا ثبت نام کنید ]{120}%20G(s)=s%5Ctan^{-1}%20%5Cleft%20(%5Cfrac{1}{s}%20%5Cright%20)%5C%5C %5C%5C

Smartie7
26-06-2011, 19:29
لطفاً نحوه ی پیدا کردن GLB و LUB و همینطور نحوه ی تشخیص توزیع پذیر بودن شبکه رو توضیح بدید.

کدامیک از نمودارهای زیر، شبکه هستند؟
کدامیک شبکه ی توزیع پذیر هستند؟



[ برای مشاهده لینک ، لطفا با نام کاربری خود وارد شوید یا ثبت نام کنید ]


[ برای مشاهده لینک ، لطفا با نام کاربری خود وارد شوید یا ثبت نام کنید ]


:11:

..:: HoRaTo ::..
27-06-2011, 12:42
احتمالاً منظور شما این نبوده؟ :

[ برای مشاهده لینک ، لطفا با نام کاربری خود وارد شوید یا ثبت نام کنید ] ht%20%29%5C%5C%5C%5C

بله ببخشید منظورم همین بود.

lebesgue
27-06-2011, 17:40
[ برای مشاهده لینک ، لطفا با نام کاربری خود وارد شوید یا ثبت نام کنید ]{120}%20%5Cmathcal{L}%5Cleft%20%5C {%20g(t)%20%5Cright%20%5C}=G(s)%5C%5C%5C%5C%20%5Cm athcal{L}%5Cleft%20%5C{%20%5Cint_{0}^{t}g(%5Ctau%2 0)d%5Ctau%20%5Cright%20%5C}=%5Cfrac{G(s)}{s}%5C%5C %5C%5C%20%5Cmathcal{L}%5Cleft%20%5C{%20t%5Cint_{0} ^{t}g(%5Ctau%20)d%5Ctau%20%5Cright%20%5C}=-%5Cfrac{%5Cmathrm{d}%20}{%5Cmathrm{d}%20s}%5Cleft% 20[%5Cfrac{G(s)}{s}%20%5Cright%20]=%5Cfrac{1}{1+s^2}%5C%5C%5C%5C%20t%3E0:%5C:%20%5C: %20%5C:%20%5C:%20t%5Cint_{0}^{t}g(%5Ctau%20)d%5Cta u=%5Csin%20t%5C%5C%5C%5C%20%5Cint_{0}^{t}g(%5Ctau% 20)d%5Ctau=%5Cfrac{%5Csin%20t}{t}%5C%5C%5C%5C%20g( t)=%5Cfrac{%5Cmathrm{d}%20}{%5Cmathrm{d}%20t}%5Cle ft%20[%5Cfrac{%5Csin%20t}{t}%20%5Cright%20]=%5Cfrac{t%5Ccos%20t-%5Csin%20t}{t^2}
در اینجا فرض شد که تبدیل لاپلاس یکطرفه مورد نظر بوده است، برای تبدیل لاپلاس دوطرفه پاسخ کمی متفاوت خواهد بود.

majj
29-06-2011, 08:32
[ برای مشاهده لینک ، لطفا با نام کاربری خود وارد شوید یا ثبت نام کنید ]

lebesgue
29-06-2011, 10:12
راه حل اول: با تعریف تبدیل لاپلاس شروع کرده و با محاسبه انتگرال، پاسخ را بیابید. برای محاسبه انتگرال میتوانید از روش جزء به جزء استفاده کنید، یا اینکه با استفاده از فرمول اویلر به یک انتگرال ساده تر دست یابید.

راه حل دوم: تابع (f(x را به صورت زیر بنویسید:

[ برای مشاهده لینک ، لطفا با نام کاربری خود وارد شوید یا ثبت نام کنید ]{120}%20f(x)=%5Csin%20(3x)%5Cleft% 20[u(x)-u(x+%5Cfrac{%5Cpi}{2})%20%5Cright%20]

که در اینجا (u(x تابع پله واحد است. در اینصورت میتوانید با استفاده از تبدیل لاپلاس sin3x و cos3x و همچنین قضیه انتقال، به جواب برسید.

mofidy1
29-06-2011, 12:51
با سلام

دوستان عزیز یکی از کاربران خارج از کشور، برای حقیر مسأله ی زیر را فرستادند و خواهان حل آن شدند. فکر کردم اگر راه حل های دوستان پی سی ورلدی را هم ببینیم، بد نباشد. اگر لطف کنید و راه حل های (احتمالاً مختلف) تان را ارسال فرمایید، ممنون خواهم شد. با تشکر قبلی.

یک باشگاه به تازگی شروع به کار کرده و هر ماه به تعداد اعضاء باشگاه افزوده می شود به این ترتیب که ؛ ماه مهر 35 نفر ، ماه آبان 42 نفر، ماه آذر 58 نفر و ماه دی 84 نفر اضافه شدند ضمن این که باشگاه تصمیم گرفته جمعه آخر هر ماه پیتزا بفروشه به اعضاء_ 50% اعضا معمولا پیتزا خرید می کنند... لطفا جواب دهید که برای ماه بعد یعنی ماه بهمن به چند پیتزا احتیاج است( نکته این که هر پیتزا به چهار قسمت مساوی و برای چهار نفر به فروش می رسد)

موفق باشید.

son_silent
30-06-2011, 17:31
سلام

کتاب جامع ریاضی نیاز دارم . که از مباحث راهنمای و دبیرستان تا دانشگاه توش باشه . تعداد کتاب مهم نیست فقط خوب توضیح داده باشه ممنون میشم اگه معرفی کنین

mofidy1
01-07-2011, 19:23
سلام

کتاب جامع ریاضی نیاز دارم . که از مباحث راهنمای و دبیرستان تا دانشگاه توش باشه . تعداد کتاب مهم نیست فقط خوب توضیح داده باشه ممنون میشم اگه معرفی کنین

با سلام

به لینک زیر مراجعه فرمایید:


برای مشاهده محتوا ، لطفا وارد شوید یا ثبت نام کنید

موفق باشید.

10 تیر 1390

ask_bl
01-07-2011, 21:36
با سلام

دوستان عزیز یکی از کاربران خارج از کشور، برای حقیر مسأله ی زیر را فرستادند و خواهان حل آن شدند. فکر کردم اگر راه حل های دوستان پی سی ورلدی را هم ببینیم، بد نباشد. اگر لطف کنید و راه حل های (احتمالاً مختلف) تان را ارسال فرمایید، ممنون خواهم شد. با تشکر قبلی.

یک باشگاه به تازگی شروع به کار کرده و هر ماه به تعداد اعضاء باشگاه افزوده می شود به این ترتیب که ؛ ماه مهر 35 نفر ، ماه آبان 42 نفر، ماه آذر 58 نفر و ماه دی 84 نفر اضافه شدند ضمن این که باشگاه تصمیم گرفته جمعه آخر هر ماه پیتزا بفروشه به اعضاء_ 50% اعضا معمولا پیتزا خرید می کنند... لطفا جواب دهید که برای ماه بعد یعنی ماه بهمن به چند پیتزا احتیاج است( نکته این که هر پیتزا به چهار قسمت مساوی و برای چهار نفر به فروش می رسد)

موفق باشید.
سلام
اگر توی بهمن ماه عضو نگیره به حداقل 28 تا پیزا نیاز داره!

peyman8012
01-07-2011, 23:29
با سپاس از آقای مفیدی برای طرح سئوال من در انجمن:
سئوال این بود که : یک باشگاه به تازگی شروع به کار کرده و هر ماه به تعداد اعضاء باشگاه افزوده می شود به این ترتیب که ؛ ماه مهر 35 نفر ، ماه آبان 42 نفر، ماه آذر 58 نفر و ماه دی 84 نفر اضافه شدند ضمن این که باشگاه تصمیم گرفته جمعه آخر هر ماه پیتزا بفروشه به اعضاء_ 50% اعضا معمولا پیتزا خرید می کنند... لطفا جواب دهید که برای ماه بعد یعنی ماه بهمن به چند پیتزا احتیاج است( نکته این که هر پیتزا به چهار قسمت مساوی و برای چهار نفر به فروش می رسد)
آقای ask_bl به سئوال جواب کوتاه 28 رو دادند... ای کاش امکان داشت که توضیح بدهند چطور به این نتیجه رسیدند... من خودم موفق به حلش نشدم و فکر می کردم که می شه از راه حل اعداد باینر به نتیجه رسید که اشتباه بود... منتظر جواب دوستان هستم...:11:

hts1369
05-07-2011, 10:33
من میدونم که عبارت اولی که نوشتم درست هست ولی در مورد دومی شک دارم میخواستم بدونم درسته یا نه؟[ برای مشاهده لینک ، لطفا با نام کاربری خود وارد شوید یا ثبت نام کنید ]

mofidy1
05-07-2011, 16:32
من میدونم که عبارت اولی که نوشتم درست هست ولی در مورد دومی شک دارم میخواستم بدونم درسته یا نه؟[ برای مشاهده لینک ، لطفا با نام کاربری خود وارد شوید یا ثبت نام کنید ]

با سلام

نه دومی هم درست نیست. به جای تتا 1 قرار دهید. طرف راست «دو پی ام» و طرف چپ «پی دوم» خواهد شد.

موفق باشید.

14 تیر 1390

lebesgue
05-07-2011, 17:47
من میدونم که عبارت اولی که نوشتم درست هست ولی در مورد دومی شک دارم میخواستم بدونم درسته یا نه؟[ برای مشاهده لینک ، لطفا با نام کاربری خود وارد شوید یا ثبت نام کنید ]

رابطه صحیح به صورت زیر است:

[ برای مشاهده لینک ، لطفا با نام کاربری خود وارد شوید یا ثبت نام کنید ]
برای بازه [1,1-] به غیر از 0.

برای دیدن روابط بیشتر میان توابع معکوس مثلثاتی میتوانید به پیوند زیر مراجعه کنید:

برای مشاهده محتوا ، لطفا وارد شوید یا ثبت نام کنید

lebesgue
06-07-2011, 11:41
فرض کنید [ برای مشاهده لینک ، لطفا با نام کاربری خود وارد شوید یا ثبت نام کنید ] چهار عدد حقیقی و مثبت باشند که [ برای مشاهده لینک ، لطفا با نام کاربری خود وارد شوید یا ثبت نام کنید ] ثابت کنید:


[ برای مشاهده لینک ، لطفا با نام کاربری خود وارد شوید یا ثبت نام کنید ]^3+x_2^3+x_3^3+x_4^3%5Cgeq%20x_1+x_2+ x_3+x_4

منبع: بعداً اضافه خواهد شد.

davy jones
11-07-2011, 16:16
با سپاس از آقای مفیدی برای طرح سئوال من در انجمن:
سئوال این بود که : یک باشگاه به تازگی شروع به کار کرده و هر ماه به تعداد اعضاء باشگاه افزوده می شود به این ترتیب که ؛ ماه مهر 35 نفر ، ماه آبان 42 نفر، ماه آذر 58 نفر و ماه دی 84 نفر اضافه شدند ضمن این که باشگاه تصمیم گرفته جمعه آخر هر ماه پیتزا بفروشه به اعضاء_ 50% اعضا معمولا پیتزا خرید می کنند... لطفا جواب دهید که برای ماه بعد یعنی ماه بهمن به چند پیتزا احتیاج است( نکته این که هر پیتزا به چهار قسمت مساوی و برای چهار نفر به فروش می رسد)
آقای ask_bl به سئوال جواب کوتاه 28 رو دادند... ای کاش امکان داشت که توضیح بدهند چطور به این نتیجه رسیدند... من خودم موفق به حلش نشدم و فکر می کردم که می شه از راه حل اعداد باینر به نتیجه رسید که اشتباه بود... منتظر جواب دوستان هستم...:11:
سلام خدمت همه ی دوستان ریاضی دان خودم خصوصا جناب استاد مفیدی!
بعد از مدتها دوری از اینجا فرصتی دست داد تا دوباره به جمع با صفای شما دوستان سری بزنم. (از اونجایی که مشغول خدمت سربازی هستم کمتر فرصت نت دست میده:31:)
در مورد این سوال دوستمون هم باید عرض کنم که به نظر من منظور طراح سوال این بوده که از راه درونیابی به روش چندجمله ایها تعداد اعضای ماه بهمن رو حدس بزنیم. به این صورت که در سوال 4 زوج مرتب به عنوان داده به ما داده شده است و ما میتوانیم از این 4 نقطه یک و فقط یک چندجمله ای از درجه ی 3 عبور دهیم. (در حالت کلی از هر n تا نقطه در فضای دوبعدی میتوان یک و فقط یک تابع چندجمله ای از درجه n-1 عبور داد) اگر ضرایب چند جمله ای را به ترتیب از درجه ی 3 تا صفر برابر با a و b و c و d بگیریم و 4 نقطه ی داده رو در تابع جایگذاری کنیم ضرایب به دست میآیند (البته به جای اسامی ماه ها از عدد 1 شروع میکنیم و یک واحد یک واحد جلو میرویم. یعنی مثلا مهماه معادل 1، آبان ماه معادل 2 و ... در دامنه خواهند بود.) بدین ترتیب مقدار تقریبی تعداد اعضای باشگاه در ماه بهمن معادل مقدار تابع چندجمله ای بدست آمده به ازای x=5 خواهد بود.
از اینکه حوصله و وقت کافی جهت محاسبه ی کامل راه حل پیشنهادی خودم رو ندارم عذر میخوام (آخه الان تو کافی نت نشستم و دارم تایپ میکنم:31:)

موفق باشین.
90/4/20

peyman8012
13-07-2011, 13:33
سلام خدمت دوستان
ممنون جناب davy jones
الان جواب پیشنهادی شما رو دیدم... ممنون از قبول زحمت و توضیح... پیشنهاد شما برای حل رو الان امتحان می کنم...

kiak22
13-07-2011, 23:03
داداشا برین کنار خلوت کنین که اقا کیانوش وارد می شود مخه معادله ام تا میتونین بهم معادله سخت بدین ولی مساله ای نه ها معادله بدین هر کی بتونه معادله کاملی رو بده که من نتونم حل کنم دمششششششششششش گرم و من کنار میرم و از مدیر هم در خواست سوال می کنم برای مثال یک نمونه که اینجوری معادله بدین
x=2
3x+6=2y
یا در سطح بالا تر مثلا
xcos+xsin=دو به توان ایکس
اینا همش مثال بود اینا که سهله سخت تر بدین ولی همینجوری واضح و شفاف

mofidy1
15-07-2011, 09:59
داداشا برین کنار خلوت کنین که اقا کیانوش وارد می شود مخه معادله ام تا میتونین بهم معادله سخت بدین ولی مساله ای نه ها معادله بدین هر کی بتونه معادله کاملی رو بده که من نتونم حل کنم دمششششششششششش گرم و من کنار میرم و از مدیر هم در خواست سوال می کنم برای مثال یک نمونه که اینجوری معادله بدین
x=2
3x+6=2y
یا در سطح بالا تر مثلا
xcos+xsin=دو به توان ایکس
اینا همش مثال بود اینا که سهله سخت تر بدین ولی همینجوری واضح و شفاف

با سلام

تمام اعداد صحیح ناصفر x و y و z را که در معادله ی زیر صدق می کنند، پیدا کنید:


[ برای مشاهده لینک ، لطفا با نام کاربری خود وارد شوید یا ثبت نام کنید ] x^3+y^3=z^3

موفق باشید.

24 تیر ماه 1390

Headphone
15-07-2011, 11:13
سلام
یه سوال داشتم : مشتق 2 به توان x چطوریه؟ یه دانش آموز کنکوری میتونه بدستش بیاره؟

ممنون

Keyser Söze
15-07-2011, 11:19
سلام
یه سوال داشتم : مشتق 2 به توان x چطوریه؟ یه دانش آموز کنکوری میتونه بدستش بیاره؟

ممنون
مشتق توابع نمايي رو توي دبيرستان نميخونيم.
فكر كنم دانشگاه كه بريم تو رياضي عمومي هامون داريم.

lebesgue
15-07-2011, 12:59
سلام
یه سوال داشتم : مشتق 2 به توان x چطوریه؟ یه دانش آموز کنکوری میتونه بدستش بیاره؟

ممنون

بله، یک دانش آموز با اطلاعات پیش دانشگاهی ریاضی میتواند آن را به صورت زیر استنتاج کند:


[ برای مشاهده لینک ، لطفا با نام کاربری خود وارد شوید یا ثبت نام کنید ](x)=2^x%5CRightarrow%20f^{-1}(y)=%5Clog_{2}^{y}


بنا به روابط لگاریتم میتوان نوشت:

[ برای مشاهده لینک ، لطفا با نام کاربری خود وارد شوید یا ثبت نام کنید ]^{-1}(y)=%5Clog_{2}^{y}=%5Cfrac{%5Clog_{e}^{y}}{%5Clo g_{e}^{2}}=%5Cfrac{%5Cln(y)}{%5Cln(2)}%5C%5C%5C%5C %20%5CRightarrow%20(f^{-1})'(y)=%5Cfrac{1}{%5Cln(2)}%5Cfrac{1}{y}

با استفاده از رابطه مشتق تابع معکوس میتوان نتیجه گرفت:

[ برای مشاهده لینک ، لطفا با نام کاربری خود وارد شوید یا ثبت نام کنید ]'(x)=%5Cfrac{1}{%20(f^{-1})'(y)}=%5Cln(2)y%5C%5C%5C%5C%20y=2^x%5CRightarro w%20f'(x)=%5Cln(2)2^x


نتیجه کلی برای a>0 به صورت زیر است:

[ برای مشاهده لینک ، لطفا با نام کاربری خود وارد شوید یا ثبت نام کنید ](x)=a^x%5CRightarrow%20f'(x)=%5Cln(a)a^ x

5233180
16-07-2011, 11:45
سلام دوستان ببخشید بنده یه سوال داشتم

راهی هست که سریع بشه فهمید که مثلا عدد دو باید به توان چند برسه تا بشه 256؟؟؟
یه فرمولی چیزی منظورم هست که نخوام هی ضرب کنم تا به اون عدد برسم
2 باید به توان 8 برسه
ولی من راه سریعترش رو بلد نیستم

ممنون میشم راهنماییم کنید

davy jones
16-07-2011, 12:02
سلام دوستان ببخشید بنده یه سوال داشتم

راهی هست که سریع بشه فهمید که مثلا عدد دو باید به توان چند برسه تا بشه 256؟؟؟
یه فرمولی چیزی منظورم هست که نخوام هی ضرب کنم تا به اون عدد برسم
2 باید به توان 8 برسه
ولی من راه سریعترش رو بلد نیستم

ممنون میشم راهنماییم کنید

سلام.
راه اصولیش اینه که از عدد داده شده (مثلا در اینجا 256) در پایه ی داده شده (که در اینجا همان عدد 2 است) لگاریتم بگیریم. در حالت کلی میخوایم ببینیم که عدد x را به چه توانی برسانیم که حاصل برابر با عدد y شود؟ جواب برابر با مقدار:

[ برای مشاهده لینک ، لطفا با نام کاربری خود وارد شوید یا ثبت نام کنید ] =%5Cfrac%7B%5Cln&space;y%7D%7B%5Cln&space;x%7D

است.

موفق باشین.
90/4/25

Keyser Söze
16-07-2011, 12:21
سلام دوستان ببخشید بنده یه سوال داشتم

راهی هست که سریع بشه فهمید که مثلا عدد دو باید به توان چند برسه تا بشه 256؟؟؟
یه فرمولی چیزی منظورم هست که نخوام هی ضرب کنم تا به اون عدد برسم
2 باید به توان 8 برسه
ولی من راه سریعترش رو بلد نیستم

ممنون میشم راهنماییم کنید


من خودم 2 به توان10 رو حفظ كرده بودم كه ميشه 1024 و براي توانهاي بالا از اون استفاده ميكردم و با تقسيم بر دو تبديلش ميكردم به توانهاي كمتر. (مثلا 2 به توان 9 پس ميشه 512 و...)
بهتره 2 به توان5 رو هم حفظ كني كه ميشه 32 اون وقت دسترسي به بقيه خيلي برات آسون ميشه.

Headphone
22-07-2011, 20:43
سلام

یه سوال دیگه :

حدود k برای اینکه تابع [ برای مشاهده لینک ، لطفا با نام کاربری خود وارد شوید یا ثبت نام کنید ] تابعی پیوسته باشد ، را بدست آورید ؟

ممنون

hts1369
23-07-2011, 08:31
سلام

یه سوال دیگه :

حدود k برای اینکه تابع [ برای مشاهده لینک ، لطفا با نام کاربری خود وارد شوید یا ثبت نام کنید ] تابعی پیوسته باشد ، را بدست آورید ؟

ممنون
باید دامنه ی تابعی که دادی رو حساب بکنی که میشه[4,4-]

davy jones
23-07-2011, 13:49
سلام

یه سوال دیگه :

حدود k برای اینکه تابع [ برای مشاهده لینک ، لطفا با نام کاربری خود وارد شوید یا ثبت نام کنید ] تابعی پیوسته باشد ، را بدست آورید ؟

ممنون
باید دلتای عبارت زیر رادیکال همواره کوچکتر مساوی با صفر باشه و ضمنا خود k هم عددی مثبت باشه تا عبارت درجه ی 2 زیر رادیکال همواره در بالای محور xها باشه. بنابراین داریم:


[ برای مشاهده لینک ، لطفا با نام کاربری خود وارد شوید یا ثبت نام کنید ] q&space;4%5CRightarrow&space;%5Cleft%5C%7B%5Cbegin%7Bmatrix%7D &space;k%5Cgeq&space;2%5C%5C&space;k%5Cleq&space;-2%5Crightarrow&space;unacceptable&space;%5Cend%7Bmatrix%7D%5Cr ight.%5CRightarrow&space;%7B%5Ccolor%7BRed%7D&space;k%5Cgeq&space;2% 7D



موفق باشین.
90/5/1

Headphone
23-07-2011, 15:22
باید دلتای عبارت زیر رادیکال همواره کوچکتر مساوی با صفر باشه و ضمنا خود k هم عددی مثبت باشه تا عبارت درجه ی 2 زیر رادیکال همواره در بالای محور xها باشه. بنابراین داریم:


[ برای مشاهده لینک ، لطفا با نام کاربری خود وارد شوید یا ثبت نام کنید ] q&space;4%5CRightarrow&space;%5Cleft%5C%7B%5Cbegin%7Bmatrix%7D &space;k%5Cgeq&space;2%5C%5C&space;k%5Cleq&space;-2%5Crightarrow&space;unacceptable&space;%5Cend%7Bmatrix%7D%5Cr ight.%5CRightarrow&space;%7B%5Ccolor%7BRed%7D&space;k%5Cgeq&space;2% 7D



موفق باشین.
90/5/1



ممنون

ولی اگه بجای k مثلا 0 قرار بدیم تابع ما به صورت [ برای مشاهده لینک ، لطفا با نام کاربری خود وارد شوید یا ثبت نام کنید ] در میاد که تابعی پیوسته است!!!

من این سوالو تو یه کتاب تست خوندم ، تو جوابش نوشته ، در این چندجمله ای نباید همزمان [ برای مشاهده لینک ، لطفا با نام کاربری خود وارد شوید یا ثبت نام کنید ] و [ برای مشاهده لینک ، لطفا با نام کاربری خود وارد شوید یا ثبت نام کنید ] برقرار باشند.... جوابش رو هم آورده [ برای مشاهده لینک ، لطفا با نام کاربری خود وارد شوید یا ثبت نام کنید ] ......

حالا من علتشو نمیفهمم......:13:

davy jones
23-07-2011, 22:49
ممنون

ولی اگه بجای k مثلا 0 قرار بدیم تابع ما به صورت [ برای مشاهده لینک ، لطفا با نام کاربری خود وارد شوید یا ثبت نام کنید ] در میاد که تابعی پیوسته است!!!


تابع رادیکال به شرطی پیوسته است که عبارت داخل اون به ازای جمیع مقادیر مثبت باشه. تابع 4x فقط به ازای x>0 مثبته و بنابراین تابع [ برای مشاهده لینک ، لطفا با نام کاربری خود وارد شوید یا ثبت نام کنید ] در کل اعداد حقیقی پیوسته نیست و در X<0 چون مقدار زیر رادیکال منفیه بنابراین تابع مقدار تعریف نشده داره.


من این سوالو تو یه کتاب تست خوندم ، تو جوابش نوشته ، در این چندجمله ای نباید همزمان [ برای مشاهده لینک ، لطفا با نام کاربری خود وارد شوید یا ثبت نام کنید ] و [ برای مشاهده لینک ، لطفا با نام کاربری خود وارد شوید یا ثبت نام کنید ] برقرار باشند.... جوابش رو هم آورده [ برای مشاهده لینک ، لطفا با نام کاربری خود وارد شوید یا ثبت نام کنید ] ......

حالا من علتشو نمیفهمم......[ برای مشاهده لینک ، لطفا با نام کاربری خود وارد شوید یا ثبت نام کنید ]


اینکه باید دلتا منفی یا صفر باشه به دلیل اینه که تابع درجه ی 2 ی ما که در زیر رادیکال قرار داره دارای ریشه ی ساده (ریشه ی غیر مضاعف) نباشه. چون ریشه در هر تابع، نمایانگر تغییر علامت مقدار تابع قبل و بعد از نقطه ی ریشه خواهد بود. من هم در پست قبلی روی همین حساب دلتا رو منفی قرار دادم.

[ برای مشاهده لینک ، لطفا با نام کاربری خود وارد شوید یا ثبت نام کنید ]

با این شرط دو محدوده برای k به دست میآد. خود مقدار k هم باید حتما مثبت باشه چون همونطور که میدونیم علامت ضریب جمله ی x^2 تعیین کننده ی علامت کل تابع در عبارات درجه 2 ایست که ریشه ندارند. اگر ضریب عبارت x^2 مثبت باشه، کل تابع در محدوده بالای محور xها قرار میگیره و اگه ضریب x^2 منفی باشه، کل تابع منفیه. با اعمال شرط دوم یکی از محدوده ها غیر قابل قبول خواهد بود.

نقل از خودم:

باید دلتای عبارت زیر رادیکال همواره کوچکتر مساوی با صفر باشه و ضمنا خود k هم عددی مثبت باشه تا عبارت درجه ی 2 زیر رادیکال همواره در بالای محور xها باشه.

به نظرم جوابی که در پست قبلی به دست آوردم درسته و کتاب تست شما غلط تایپی داشته و 2 رو اشتباها 2- نوشته.


موفق باشین.
90/5/1

Headphone
24-07-2011, 07:04
آره شما درست میگید

ولی برای ما اومدن کتاب حسابان رو عوض کردن ، در نتیجه بعضی از تعریف ها عوض شده ، مثلاهمون تابع[ برای مشاهده لینک ، لطفا با نام کاربری خود وارد شوید یا ثبت نام کنید ] چون به ازای x<0 تعریف نشده ، پس نمیتونیم در مورد حد چپش صحبت کنیم و منظور از حد در این تابع همون حد راسته !!:31: پیوستگی هم همینطور ! تابع [ برای مشاهده لینک ، لطفا با نام کاربری خود وارد شوید یا ثبت نام کنید ] به ازای نقاط دامنه اش پیوسته است


کلا زدن داغون کردن ریاضیات مارو!!:41:

ممنون از این که وقت با ارزشتون رو صرف جواب دادن به سوال من کردید:11:

13!
25-07-2011, 13:23
من نمیدونم جاش اینجا هست یا نه، در هر حال یه سوالی دارم. میدونیم که باقیمونده چندجمله ای به دوجمله ای بدون انجام عمل تقسیم، رو باید با پیدا کردن ریشه های دوجمله ای و جایگذاریشون تو چندجمله ای بدست بیاریم. حالا سوال من اینه که این روش چجوری بوجود اومده و چجوری رسیدن به این نکته؟ در واقع نحوه اثبات این قضیه رو میخوام.

mofidy1
26-07-2011, 22:32
من نمیدونم جاش اینجا هست یا نه، در هر حال یه سوالی دارم. میدونیم که باقیمونده چندجمله ای به دوجمله ای بدون انجام عمل تقسیم، رو باید با پیدا کردن ریشه های دوجمله ای و جایگذاریشون تو چندجمله ای بدست بیاریم. حالا سوال من اینه که این روش چجوری بوجود اومده و چجوری رسیدن به این نکته؟ در واقع نحوه اثبات این قضیه رو میخوام.

با سلام

اثبات سختی ندارد. فرض کنید که چند جمله ای p را بر دو جمله ای q تقسیم کرده ایم و خارج قسمت s و باقی مانده ی r به دست آمده باشد؛ بنابر این p=qs+r، حال اگر ریشه ی دو جمله ای را در این تساوی - و به صورت دقیق تر، در این اتحاد - قرار دهید، طرف راست، r - که یک عدد است - و طرف دیگر، مقدار p به ازای ریشه ی چند جمله ای به دست می آید، زیرا با این جایگذاری، مقدار qs صفر خواهد شد.

موفق باشید.

4 مرداد 1390

hts1369
28-07-2011, 13:24
امروز کنکور کاردانی به کارشناسی داشتیم یه چندتا از سوالها رو از دوستان میپرسم اگه جواب بدن ممنون میشم.
1-دامنه ی این تابع را تعیین کنید.[ برای مشاهده لینک ، لطفا با نام کاربری خود وارد شوید یا ثبت نام کنید ]
2-تابع داده شده در چه محدوده ای معکوس پذیر است و معکوس ان کدام است.[ برای مشاهده لینک ، لطفا با نام کاربری خود وارد شوید یا ثبت نام کنید ]
فعلا اینها رو یادم میاد بازم یادم اومد میزارم دمتون گرم ممنون

eli*
31-07-2011, 00:23
با سلام ، می خواستم بدونم spray در ریاضی به چه معناست ؟

aqronis
31-07-2011, 12:00
درود
دوستان بنده میخوام بدونم با استفاده از چه روشی میشه معادله بهترین صفحه بین 4 نقطه (که روی یک صفحه نیستن) رو بدست آورد. منظور بهترین صفحه ای است که کمترین فاصله از نقاط را داشته باشد.
لطفا راهنمایی کنید.
با تشکر.

hadi2222
31-07-2011, 14:45
سلام دوستان



من 20 تا معادله دیفرانسیل دارم که حلشو نیاز دارم خیلی هم نیاز دارم اما از بچه های خودمون کسی نمی تونه حلشون کنه ( رشتم عمرانه)
شما نرم افزاری سایتی چیزی که بتونید باهاش اینا رو حل کنید و بهم بدین سراغ دارین ؟
البته مسیر حلش رو هم می خوام هااااااا wolfarmalpha مسیر حلش رو نمیده

همشون مرتبه اول هستن و 3 4 تاشون مرتبه دوم

پگاه م
01-08-2011, 09:16
سلام
دوستان من دنبال اثبات قضیه کوشی ریمان از راه قطبی هستم.تو کتابا فقط اثبات دکارتیش هست.ممنون میشم اگه کمکم کنید.

hts1369
01-08-2011, 17:59
امروز کنکور کاردانی به کارشناسی داشتیم یه چندتا از سوالها رو از دوستان میپرسم اگه جواب بدن ممنون میشم.
1-دامنه ی این تابع را تعیین کنید.[ برای مشاهده لینک ، لطفا با نام کاربری خود وارد شوید یا ثبت نام کنید ]
2-تابع داده شده در چه محدوده ای معکوس پذیر است و معکوس ان کدام است.[ برای مشاهده لینک ، لطفا با نام کاربری خود وارد شوید یا ثبت نام کنید ]
فعلا اینها رو یادم میاد بازم یادم اومد میزارم دمتون گرم ممنون
نبود. اقای مفیدی و یا بقیه ی بچه ها کجا هستن؟؟

Headphone
01-08-2011, 18:54
امروز کنکور کاردانی به کارشناسی داشتیم یه چندتا از سوالها رو از دوستان میپرسم اگه جواب بدن ممنون میشم.
1-دامنه ی این تابع را تعیین کنید.[ برای مشاهده لینک ، لطفا با نام کاربری خود وارد شوید یا ثبت نام کنید ]
2-تابع داده شده در چه محدوده ای معکوس پذیر است و معکوس ان کدام است.[ برای مشاهده لینک ، لطفا با نام کاربری خود وارد شوید یا ثبت نام کنید ]
فعلا اینها رو یادم میاد بازم یادم اومد میزارم دمتون گرم ممنون
اولی رو که فعلا بلد نیستم

اما دومی:
در این تابع باید داخل قدر مطلق نامثبت باشه ، چون اگه داخل قدر مطلق مثبت باشه تابع ما بصورت [ برای مشاهده لینک ، لطفا با نام کاربری خود وارد شوید یا ثبت نام کنید ] در میاد که تابعی معکوس ناپذیره پس باید :


[ برای مشاهده لینک ، لطفا با نام کاربری خود وارد شوید یا ثبت نام کنید ] %20x%5Cleqslant%202%7D


بدست آوردن معکوسش :
[ برای مشاهده لینک ، لطفا با نام کاربری خود وارد شوید یا ثبت نام کنید ]

hts1369
01-08-2011, 20:03
اولی رو که فعلا بلد نیستم

اما دومی:
در این تابع باید داخل قدر مطلق نامثبت باشه ، چون اگه داخل قدر مطلق مثبت باشه تابع ما بصورت [ برای مشاهده لینک ، لطفا با نام کاربری خود وارد شوید یا ثبت نام کنید ] در میاد که تابعی معکوس ناپذیره پس باید :


[ برای مشاهده لینک ، لطفا با نام کاربری خود وارد شوید یا ثبت نام کنید ] %20x%5Cleqslant%202%7D


بدست آوردن معکوسش :
[ برای مشاهده لینک ، لطفا با نام کاربری خود وارد شوید یا ثبت نام کنید ]






ایول به تو منم برا دومی همین جواب رو بدست اوردم ولی یه گزینه بود که محدوده ی من رو بدست اورده بود ولی ضابطه درست نبود یکی دیگه همین ضابطه رو بدست اورده بود ولی محدوده ی معکوس پذیری رو چیز دیگه ای نوشته بود.

lebesgue
01-08-2011, 21:24
1-دامنه ی این تابع را تعیین کنید.[ برای مشاهده لینک ، لطفا با نام کاربری خود وارد شوید یا ثبت نام کنید ]


تابع [ برای مشاهده لینک ، لطفا با نام کاربری خود وارد شوید یا ثبت نام کنید ]^{-1} در اعداد حقیقی، تنها برای بازه ی (1,1-) تعریف می شود، در نتیجه دامنه تابع بالا تهی است.


درود
دوستان بنده میخوام بدونم با استفاده از چه روشی میشه معادله بهترین صفحه بین 4 نقطه (که روی یک صفحه نیستن) رو بدست آورد. منظور بهترین صفحه ای است که کمترین فاصله از نقاط را داشته باشد.
لطفا راهنمایی کنید.
با تشکر.
فرض کنید m نقطه دارید با مختصات [ برای مشاهده لینک ، لطفا با نام کاربری خود وارد شوید یا ثبت نام کنید ](x_i,y_i,z_i) برای i = 1,2,...,m.
اگر z = Ax + By + C معادله صفحه کمترین مربعات باشد، آنگاه میتوان نشان داد که B، A و C از حل دستگاه زیر بدست می آیند:

[ برای مشاهده لینک ، لطفا با نام کاربری خود وارد شوید یا ثبت نام کنید ]


سلام
دوستان من دنبال اثبات قضیه کوشی ریمان از راه قطبی هستم.تو کتابا فقط اثبات دکارتیش هست.ممنون میشم اگه کمکم کنید.

برای مشاهده محتوا ، لطفا وارد شوید یا ثبت نام کنید

skyzare
02-08-2011, 11:39
با سلام ........:)

من یه سوال داشتم ...میخواستم بدونم دو تا ماتریس رو اگه داشته باشیم و بخوایم

داخل هم کانولوشن کنیم باید از چه فرمولی استفاده کنیم ؟؟؟ و این که ماتریس حاصل

چی رو به ما نشوم میده ؟؟

با تشکر از پاسختون .... .:20:

hts1369
03-08-2011, 20:56
با توجه به تعریف[ برای مشاهده لینک ، لطفا با نام کاربری خود وارد شوید یا ثبت نام کنید ] و [ برای مشاهده لینک ، لطفا با نام کاربری خود وارد شوید یا ثبت نام کنید ] مقدار [ برای مشاهده لینک ، لطفا با نام کاربری خود وارد شوید یا ثبت نام کنید ] کدام است؟
دو خط راست موازی نیمساز ناحیه دوم بر منحنی به معادله [ برای مشاهده لینک ، لطفا با نام کاربری خود وارد شوید یا ثبت نام کنید ] مماس هستند. فاصله این دو خط کدام است؟

mofidy1
03-08-2011, 23:39
با سلام ........:)

من یه سوال داشتم ...میخواستم بدونم دو تا ماتریس رو اگه داشته باشیم و بخوایم

داخل هم کانولوشن کنیم باید از چه فرمولی استفاده کنیم ؟؟؟ و این که ماتریس حاصل

چی رو به ما نشوم میده ؟؟

با تشکر از پاسختون .... .:20:

با سلام

اگر منظور شما همان کانولوشنی است که در پردازش تصاویر استفاده می شود، فکر می کنم بهتر است آن را در انجمن رایانه و برنامه نویسی سایت مطرح فرمایید. البته در اینترنت هم مقالات خوبی در این زمینه پیدا می شود.

موفق باشید.

13 مرداد 1390

mofidy1
04-08-2011, 00:28
با توجه به تعریف[ برای مشاهده لینک ، لطفا با نام کاربری خود وارد شوید یا ثبت نام کنید ] و [ برای مشاهده لینک ، لطفا با نام کاربری خود وارد شوید یا ثبت نام کنید ] مقدار [ برای مشاهده لینک ، لطفا با نام کاربری خود وارد شوید یا ثبت نام کنید ] کدام است؟
دو خط راست موازی نیمساز ناحیه دوم بر منحنی به معادله [ برای مشاهده لینک ، لطفا با نام کاربری خود وارد شوید یا ثبت نام کنید ] مماس هستند. فاصله این دو خط کدام است؟

با سلام

برای مساله ی اول از لگاریتم نپری استفاده کنید و حاصل ضرب ایجاد شده را به کسر تبدیل و از قاعده ی هوپیتال استفاده کنید. البته برای این که تابع درست تعریف شود، باید g مثبت باشد و لذا باید حد از سمت چپ به pi/2 نزدیک شود.

برای مساله ی دوم از مشتق ضمنی استفاده کنید و توجه کنید که 'y باید 1- باشد.

موفق باشید.

13 مرداد 1390

hts1369
04-08-2011, 07:29
با سلام

برای مساله ی اول از لگاریتم نپری استفاده کنید و حاصل ضرب ایجاد شده را به کسر تبدیل و از قاعده ی هوپیتال استفاده کنید. البته برای این که تابع درست تعریف شود، باید g مثبت باشد و لذا باید حد از سمت چپ به pi/2 نزدیک شود.

برای مساله ی دوم از مشتق ضمنی استفاده کنید و توجه کنید که 'y باید 1- باشد.

موفق باشید.

13 مرداد 1390
دست شما درد نکنه اقای مفیدی ولی من راه حل اینها رو میدونم ولی در مورد اولی هر کاری کردم به جواب نرسیدم ودر مورد دومی به جوابی که تو گزینه ها بود رسیدم ولی اطمینان ندارم اگه شما و یا دوستان دیگه راه حل و جواب اخر رو بدن ممنون میشم.

skyzare
04-08-2011, 12:03
با سلام

اگر منظور شما همان کانولوشنی است که در پردازش تصاویر استفاده می شود، فکر می کنم بهتر است آن را در انجمن رایانه و برنامه نویسی سایت مطرح فرمایید. البته در اینترنت هم مقالات خوبی در این زمینه پیدا می شود.
موفق باشید.


با سلام ....

با تشکر از پاسختون .....

اره دقیقا برای پردازش تصویر میخواستم .... یه سوال دیگه من دنبال یه کتاب ریاضی میگردم که مباحث ریاضی رو

توی حوزه گسسته بیان کرده باشه ...مثلا همین مشتق ماتریس ؛ گردایانش ؛ و غیره ....داخلش باشه ...کتابی

که رشته های راضی توی دبیرستان میخون که ریاضی گسسته اسمش هست رو یه نگاه کردم ولی اون چیزایی

که من میخواستم توش نبود ....

mofidy1
04-08-2011, 13:57
با سلام ....

با تشکر از پاسختون .....

اره دقیقا برای پردازش تصویر میخواستم .... یه سوال دیگه من دنبال یه کتاب ریاضی میگردم که مباحث ریاضی رو

توی حوزه گسسته بیان کرده باشه ...مثلا همین مشتق ماتریس ؛ گردایانش ؛ و غیره ....داخلش باشه ...کتابی

که رشته های راضی توی دبیرستان میخون که ریاضی گسسته اسمش هست رو یه نگاه کردم ولی اون چیزایی

که من میخواستم توش نبود ....

با سلام

شما مباحثی که می خواهید به طور دقیق نام ببرید، شاید توانستیم کمکتان کنیم.

موفق باشید.13 مرداد 1390

skyzare
05-08-2011, 09:51
با سلام

شما مباحثی که می خواهید به طور دقیق نام ببرید، شاید توانستیم کمکتان کنیم.

موفق باشید.13 مرداد 1390

سلام ....:20:

خیلی ممنون از لطفتون .....:20:

من فعلا فقط 4 مورد رو میخوام ....البته خودم هم یه چیزایی دستم اومده ولی نه کامل ....

1- مشتق گرفتن از یه ماتریس

2- کانولوشن یه ماتریس

3- انحراف استاندارد یه ماتریس

4-گرادیان و لاپلاس یه ماتریس ؛ که فکر کنم مورد اول رو یعنی مشتق یه ماتریس رو بفهمم این دو تا هم حل بشه

در مورد کانولوشن تا حدودی به نتیجه رسیدم اما وقتی با توابع متلب چک میکنم توی تابع یه بعدی اش محاسبه

دستی ام درست جواب نمیده ...

محاسبه کانولوشن یک ماتریس یک بعدی از نابع conv

محاسبه کانولوشن یک ماتریس دو بعدی از تابع conv2

که هر کدام سه حالت مختلف full ؛ same ؛valid رو داره ...

مشتق هم یه چیزایی فهمیدم ..... ولی خیلی ناقص

از انحراف استاندار د هم هیچی نمیدونم !! :41: البته یه چیزای میدونم مثلا تابعی که داخل متلب ازش استفاده میکنه اسمش stdfilt

هست من خودم هلپ ایت تابع رو هم یه نگاه کردم ولی توضیحی درباره این که محاسبه اش چه جوری هست نداده بود ...نمیدونم شایدمن

درست متوجه نشدم ...:37:

arash_kk
06-08-2011, 00:31
سلام
لطفا راهنمایی نمایید.

[ برای مشاهده لینک ، لطفا با نام کاربری خود وارد شوید یا ثبت نام کنید ][]{5}~~,~~%20a_{n+1}=a_n^2~.%20~~~~then%20~~~~\lim_{ x\rightarrow%20\infty%20}%20\frac{a_1a_2...a_n}{a_ {n+1}}=~???

lebesgue
06-08-2011, 14:25
[ برای مشاهده لینک ، لطفا با نام کاربری خود وارد شوید یا ثبت نام کنید ]{120}%20%5Cfrac{a_1a_2a_3...a_n}{a _{n+1}}=%5Cfrac{a_1^{2^0}a_1^{2^1}a_1^{2^2}...a_1^ {2^{n-1}}}{a_1^{2^n}}=%5Cfrac{a_1^{2^0+2^1+2^2+%5Ccdots% 202^{n-1}}}{a_1^{2^n}}=%5Cfrac{a_1^{2^{n}-1}}{a_1^{2^n}}=%5Cfrac{1}{a_1}

ALt3rnA
06-08-2011, 23:13
سلام ! یه فرمول جزء صحیح معروف داریم :

[a] + [b] = [a+b]
یا
[a+b] منهای یک

بنده سوالم اینه این فرمول چرا هیچوقت کار نمیکنه برا من !؟
مثلا این سوال

[ برای مشاهده لینک ، لطفا با نام کاربری خود وارد شوید یا ثبت نام کنید ] 10806_203904.jpg



که جواب غلطیست و ایکس بین 1/2 و 2/3 هست
مشکل کجاست ؟

ادیت : 2/3 نیست 2/5 هست تو عکس

hts1369
07-08-2011, 12:05
سلام ! یه فرمول جزء صحیح معروف داریم :

[a] + [b] = [a+b]
یا
[a+b] منهای یک

بنده سوالم اینه این فرمول چرا هیچوقت کار نمیکنه برا من !؟
مثلا این سوال

[ برای مشاهده لینک ، لطفا با نام کاربری خود وارد شوید یا ثبت نام کنید ] 10806_203904.jpg



که جواب غلطیست و ایکس بین 1/2 و 2/3 هست
مشکل کجاست ؟

ادیت : 2/3 نیست 2/5 هست تو عکس






فرمولی که شما نوشتی فقط برای اعداد صحیح کاربرد داره و در مورد اعداد اعشاری اشتباه هست.

lebesgue
07-08-2011, 12:14
سلام ! یه فرمول جزء صحیح معروف داریم :

[a] + [b] = [a+b]
یا
[a+b] منهای یک

بنده سوالم اینه این فرمول چرا هیچوقت کار نمیکنه برا من !؟
مثلا این سوال

[ برای مشاهده لینک ، لطفا با نام کاربری خود وارد شوید یا ثبت نام کنید ] 10806_203904.jpg



که جواب غلطیست و ایکس بین 1/2 و 2/3 هست
مشکل کجاست ؟

ادیت : 2/3 نیست 2/5 هست تو عکس





خب تحت شرایط مشخصی برای a و b، معادله اول صادق بوده و در غیراینصورت معادله دوم. (رندم که نیست! :31:)
اگر مجموع جزء های ناصحیح a و b کوچکتر از 1 باشد، معادله اول برقرار است و در غیراینصورت معادله دوم.
با در نظر گرفتن این شرایط، محدودیتهایی برای x در هر معادله بدست می آید که با اعمال آنها به جواب صحیح می رسید.

ali1234
07-08-2011, 12:14
سلام
جواب اين حد رو مي خواستم :



tan x - sin x ) / x^3 )


وقتي كه ايكس به سمت صفر ميل ميكنه .

ALt3rnA
07-08-2011, 12:45
فرمولی که شما نوشتی فقط برای اعداد صحیح کاربرد داره و در مورد اعداد اعشاری اشتباه هست.
خیر عزیز اشتباه میکنی
بر اساس اینکه مجموع اعشار بیشتر از 1 شه یا کمتر فرمول دو ضابطه داره



خب تحت شرایط مشخصی برای a و b، معادله اول صادق بوده و در غیراینصورت معادله دوم. (رندم که نیست! [ برای مشاهده لینک ، لطفا با نام کاربری خود وارد شوید یا ثبت نام کنید ])
اگر مجموع جزء های ناصحیح a و b کوچکتر از 1 باشد، معادله اول برقرار است و در غیراینصورت معادله دوم.
با در نظر گرفتن این شرایط، محدودیتهایی برای x در هر معادله بدست می آید که با اعمال آنها به جواب صحیح می رسید.
یعنی قسمتی از جواب تو اولی میوفته و قسمتیش تو دومی ؟ این یعنی به درد حل معادله نمیخوره ؟
پس معادلات اینجوری چطور حل میشن ؟
حالا این دوتا جزء صحیح داره . تو کتاب دیف آقای عسلی یه سوال داده که سه تا جز صحیح داره .
من به شخصه با تعریف جزء صحیح میرم و یه محدوده به دست میاد و تو اون محدوده اعداد رو جایگذاری میکنم که این فقط به درد سوالایی میخوره که جواب منفرد دارن و نه مجوعه جواب ! چیکار کنم ؟

hts1369
07-08-2011, 13:31
سلام
جواب اين حد رو مي خواستم :



tan x - sin x ) / x^3 )


وقتي كه ايكس به سمت صفر ميل ميكنه .
من لقمه رو برات جویدم گذاشتم دهنت
[ برای مشاهده لینک ، لطفا با نام کاربری خود وارد شوید یا ثبت نام کنید ]

ALt3rnA
07-08-2011, 16:15
سلام
جواب اين حد رو مي خواستم :



tan x - sin x ) / x^3 )


وقتي كه ايكس به سمت صفر ميل ميكنه .
سلام
میتونید با هم ارزی مرتبه دو به راحتی حلش کنید

dkhatibi
07-08-2011, 16:44
نمونه درست تر سوال اینه
[ برای مشاهده لینک ، لطفا با نام کاربری خود وارد شوید یا ثبت نام کنید ]{n+1}=a_n^2-2 صحیح است.

[ برای مشاهده لینک ، لطفا با نام کاربری خود وارد شوید یا ثبت نام کنید ][]{5}~~,~~%20a_{n+1}=a_n^2-2~.%20~~~~then%20~~~~\lim_{x\rightarrow%20\infty%2 0}%20\frac{a_1a_2...a_n}{a_{n+1}}=~???
می توانید اینجا ببینید.

برای مشاهده محتوا ، لطفا وارد شوید یا ثبت نام کنید

مشاهده ی رایگان سوالات آزمون دکترا در

برای مشاهده محتوا ، لطفا وارد شوید یا ثبت نام کنید

neon_91
07-08-2011, 19:43
سلام من روش حل معادله بازگشتی رو می خواستم معادله هم این است




[ برای مشاهده لینک ، لطفا با نام کاربری خود وارد شوید یا ثبت نام کنید ]


در ضمن روش حل را هم در اینجا قرار بدید ممنون میشم:11:

hts1369
07-08-2011, 19:51
سلام
میتونید با هم ارزی مرتبه دو به راحتی حلش کنید
استفاده از هم ارزی در مورد این حد درست نیست.اگه از هم ارزی استفاده کنید جواب میشه یک تقسیم بر 3 که اشتباهه

ALt3rnA
07-08-2011, 20:47
استفاده از هم ارزی در مورد این حد درست نیست.اگه از هم ارزی استفاده کنید جواب میشه یک تقسیم بر 3 که اشتباهه
شما خودتون دوبار هم ارزی استفاده کردید یه بار قبل هوپیتال یه بار بعدش . چرا اونجا شد و اینجا نه ؟
با هم ارزی هم حساب کنید میشه همون 1/2 ! میشه 1/3 + 1/6 که میشه 1/2 ام ! شما چطوری حساب کردید ؟

من برای این سوال بار اول هوپیتال زدم (بدون ساده سازی) و هم ارزی 1/3 درومد
بار دوم که هوپیتال گرفتم+هم ارزی شد 1/2 !
ظاهرا نمیشه بعضی وقتا قبل از هوپیتال هم ارزی زد :-؟

lebesgue
07-08-2011, 22:54
سلام
جواب اين حد رو مي خواستم :



tan x - sin x ) / x^3 )


وقتي كه ايكس به سمت صفر ميل ميكنه .

هم ارزیهای سینوس و تانژانت و... از سری تیلور آنها بدست می آید.
میتوان به طور مستقیم سری تیلور را جایگذاری کرد. سری تیلور سینوس و تانژانت حول صفر، به صورت زیر است:

[ برای مشاهده لینک ، لطفا با نام کاربری خود وارد شوید یا ثبت نام کنید ]

[ برای مشاهده لینک ، لطفا با نام کاربری خود وارد شوید یا ثبت نام کنید ]
در نتیجه سری تیلور tanx - sinx بدست می آید:

[ برای مشاهده لینک ، لطفا با نام کاربری خود وارد شوید یا ثبت نام کنید ]{120}%20%5Ctan%20x-%5Csin%20x=%5Cfrac{x^3}{2}+%5Cfrac{x^5}{8}+%5Ccdot s
و در نتیجه:

[ برای مشاهده لینک ، لطفا با نام کاربری خود وارد شوید یا ثبت نام کنید ]{120}%20%5Cfrac{%5Ctan%20x-%5Csin%20x}{x^3}=%5Cfrac{1}{2}+%5Cfrac{x^2}{8}+%5C cdots

حال کافی است که حد این عبارت محاسبه شود. در اینجا میتوان دید که ببرداشتن دو جمله اول سری تیلور -به عنوان یک هم ارزی- پاسخ صحیحی برای این حد می دهد.
البته درباره همگرایی سریها باید محتاط بود.

lebesgue
07-08-2011, 23:06
یعنی قسمتی از جواب تو اولی میوفته و قسمتیش تو دومی ؟ این یعنی به درد حل معادله نمیخوره ؟
پس معادلات اینجوری چطور حل میشن ؟
حالا این دوتا جزء صحیح داره . تو کتاب دیف آقای عسلی یه سوال داده که سه تا جز صحیح داره .
من به شخصه با تعریف جزء صحیح میرم و یه محدوده به دست میاد و تو اون محدوده اعداد رو جایگذاری میکنم که این فقط به درد سوالایی میخوره که جواب منفرد دارن و نه مجوعه جواب ! چیکار کنم ؟
- بله، همینطور است.
- در مورد این معادله، تابع [f(x) = [2x] + [3x را در نظر بگیرید. این تابع صعودی است. در نتیجه به سادگی میتوان نشان داد که مقدار تابع به ازای x<1/2 کمتر از 2 و به ازای x≥2/3 بزرگتر از 2 بوده
و همچنین در بازه (2/3 , 1/2] برابر 2 است.
- آن سوالی که میفرمایید را در صورت تمایل، بنویسید.

lebesgue
07-08-2011, 23:19
نمونه درست تر سوال اینه
[ برای مشاهده لینک ، لطفا با نام کاربری خود وارد شوید یا ثبت نام کنید ]{n+1}=a_n^2-2 صحیح است.

[ برای مشاهده لینک ، لطفا با نام کاربری خود وارد شوید یا ثبت نام کنید ][]{5}~~,~~%20a_{n+1}=a_n^2-2~.%20~~~~then%20~~~~\lim_{x\rightarrow%20\infty%2 0}%20\frac{a_1a_2...a_n}{a_{n+1}}=~???
می توانید اینجا ببینید.

برای مشاهده محتوا ، لطفا وارد شوید یا ثبت نام کنید

میتوان نشان داد که:

[ برای مشاهده لینک ، لطفا با نام کاربری خود وارد شوید یا ثبت نام کنید ]{120}%20a_n=%5Cvarphi^{2^{n-1}}+(%5Cvarphi-1)^{2^{n-1}}
و در نتیجه برای n>1:

[ برای مشاهده لینک ، لطفا با نام کاربری خود وارد شوید یا ثبت نام کنید ]{120}%20a_n=%5Cleft%20%5Clceil%20% 5Cvarphi^{2^{n-1}}%20%5Cright%20%5Crceil

که φ همان عدد نسبت طلایی [ برای مشاهده لینک ، لطفا با نام کاربری خود وارد شوید یا ثبت نام کنید ](%5Csqrt{5}+1)/2 می باشد. در اینصورت میتوان محاسبه کرد که مقدار حد برابر 1 است.
اما گمان می کنم که باید راه حل سرراست تری هم باشد، آیا این مسئله با کسرهای مسلسل و تقریب اعداد گنگ با اعداد گویا مرتبط نیست؟
پیوند زیر را ببینید:

برای مشاهده محتوا ، لطفا وارد شوید یا ثبت نام کنید

lebesgue
07-08-2011, 23:27
سلام من روش حل معادله بازگشتی رو می خواستم معادله هم این است




[ برای مشاهده لینک ، لطفا با نام کاربری خود وارد شوید یا ثبت نام کنید ]


در ضمن روش حل را هم در اینجا قرار بدید ممنون میشم:11:

آیا منظور شما این است:

[ برای مشاهده لینک ، لطفا با نام کاربری خود وارد شوید یا ثبت نام کنید ]{120}%20a_n=a_{n-1}+%5Cfrac{2.5}{150}a_{n-1}=%5Cfrac{61}{60}a_{n-1}
اگر چنین است، به ازای هیچ مقدار صحیح n، تساوی دوم برقرار نخواهد شد.
به ازای n=58 نزدیک ترین پاسخ به 30000 بدست می آید حدود 29732.
اگر همین مورد نظر شماست، بفرمایید تا راه حل را قرار دهم.

dkhatibi
07-08-2011, 23:50
میتوان نشان داد که:

[ برای مشاهده لینک ، لطفا با نام کاربری خود وارد شوید یا ثبت نام کنید ]{120}%20a_n=%5Cvarphi^{2^{n-1}}+(%5Cvarphi-1)^{2^{n-1}}
و در نتیجه برای n>1:

[ برای مشاهده لینک ، لطفا با نام کاربری خود وارد شوید یا ثبت نام کنید ]{120}%20a_n=%5Cleft%20%5Clceil%20% 5Cvarphi^{2^{n-1}}%20%5Cright%20%5Crceil

که φ همان عدد نسبت طلایی [ برای مشاهده لینک ، لطفا با نام کاربری خود وارد شوید یا ثبت نام کنید ](%5Csqrt{5}+1)/2 می باشد. در اینصورت میتوان محاسبه کرد که مقدار حد برابر 1 است.
اما گمان می کنم که باید راه حل سرراست تری هم باشد، آیا این مسئله با کسرهای مسلسل و تقریب اعداد گنگ با اعداد گویا مرتبط نیست؟
پیوند زیر را ببینید:

برای مشاهده محتوا ، لطفا وارد شوید یا ثبت نام کنید

سوال آزمون دکترای امسال بوده.
خط اول و دوم به کاربرده شده از کجا به دست آمده اند؟

مشاهده ی رایگان سوالات آزمون دکترا در

برای مشاهده محتوا ، لطفا وارد شوید یا ثبت نام کنید

ALt3rnA
08-08-2011, 02:01
- بله، همینطور است.
- در مورد این معادله، تابع [f(x) = [2x] + [3x را در نظر بگیرید. این تابع صعودی است. در نتیجه به سادگی میتوان نشان داد که مقدار تابع به ازای x<1/2 کمتر از 2 و به ازای x≥2/3 بزرگتر از 2 بوده
و همچنین در بازه (2/3 , 1/2] برابر 2 است.
- آن سوالی که میفرمایید را در صورت تمایل، بنویسید.
بله میشه همچین چیزی رو نشون داد ولی چطور میشه مجموعه جواب رو بدست آورد ؟ من تنها روشی که دیدم عدد گذاری یا رسم نمودار بوده که به نظر من روش محکمی نیست . روش جبری وجود نداره ؟
سوالی که بود [2x] + [3x] + [5x] رو مساوی 9 قرار داده بود و مجموعه جواب رو میخواست

hts1369
08-08-2011, 10:26
من دو تا سوال تو این پست پرسیدم و دنبال جواب اخر این تستها هستم . راه حل اینها رو بلدم ولی در مورد اولی هر کاری کردم به جواب نرسیدم ودر مورد دومی به جوابی که تو گزینه ها بود رسیدم ولی اطمینان ندارم اگه دوستان راه حل و جواب اخر رو بدن ممنون میشم.

برای مشاهده محتوا ، لطفا وارد شوید یا ثبت نام کنید

lebesgue
08-08-2011, 18:16
سوال آزمون دکترای امسال بوده.
خط اول و دوم به کاربرده شده از کجا به دست آمده اند؟
البته با استقرای ریاضی میتوان درستی آن را نشان داد، اما از ابتدا میتوان اینگونه شروع کرد:
از نرخ رشد دنباله میتوان پاسخ را به فرم [ برای مشاهده لینک ، لطفا با نام کاربری خود وارد شوید یا ثبت نام کنید ]^{2^n}+B^{2^n} حدس زد. با صدق این پاسخ در رابطه بازگشتی داده شده بدست می آید:


[ برای مشاهده لینک ، لطفا با نام کاربری خود وارد شوید یا ثبت نام کنید ]^{2^{n+1}}+B^{2^{n+1}}=%5Cle ft%20(A^{2^{n}}+B^{2^{n}}%20%5Cright%20)^2-2%5C%5C%5C%5C%20%5CRightarrow%20A^{2^{n+1}}+B^{2^{ n+1}}=A^{2^{n+1}}+B^{2^{n+1}}+2(AB)^{2^{n}}-2%5C%5C%5C%5C%20%5CRightarrow%20(AB)^{2^{n}}=1%5C% 5C%5C%5C%20%5CRightarrow%20B=%5Cfrac{1}{A}%5C%5C%5 C%5C%20%5CRightarrow%20a_1=A^2+%5Cleft%20(%5Cfrac{ 1}{A}%20%5Cright%20)^2=%5Csqrt{5}

و با حل این معادله بر حسب [ برای مشاهده لینک ، لطفا با نام کاربری خود وارد شوید یا ثبت نام کنید ]^2 بدست می آید:


[ برای مشاهده لینک ، لطفا با نام کاربری خود وارد شوید یا ثبت نام کنید ]{120}%20A^2=%5Cfrac{%5Csqrt5+1}{2} =%5Cvarphi%5CRightarrow%20A=%5Cvarphi^{%5Cfrac{1}{ 2}},B=%5Cvarphi^{-%5Cfrac{1}{2}}%5C%5C%5C%5C%20a_n=%5Cleft%20(%5Cvar phi^{%5Cfrac{1}{2}}%20%5Cright%20)^{2^n}+%5Cleft%2 0(%5Cvarphi^{-%5Cfrac{1}{2}}%20%5Cright%20)^{2^n}=%5Cvarphi^{2^{ n-1}}+%5Cvarphi^{-2^{n-1}}

از آنجا که رابطه [ برای مشاهده لینک ، لطفا با نام کاربری خود وارد شوید یا ثبت نام کنید ]^{-1}%20=%5Cvarphi-1 برقرار است، این پاسخ معادل همان پاسخ پست قبلی است. می توان آن را به صورت زیر نوشت:


[ برای مشاهده لینک ، لطفا با نام کاربری خود وارد شوید یا ثبت نام کنید ]{120}%20a_n=%5Cleft%20(%5Cvarphi%2 0^{2^{n-1}}+%5Cvarphi%20^{-2^{n-1}}%20%5Cright%20)%5Cfrac{%5Cvarphi%20^{2^{n-1}}-%5Cvarphi%20^{-2^{n-1}}}{%5Cvarphi%20^{2^{n-1}}-%5Cvarphi%20^{-2^{n-1}}}=%5Cfrac{%5Cvarphi%20^{2^{n}}-%5Cvarphi%20^{-2^{n}}}{%5Cvarphi%20^{2^{n-1}}-%5Cvarphi%20^{-2^{n-1}}}

و از اینجا میتوان عبارت مورد نظر را به صورت زیر ساده کرد :

[ برای مشاهده لینک ، لطفا با نام کاربری خود وارد شوید یا ثبت نام کنید ]{120}%20g_n=a_1a_2%5Ccdots%20a_n%5 Cfrac{1}{a_{n+1}}%5C%5C%5C%5C%20=%5Cleft%20(%5Cfra c{%5Cvarphi%20^{2^{1}}-%5Cvarphi%20^{-2^{1}}}{%5Cvarphi%20^{2^{0}}-%5Cvarphi%20^{-2^{0}}}%20%5Cright%20)%5Cleft%20(%5Cfrac{%5Cvarphi %20^{2^{2}}-%5Cvarphi%20^{-2^{2}}}{%5Cvarphi%20^{2^{1}}-%5Cvarphi%20^{-2^{1}}}%20%5Cright%20)%5Ccdots%20%5Cleft%20(%5Cfra c{%5Cvarphi%20^{2^{n}}-%5Cvarphi%20^{-2^{n}}}{%5Cvarphi%20^{2^{n-1}}-%5Cvarphi%20^{-2^{n-1}}}%20%5Cright%20)%5Cleft%20(%5Cfrac{%5Cvarphi%20 ^{2^{n}}-%5Cvarphi%20^{-2^{n}}}{%5Cvarphi%20^{2^{n+1}}-%5Cvarphi%20^{-2^{n+1}}}%20%5Cright%20)%5C%5C%5C%5C%20=%5Cfrac{%5 Cleft%20(%5Cvarphi%20^{2^{n}}-%5Cvarphi%20^{-2^{n}}%20%5Cright%20)^2}{%5Cleft%20(%5Cvarphi%20^{ 2^{n+1}}-%5Cvarphi%20^{-2^{n+1}}%20%5Cright%20)%5Cleft%20(%5Cvarphi%20^{2^ {0}}-%5Cvarphi%20^{-2^{0}}%20%5Cright%20)}=%5Cfrac{%5Cvarphi%20^{2^{n+ 1}}+%5Cvarphi%20^{-2^{n+1}}-2}{%5Cvarphi%20^{2^{n+1}}-%5Cvarphi%20^{-2^{n+1}}}

و اکنون به سادگی میتوان دید که :

[ برای مشاهده لینک ، لطفا با نام کاربری خود وارد شوید یا ثبت نام کنید ]{120}%20%5Clim_{n%5Crightarrow%20% 5Cinfty}g_n=1

lebesgue
08-08-2011, 18:40
بله میشه همچین چیزی رو نشون داد ولی چطور میشه مجموعه جواب رو بدست آورد ؟ من تنها روشی که دیدم عدد گذاری یا رسم نمودار بوده که به نظر من روش محکمی نیست . روش جبری وجود نداره ؟
سوالی که بود [2x] + [3x] + [5x] رو مساوی 9 قرار داده بود و مجموعه جواب رو میخواست
خب اگر چنین چیزی را نشان بدهید، نشان داده اید که مجموعه جواب، بازه (2/3 , 1/2] است، چرا که مجموعه جواب، یعنی مجموعه اعدادی برای x که به ازای آنها معادله برقرار باشد.
برای مسئله دوم، در حالت کلی تابع [ برای مشاهده لینک ، لطفا با نام کاربری خود وارد شوید یا ثبت نام کنید ](x)=%5Cleft%20%5Clfloor%20a_ 1x%20%5Cright%20%5Crfloor+%5Cleft%20%5Clfloor%20a_ 2x%20%5Cright%20%5Crfloor%5Ccdots%20%5Cleft%20%5Cl floor%20a_nx%20%5Cright%20%5Crfloor که در آن [ برای مشاهده لینک ، لطفا با نام کاربری خود وارد شوید یا ثبت نام کنید ] ها اعداد صحیح مثبت هستند را در نظر بگیرید.
مقدار این تابع، تنها در مضارب صحیح [ برای مشاهده لینک ، لطفا با نام کاربری خود وارد شوید یا ثبت نام کنید ] ها تغییر می کند و در میان آنها ثابت است. در نتیجه میتوان با عدد گذاری در نقاط مناسب، به سرعت بازه جواب را پیدا و سپس با توجه به صعودی بودن تابع، اثبات نمود.

ALt3rnA
08-08-2011, 19:09
خب اگر چنین چیزی را نشان بدهید، نشان داده اید که مجموعه جواب، بازه (2/3 , 1/2] است، چرا که مجموعه جواب، یعنی مجموعه اعدادی برای x که به ازای آنها معادله برقرار باشد. بله این که بدیهیه ! سوال من اینه که بازه رو از کجا باید پیدا کرد ؟ کدوم معادله رو باید حل کرد تا به این بازه رسید ؟ منظورم به جز عدد گذاری یا رسم نموداره !

برای مسئله دوم، در حالت کلی تابع [ برای مشاهده لینک ، لطفا با نام کاربری خود وارد شوید یا ثبت نام کنید ] 20a_1x%20%5Cright%20%5Crfloor+%5Cleft%20%5Clfloor% 20a_2x%20%5Cright%20%5Crfloor%5Ccdots%20%5Cleft%20 %5Clfloor%20a_nx%20%5Cright%20%5Crfloor که در آن [ برای مشاهده لینک ، لطفا با نام کاربری خود وارد شوید یا ثبت نام کنید ] ها اعداد صحیح مثبت هستند را در نظر بگیرید.
مقدار این تابع، تنها در مضارب صحیح [ برای مشاهده لینک ، لطفا با نام کاربری خود وارد شوید یا ثبت نام کنید ] ها تغییر می کند و در میان آنها ثابت است. در نتیجه میتوان با عدد گذاری در نقاط مناسب، به سرعت بازه جواب را پیدا و سپس با توجه به صعودی بودن تابع، اثبات نمود. بله خوب اینم مسلما با عدد گذاری حل میشه ولی عدد گذاری روش جالبی نیست .
مثلا توی این سوال مجموعه جواب تهی هست .
میخوام ببینم هیچ گونه راه حل جبری برای حل این طور سوالات نیست ؟

یه سوال دیگه هم داشتم . معیار خاصی برای تغییر یک تابع بصورت کلی وجود داره ؟ مثلا آیا با روشی غیر از نمودار یا استقرا میشه تشخیص داد که 2^n و دو به توان n در چه بازه هایی از هم بیشتر میشن ؟ یا لاقل تعداد جواب ها ؟

lebesgue
08-08-2011, 20:07
بله این که بدیهیه ! سوال من اینه که بازه رو از کجا باید پیدا کرد ؟ کدوم معادله رو باید حل کرد تا به این بازه رسید ؟ منظورم به جز عدد گذاری یا رسم نموداره !
بله خوب اینم مسلما با عدد گذاری حل میشه ولی عدد گذاری روش جالبی نیست .
مثلا توی این سوال مجموعه جواب تهی هست .
میخوام ببینم هیچ گونه راه حل جبری برای حل این طور سوالات نیست ؟

معادله [ برای مشاهده لینک ، لطفا با نام کاربری خود وارد شوید یا ثبت نام کنید ](x)=%5Cleft%20%5Clfloor%20a_ 1x%20%5Cright%20%5Crfloor+%5Cleft%20%5Clfloor%20a_ 2x%20%5Cright%20%5Crfloor+%5Ccdots+%20%5Cleft%20%5 Clfloor%20a_nx%20%5Cright%20%5Crfloor=m که در آن m و [ برای مشاهده لینک ، لطفا با نام کاربری خود وارد شوید یا ثبت نام کنید ] ها اعداد صحیح مثبت هستند را در نظر بگیرید. فرض کنید [ برای مشاهده لینک ، لطفا با نام کاربری خود وارد شوید یا ثبت نام کنید ] دنباله مضارب صحیح مثبت [ برای مشاهده لینک ، لطفا با نام کاربری خود وارد شوید یا ثبت نام کنید ] ها باشد که به صورت صعودی مرتب شده است. می توان نشان داد که پاسخ معادله برابر است با بازه [ برای مشاهده لینک ، لطفا با نام کاربری خود وارد شوید یا ثبت نام کنید ][b_m,b_{m+1}).
اگر [ برای مشاهده لینک ، لطفا با نام کاربری خود وارد شوید یا ثبت نام کنید ]{m+1} باشد، به این معناست که مجموعه جواب تهی است. اینکه آیا میتوان [ برای مشاهده لینک ، لطفا با نام کاربری خود وارد شوید یا ثبت نام کنید ] و [ برای مشاهده لینک ، لطفا با نام کاربری خود وارد شوید یا ثبت نام کنید ]{m+1} را برحسب m و [ برای مشاهده لینک ، لطفا با نام کاربری خود وارد شوید یا ثبت نام کنید ] ها نوشت یا خیر را نمیدانم، اما گمان می کنم به سادگی شدنی نیست.
در ضمن، بسیاری از مسائل در ریاضیات، راه حل تحلیلی ندارند. بسیاری از معادلات نیز (مانند x = cosx) تنها به روش عددی حل می شوند.

ali1234
09-08-2011, 04:25
من لقمه رو برات جویدم گذاشتم دهنت
[ برای مشاهده لینک ، لطفا با نام کاربری خود وارد شوید یا ثبت نام کنید ]

سلام
دستت درد نكنه جويدي گذاشتي دهنم :2:
اما مي خوام برام بيشتر بجويش !:31:

اين يكي از تمرينهاي كتاب رياضي سوم دبيرستان اونهم رشته تجربيه

هنوز مشتق نخونده كه بتونه از هوپيتال استفاده كنه و از هم ارزي و ..... به همان دليلي كه گفتم نميشه استفاده كرد

فقط از محاسبات جبري ساده مثل فاكتور گيري و تجزيه و .... ميشه استفاده كرد

ALt3rnA
09-08-2011, 12:22
سلام
دستت درد نكنه جويدي گذاشتي دهنم :2:
اما مي خوام برام بيشتر بجويش !:31:

اين يكي از تمرينهاي كتاب رياضي سوم دبيرستان اونهم رشته تجربيه

هنوز مشتق نخونده كه بتونه از هوپيتال استفاده كنه و از هم ارزي و ..... به همان دليلي كه گفتم نميشه استفاده كرد

فقط از محاسبات جبري ساده مثل فاكتور گيري و تجزيه و .... ميشه استفاده كرد

[ برای مشاهده لینک ، لطفا با نام کاربری خود وارد شوید یا ثبت نام کنید ] g
از اتحاد های مثلثاتی که میتونن استفاده کنن ؟ رابطه ی طلایی

lebesgue
09-08-2011, 13:28
البته باز هم در انتها از هم ارزی sinx~x استفاده شده است.
من با یک تغییر در پاسخ بالا، راه حلی را در زیر می آورم که در چارچوب کتاب ریاضی سوم دبیرستان تجربی قابل پذیرش باشد!


[ برای مشاهده لینک ، لطفا با نام کاربری خود وارد شوید یا ثبت نام کنید ]{120}%20L=%5Clim_{x%5Crightarrow%2 00}%5Cfrac{%5Ctan%20x-%5Csin%20x}{x^3}=%5Clim_{x%5Crightarrow%200}%5Cfra c{%5Csin%20x(1-%5Ccos%20x)}{x^3%5Ccos%20x}%5C%5C%5C%5C%5C%5C=%5Cl im_{x%5Crightarrow%200}%5Cfrac{2%5Csin%20%5Cfrac{x }{2}%5Ccos%20%5Cfrac{x}{2}(2%5Csin^2%20%5Cfrac{x}{ 2})}{x^3%5Ccos%20x}=%5Clim_{x%5Crightarrow%200}%5C frac{%5Csin^3%20%5Cfrac{x}{2}%5Ccos%20%5Cfrac{x}{2 }}{2%5Cleft%20(%5Cfrac{x}{2}%20%5Cright%20)^3%5Cco s%20x}

با استفاده از قضیه مربوط به حد ضرب دو تابع، می توان نوشت:


[ برای مشاهده لینک ، لطفا با نام کاربری خود وارد شوید یا ثبت نام کنید ]{120}%20L=%5Clim_{x%5Crightarrow%2 00}%5Cleft%20(%5Cfrac{%5Csin%20%5Cfrac{x}{2}}{%5Cf rac{x}{2}}%20%5Cright%20)^3%5Clim_{x%5Crightarrow% 200}%5Cfrac{%5Ccos%20%5Cfrac{x}{2}}{2%5Ccos%20x}=% 5Cfrac{1}{2}%5Clim_{x%5Crightarrow%200}%5Cleft%20( %5Cfrac{%5Csin%20%5Cfrac{x}{2}}{%5Cfrac{x}{2}}%20% 5Cright%20)^3

از قضیه مربوط به توان توابع (که نتیجه ساده از همان قضیه ضرب است) ، نتیجه می شود:


[ برای مشاهده لینک ، لطفا با نام کاربری خود وارد شوید یا ثبت نام کنید ]{120}%20L=%5Cfrac{1}{2}%5Cleft%20( %5Clim_{x%5Crightarrow%200}%5Cfrac{%5Csin%20%5Cfra c{x}{2}}{%5Cfrac{x}{2}}%20%5Cright%20)^3=%5Cfrac{1 }{2}

Samba
09-08-2011, 14:00
سلام

اين سوالو تو آموزش عالي گذاشته بودم و اونجا هم بهش جواب دادم:31: گفتم اينجا هم بذارمش تا ببينيم راه حل ديگه اي هم داره؟

دنباله زير به چه عددي همگراست؟

[ برای مشاهده لینک ، لطفا با نام کاربری خود وارد شوید یا ثبت نام کنید ][n]%7Bn%21%7D&space;%7D%7Bn%7D


برای مشاهده محتوا ، لطفا وارد شوید یا ثبت نام کنید

dkhatibi
09-08-2011, 20:58
اینم یکی دیگه

[ برای مشاهده لینک ، لطفا با نام کاربری خود وارد شوید یا ثبت نام کنید ]{a_{n-1}+a_{n-2}}{2}\\\Rightarrow%20\lim_{n%20\to%20\infty%20}a_ n=?

Reza_994
10-08-2011, 14:50
سلام دوستان یه سری سوال چراییی !!!

1. چرا حاصل ضرب خارجی دو بردار بر بردار عمود میشه ؟ ( دلیلش ؟ )
2. چرا از ماتریس استفاده میشه ؟؟ ( ضرب خارجی )
3. شکل حاصل ضرب داخلی ؟ ( اگه بگین از cos چرا استفاده میشه چه بهتر ! )


متاسفانه معلمای ما خیلی حفظی درس میدن !

ALt3rnA
10-08-2011, 17:13
1. چرا حاصل ضرب خارجی دو بردار بر بردار عمود میشه ؟ ( دلیلش ؟ )
2. چرا از ماتریس استفاده میشه ؟؟ ( ضرب خارجی )
3. شکل حاصل ضرب داخلی ؟ ( اگه بگین از cos چرا استفاده میشه چه بهتر ! )1-یعنی چی چرا ؟ اگه منظورتون اثباتش هست که خیلی بدیهیه ولی با ضرب داخلی میتونید اثباتش کنید
2-فقط ماتریس نیست . روش های دیگه ای هم هست . برای اثباتش میتونید عبارت Vector Cross Product Proof رو سرچ کنید تو نت
3-حاصلضرب داخلی عدده و شکل نداره . اما اگه بخواین به یه درک شهودی برسید میشه اندازه به بردار * اندازه تصویر بردار دیگه (اندازه جبری)
برای اینکه چرا از کسینوس استفاده میشه دو تا بردار رو بکش . اندازه تصویر بردار دومی رو چطوری بدست میاری بر حسب خودش ؟ با کسینوس

ALt3rnA
10-08-2011, 18:49
اين سوالو تو آموزش عالي گذاشته بودم و اونجا هم بهش جواب دادم[ برای مشاهده لینک ، لطفا با نام کاربری خود وارد شوید یا ثبت نام کنید ] گفتم اينجا هم بذارمش تا ببينيم راه حل ديگه اي هم داره؟

دنباله زير به چه عددي همگراست؟

[ برای مشاهده لینک ، لطفا با نام کاربری خود وارد شوید یا ثبت نام کنید ][n]%7Bn%21%7D&space;%7D%7Bn%7D
باور کن 3 باره میخوام جواب این سوالتو بدم هر بار یه چیزی میشه
آخرین بارش دیشب بود یهو سایت پوکید ! :دی
راه حلتو دیدم . درسته مسلما !
ولی یه راه حل خیلی ساده تر هم هست . یه هم ارزی داریم به نام هم ارزی استرلینگ .
ازون میتونی به این نتیجه برسی که n فاکتوریل فرجه n در بی نهایت با n رو e هم ارز هست !
پس برای این سوال فقط یه تقسیم بر n میمونه که جواب آخرش میشه 1 روی e
به همین راحتی به همین خوش مزگی

ALt3rnA
10-08-2011, 19:55
سلام !
آقا یک نفر لطف میکنه بسط سری 1 + 2 + 3 + 4 + ... + n
رو بزاره برای توان های بیشتر از 1 ؟
توی دنباله نیاز پیدا کردم بهش
ممنون

dkhatibi
10-08-2011, 22:18
اینم یکی دیگه

[ برای مشاهده لینک ، لطفا با نام کاربری خود وارد شوید یا ثبت نام کنید ]{a_{n-1}+a_{n-2}}{2}\\\Rightarrow%20\lim_{n%20\to%20\infty%20}a_ n=?


[ برای مشاهده لینک ، لطفا با نام کاربری خود وارد شوید یا ثبت نام کنید ]{1}{3}a+\frac{2}{3}b???!

lebesgue
10-08-2011, 22:35
سلام !
آقا یک نفر لطف میکنه بسط سری 1 + 2 + 3 + 4 + ... + n
رو بزاره برای توان های بیشتر از 1 ؟
توی دنباله نیاز پیدا کردم بهش
ممنون
پیوند زیر را ببینید:

برای مشاهده محتوا ، لطفا وارد شوید یا ثبت نام کنید

[ برای مشاهده لینک ، لطفا با نام کاربری خود وارد شوید یا ثبت نام کنید ]{1}{3}a+\frac{2}{3}b???!


من نیز به همین پاسخ رسیدم.

ALt3rnA
10-08-2011, 23:25
پیوند زیر را ببینید:
خیلی ممنون
دقیقا همون چیزی بود که میخواستم
ولی خیلی از چیزی که فکر میکردم پیچیده تره ! حساب دستیش واقعا سخته به خصوص برای اون اعداد برنولیش .
فعلا مثل اینکه مجبورم هم ارزیش تو بینهایت رو حفظ کنم که آسون تره . ولی بعضی سوالا جواب نمیده .
ولی ظاهرا به این سری بعدا تو انتگرال هم نیاز خواهم داشت ! اونموقع چکار کنم ؟

lebesgue
11-08-2011, 12:05
خیلی ممنون
دقیقا همون چیزی بود که میخواستم
ولی خیلی از چیزی که فکر میکردم پیچیده تره ! حساب دستیش واقعا سخته به خصوص برای اون اعداد برنولیش .
فعلا مثل اینکه مجبورم هم ارزیش تو بینهایت رو حفظ کنم که آسون تره . ولی بعضی سوالا جواب نمیده .

در بینهایت، این دو معادل یکدیگرند:


[ برای مشاهده لینک ، لطفا با نام کاربری خود وارد شوید یا ثبت نام کنید ]{120}%20%5Csum_{k=1}^{n}k^p%5Csim% 20%5Cfrac{n^{p+1}}{p+1}

یا به عبارت دقیقتر:


[ برای مشاهده لینک ، لطفا با نام کاربری خود وارد شوید یا ثبت نام کنید ]{120}%20%5Clim_{n%5Crightarrow%20% 5Cinfty}%5Cfrac{%5Csum_{k=1}^{n}k^p}{n^{p+1}/(p+1)}=1



ولی ظاهرا به این سری بعدا تو انتگرال هم نیاز خواهم داشت ! اونموقع چکار کنم ؟
نه، قضایای اساسی حسابان برای همین هستند دیگر!

ALt3rnA
11-08-2011, 12:55
در بینهایت، این دو معادل یکدیگرند:


[ برای مشاهده لینک ، لطفا با نام کاربری خود وارد شوید یا ثبت نام کنید ] n%7Dk%5Ep%5Csim%20%5Cfrac%7Bn%5E%7Bp+1%7D%7D%7Bp+1 %7D

این هم ارزی همیشه کاربرد نداره
سوالی که من دارم این سری رو با توان 4 داده و بعد 3 جمله اول هم ارزی رو ازش کم کرده و گفته حد رو بدست بیارید

Headphone
14-08-2011, 20:16
سلام

یه سوال :

به ازای چند مقدار طبیعی برای k ، رابطه ی غیر تهی [ برای مشاهده لینک ، لطفا با نام کاربری خود وارد شوید یا ثبت نام کنید ] ، تشکیل تابعی

بر حسب x نمی دهد ؟

ممنون

ALt3rnA
14-08-2011, 22:25
سلام

یه سوال :

به ازای چند مقدار طبیعی برای k ، رابطه ی غیر تهی [ برای مشاهده لینک ، لطفا با نام کاربری خود وارد شوید یا ثبت نام کنید ] ، تشکیل تابعی

بر حسب x نمی دهد ؟

ممنون
سلام
ببین اول مربع کامل تشکیل بده
میشه :

[ برای مشاهده لینک ، لطفا با نام کاربری خود وارد شوید یا ثبت نام کنید ]
این معادله یه دایره است !
وقتی جواب داره که سمت راستش مثبت باشه درون صورت تو صفحه مختصات تابعت دقیقا یه دایره میشه که مسلما تابع نیست !
اگه سمت راست منفی بشه تابع تهی میشه چون هیچ دوتا عددی نیستن که مثبت باشن و جمعشون منفی شه
میمونه اینکه سمت راست صفر باشه ! پس k = 5
در اینصورت تابع یه نقطه میشه فقط

نمودار به ازائ k=0

[ برای مشاهده لینک ، لطفا با نام کاربری خود وارد شوید یا ثبت نام کنید ] eType=image/gif&s=53&w=200&h=198&cdf=Coordinates&cdf=Tooltips

lebesgue
14-08-2011, 22:39
راهنمایی: معادله را به صورت [ برای مشاهده لینک ، لطفا با نام کاربری خود وارد شوید یا ثبت نام کنید ](x+2)^2+(y-1)^2=5-k بازنویسی کنید.

---------- Post added at 11:34 PM ---------- Previous post was at 11:33 PM ----------

پست بالا را ندیدم. با تشکر از ALt3rnA.
پاسخ نهایی مسئله، مجموعه {1,2,3,4} برای k است، یعنی 4 مقدار.

Headphone
14-08-2011, 22:53
سلام
ببین اول مربع کامل تشکیل بده
میشه :

[ برای مشاهده لینک ، لطفا با نام کاربری خود وارد شوید یا ثبت نام کنید ]
این معادله یه دایره است !
وقتی جواب داره که سمت راستش مثبت باشه درون صورت تو صفحه مختصات تابعت دقیقا یه دایره میشه که مسلما تابع نیست !
اگه سمت راست منفی بشه تابع تهی میشه چون هیچ دوتا عددی نیستن که مثبت باشن و جمعشون منفی شه
میمونه اینکه سمت راست صفر باشه ! پس k = 5
در اینصورت تابع یه نقطه میشه فقط
پس اگه 5 نباشه تابع غیر تهی نیست
نمودار به ازائ k=0

[ برای مشاهده لینک ، لطفا با نام کاربری خود وارد شوید یا ثبت نام کنید ] eType=image/gif&s=53&w=200&h=198&cdf=Coordinates&cdf=Tooltips


راهنمایی: معادله را به صورت [ برای مشاهده لینک ، لطفا با نام کاربری خود وارد شوید یا ثبت نام کنید ] بازنویسی کنید.

---------- Post added at 11:34 PM ---------- Previous post was at 11:33 PM ----------

[COLOR\]
پست بالا را ندیدم. با تشکر از پاسخ نهایی مسئله، مجموعه {1,2,3,4} برای k است، یعنی 4 مقدار.[COLOR=Silver]

---------- Post added at 11:39 PM ---------- Previous post was at 11:36 PM ----------

gfhfgjghkjghj

ممنون ، منم خودم از این راه حل رفتم و جواب رو بدست آوردم ، اما یه طور دیگه رفتم ولی به جواب نرسیدم : به این ترتیب :

[ برای مشاهده لینک ، لطفا با نام کاربری خود وارد شوید یا ثبت نام کنید ] اگه اینو یه معادله درجه 2 در نظر بگیریم با [ برای مشاهده لینک ، لطفا با نام کاربری خود وارد شوید یا ثبت نام کنید ] ، در نتیجه : [ برای مشاهده لینک ، لطفا با نام کاربری خود وارد شوید یا ثبت نام کنید ] or%7BRed%7D%20%7B4-4%284x+x%5E2+k%29%7D%7D%7D%7B2%7D

حالا با خودم گفتم اگه بخواد تابع نباشه باید عبارت قرمز رنگ برابر صفر نشه، در نتیجه [ برای مشاهده لینک ، لطفا با نام کاربری خود وارد شوید یا ثبت نام کنید ] پس باید دلتا منفی باشه یعنی : k>5 !!!:31:

خودم میدونم یه جا یه اشتباه مهلک ( و مسخره ای !!:31:) کردم ، حالا کجاس؟

ALt3rnA
14-08-2011, 23:11
ممنون ، منم خودم از این راه حل رفتم و جواب رو بدست آوردم ، اما یه طور دیگه رفتم ولی به جواب نرسیدم : به این ترتیب :

[ برای مشاهده لینک ، لطفا با نام کاربری خود وارد شوید یا ثبت نام کنید ] اگه اینو یه معادله درجه 2 در نظر بگیریم با [ برای مشاهده لینک ، لطفا با نام کاربری خود وارد شوید یا ثبت نام کنید ] ، در نتیجه : [ برای مشاهده لینک ، لطفا با نام کاربری خود وارد شوید یا ثبت نام کنید ] or%7BRed%7D%20%7B4-4%284x+x%5E2+k%29%7D%7D%7D%7B2%7D

حالا با خودم گفتم اگه بخواد تابع نباشه باید عبارت قرمز رنگ برابر صفر نشه، در نتیجه [ برای مشاهده لینک ، لطفا با نام کاربری خود وارد شوید یا ثبت نام کنید ] پس باید دلتا منفی باشه یعنی : k>5 !!![ برای مشاهده لینک ، لطفا با نام کاربری خود وارد شوید یا ثبت نام کنید ]

خودم میدونم یه جا یه اشتباه مهلک ( و مسخره ای !![ برای مشاهده لینک ، لطفا با نام کاربری خود وارد شوید یا ثبت نام کنید ]) کردم ، حالا کجاس؟ اشتباه اینجاست !
ببین معادله ای که تشکیل دادی درست !
اما جوابی که شما به دست آوردی تمامش تابع های تهی اند ! در حالی که تو فرض گفته تهی نباشه !
شما فرض کردی برای اینکه تابع نباشه نباید y بدست بیاد ! در حالی که اینطور نیست و به ازا x هایی که دوتا y هم بدن تابع نیست و اونا رو هم باید حساب کرد !
کلا اگه y بدست نیاد و جواب نداشته رابطه تهی هست . رابطه ی تهی همیشه تابعه !
مثلا توی همین معادله ی خودت k رو برابر 1 بده ! میبینی به ازا y دوتا جواب داره یعنی همون دایره که گفتم !
پس چیکار باید کنی تا این راه هم جواب بده ؟ یه معادله بر حسب x تشکیل بده اینبار ! :دی
امیدوارم متوجه شده باشی

Headphone
14-08-2011, 23:15
پس چیکار باید کنی تا این راه هم جواب بده ؟ یه معادله بر حسب x تشکیل بده اینبار ! :دی

یعنی چی ؟ میشه دقیق تر بگی

ALt3rnA
14-08-2011, 23:39
یعنی چی ؟ میشه دقیق تر بگی
آخ ! آقا شرمنده یه لحظه حواسم پرت شد فکر کردم میخوایم رابطه بودنشو بررسی کنیم !
ببین این معادله ای که شما نوشتی و همه حرفای من به جز خط آخر :دی درسته !
فقط به این دقت کن ! زیر رادیکال رو اگه ساده کنی به این میرسی اگه 5 منهای k مثبت باشه به ازا مقادیری از x دوتا y بدست میاد ! که میشه همون دایره
اگه منفی بشه هیچ راهی نیست جواب داشته باشه ! و اگه صفر باشه فقط یه نقطه است !
شما اشکال کارت اینه که فقط زیر رادیکال رو منفی حساب کردی !
اگه رادیکال زیرش مثبت باشه یعنی به ازا یه دونه X دو تا y بدست آوردی ! این تابعه ؟ نیست دیگه !
مثلا به ایکس بده -2 !
میبینی که به ازا کا های کوچیکتر از 5 زیر رادیکال مثبت میشه ! یعنی با یدونه ایکس دو تا جواب برای y بدست اومده
آقا بازم شرمنده من فردا امتحان شیمی دارم 3 روزه دارم مثل چارپا میخونم صبح تا شب اصلا حواسم نیست !
همینو هم یه تحلیل بکن شاید همون اولیه درست بود :دی

hossein_porru
15-08-2011, 05:54
سلام دوستان . دو تا سوال داشتم راجع تابع کسی بتونه حل کنه ممنون میشم کلی مخمو کار گرفتم حل کنم هنگ کردم. اخرشم حل نشد.

در هر یک از توابع زیر اف ایکس را محاسبه کنید.
به جای علامت توان ^ گذاشتم و به جای خط کسری/ گذاشتم.

f(x/x-1)=2x^2-3x+1
f(x-2/x)=1/x

afshin b
15-08-2011, 10:50
سلام دوستان . دو تا سوال داشتم راجع تابع کسی بتونه حل کنه ممنون میشم کلی مخمو کار گرفتم حل کنم هنگ کردم. اخرشم حل نشد.

در هر یک از توابع زیر اف ایکس را محاسبه کنید.
به جای علامت توان ^ گذاشتم و به جای خط کسری/ گذاشتم.

f(x/x-1)=2x^2-3x+1
f(x-2/x)=1/x

برای اولی:

[ برای مشاهده لینک ، لطفا با نام کاربری خود وارد شوید یا ثبت نام کنید ]

اگه توی معادله بجای ایکس مقدار جدیدی که برحسب u بدست آوردیم رو قرار بدیم و سادش کنیم به این میرسیم:

[ برای مشاهده لینک ، لطفا با نام کاربری خود وارد شوید یا ثبت نام کنید ]

معادله د.م هم همینطور حل میشه و جواب:

[ برای مشاهده لینک ، لطفا با نام کاربری خود وارد شوید یا ثبت نام کنید ]

hossein_porru
15-08-2011, 16:40
داداش ممنون از کمکت چون عضو جدیدم دکمه تشکر ندارم نوشتاری تشکر میکنم!!!!!!
من نرم افزار مث تایپ دانلود کردم ولی نمیدونم چطوری فرمولی رو که توش نوشتم توی سایت قرار بدم . ممنون میشم راهنمایی کنی.

hossein_porru
16-08-2011, 00:13
داداش دکمه تشکرو پیدا کردم و همین طور نحوه تایپ فرمولهای ریاضی.
این سوال رو کی میتونه حل کنه؟؟؟؟
فاصله هر کانون از خط مجانب هذلولی به معادله [ برای مشاهده لینک ، لطفا با نام کاربری خود وارد شوید یا ثبت نام کنید ]^2-3y^2=12 کدام است.
1)2 2)3 3)[ برای مشاهده لینک ، لطفا با نام کاربری خود وارد شوید یا ثبت نام کنید ]{3} 4)4

رقم سمت راست [ برای مشاهده لینک ، لطفا با نام کاربری خود وارد شوید یا ثبت نام کنید ]^{1381} با رقم راست کدام عدد برابر است؟
1)[ برای مشاهده لینک ، لطفا با نام کاربری خود وارد شوید یا ثبت نام کنید ]^{1999} 2)[ برای مشاهده لینک ، لطفا با نام کاربری خود وارد شوید یا ثبت نام کنید ]^{2002} 3)[ برای مشاهده لینک ، لطفا با نام کاربری خود وارد شوید یا ثبت نام کنید ]^{2003} 4)[ برای مشاهده لینک ، لطفا با نام کاربری خود وارد شوید یا ثبت نام کنید ]^{2005}

afshin b
16-08-2011, 00:58
داداش ممنون از کمکت چون عضو جدیدم دکمه تشکر ندارم نوشتاری تشکر میکنم!!!!!!
من نرم افزار مث تایپ دانلود کردم ولی نمیدونم چطوری فرمولی رو که توش نوشتم توی سایت قرار بدم . ممنون میشم راهنمایی کنی.
خواهش میکنم. ولی مثل اینکه تونستین تشکر کنین!

باید ازش عکس بگیرین و توی یه سایت مثل:

برای مشاهده محتوا ، لطفا وارد شوید یا ثبت نام کنیدآپلود کنین و بعد لینک عکس رو با دکمه [ برای مشاهده لینک ، لطفا با نام کاربری خود وارد شوید یا ثبت نام کنید ] توی پستتون وارد کنین.
اما ما اینجا از این سایت برای نوشتن ریاضی استفاده میکنم:

برای مشاهده محتوا ، لطفا وارد شوید یا ثبت نام کنید

PARHAM.STR
17-08-2011, 20:27
من این مساله رو با استفاده از محاسبه طول نیمساز بر حسب اضلاع حل کردم،یکم طولانی میشه ولی راحته!و راحت به ذهن میرسه و عجیب نیست!راه حلی که اینجا میگم نمیدونم مال کیه!ولی بی نهایت قشنگه....
از دو قضیه زیر در حل این مساله استفاده میشه:
قضیه یک:در مثلث مفروض abc داریم b>c اگر و تنها اگر ac>ab.(یعنی زاویه b بزرگتر از زاویه c است اگر و تنها اگر ضلع ac بزرگتر از ضلع ab باشد)
قضیه دو:در دو مثلث abc و xyz داریم و ab=xy و ac=xz .آنگاه a>x اگر و تنها اگر bc>yz .(این قضیه یک خاصیت شهودا بدیهی قیچی را بیان می کند!یعنی هرچه قیچی را بیشتر باز کنیم طول دهانه اش بیشتر می شود!)
مثلث abc را در نظر بگیرید که دو نیمساز bd و ce با هم برابرند.می خواهیم ثابت کنیم ab=ac
اثبات:
برهان خلف:فرض کنید ab با ac برابر نباشد،پس بدون کم شدن از کلیت مساله می توان فرض کرد ac>ab.
پس طبق قضیه یک داریم b>c. بنابر این b/2>c/2 پس در دو مثلث bdc و ceb طبق قضیه دو داریم cd>be.
حال از نقطه e پاره خط ef را موازی و برابر bd رسم کنید.پس چهارضلعی befd متوازی الاضلاع است.و مثلث efc متساوی الساقین است.حال دقت کنید که زاویه efd برابر b/2 است و زاویه ecd برابر c/2 است،پس با توجه به اینکه دو زاویه efc وecf برابرند، بدست می اید که زاویه dfc از زاویه dcf کوچکتر است.پس طبق قضیه یک داریم cd<df . اما be=df و قبلان داشتیم cd>be که تناقض است.
البته میشد از طریق موازی مورب و بدون برهان خلف حلش کرد.

Smartie7
22-08-2011, 10:46
سلام ...

دوستان لطف کنید این نامعادله رو حل کنید.


[ برای مشاهده لینک ، لطفا با نام کاربری خود وارد شوید یا ثبت نام کنید ]

lebesgue
22-08-2011, 13:04
با قرار دادن [ برای مشاهده لینک ، لطفا با نام کاربری خود وارد شوید یا ثبت نام کنید ]^k نامعادله به صورت زیر در می آید:


[ برای مشاهده لینک ، لطفا با نام کاربری خود وارد شوید یا ثبت نام کنید ]{120}%20%5Cfrac{(1-0.88)-x}{1-0.88x}%3C0.0005

با حل نامعادله، پاسخی به صورت [ برای مشاهده لینک ، لطفا با نام کاربری خود وارد شوید یا ثبت نام کنید ] بدست می آید که از اینجا میتوانید نتیجه بگیرید:


[ برای مشاهده لینک ، لطفا با نام کاربری خود وارد شوید یا ثبت نام کنید ]{120}%20%5Csmall%20%5Cfrac{%5Cln%2 0b}{%5Cln%200.88}%3Ck%3C%5Cfrac{%5Cln%20a}{%5Cln%2 00.88}

dkhatibi
24-08-2011, 13:05
فرض کنید [ برای مشاهده لینک ، لطفا با نام کاربری خود وارد شوید یا ثبت نام کنید ] یک تابع صعودی باشد و [ برای مشاهده لینک ، لطفا با نام کاربری خود وارد شوید یا ثبت نام کنید ]{x\rightarrow \infty }\frac{f(2x)}{f(x)}=1~ ان گاه حاصل حد زیر چیست؟


[ برای مشاهده لینک ، لطفا با نام کاربری خود وارد شوید یا ثبت نام کنید ]{x\rightarrow \infty }\frac{f(cx)}{f(x)}=?~

lebesgue
24-08-2011, 18:37
احیاناً منظور شما این نبوده؟


فرض کنید [ برای مشاهده لینک ، لطفا با نام کاربری خود وارد شوید یا ثبت نام کنید ] یک تابع صعودی باشد و [ برای مشاهده لینک ، لطفا با نام کاربری خود وارد شوید یا ثبت نام کنید ]{x\rightarrow \infty }\frac{f(2x)}{f(x)}=1~ ان گاه حاصل حد زیر چیست؟


[ برای مشاهده لینک ، لطفا با نام کاربری خود وارد شوید یا ثبت نام کنید ]{x\rightarrow \infty }\frac{f(cx)}{f(x)}=?~

amh1365
24-08-2011, 19:11
[QUOTE=1233445566;6398003]احیاناً منظور شما این نبوده؟[/QUOTE

من فکر کنم جواب 1 باشه. چون اگه c=2 باشد همون فرض مساله را داریم.

dkhatibi
24-08-2011, 23:17
[QUOTE=1233445566;6398003]احیاناً منظور شما این نبوده؟[/QUOTE

من فکر کنم جواب 1 باشه. چون اگه c=2 باشد همون فرض مساله را داریم.

احیانا منظور من؟
شاید یک باشد. می شه تابعی مثال زد که بنظر 1 درسته. اثبات ؟
ضمنا می تونه c/2 باشه یا ...

lebesgue
25-08-2011, 12:15
احیانا منظور من؟

صورت مسئله به آن فرمی که شما نوشتید، نادرست است و معلوم نیست که تابع f برای چه معرفی شده است!
من فرض می کنم که مسئله مورد نظر شما به صورت زیر بوده است:

فرض کنید [ برای مشاهده لینک ، لطفا با نام کاربری خود وارد شوید یا ثبت نام کنید ] یک تابع صعودی باشد و [ برای مشاهده لینک ، لطفا با نام کاربری خود وارد شوید یا ثبت نام کنید ]{x\rightarrow \infty }\frac{f(2x)}{f(x)}=1~ ان گاه حاصل حد زیر چیست؟


[ برای مشاهده لینک ، لطفا با نام کاربری خود وارد شوید یا ثبت نام کنید ]{x\rightarrow \infty }\frac{f(cx)}{f(x)}=?~

با استقراء ریاضی نشان می دهیم برای هر n صحیح مثبت، داریم:

[ برای مشاهده لینک ، لطفا با نام کاربری خود وارد شوید یا ثبت نام کنید ]{120}%20%5Clim_{x%5Crightarrow%20% 5Cinfty%20}%5Cfrac{f(2^nx)}{f(x)}=1

برای n=1 که بنا به فرض مسئله برقرار است، فرض کنیم برای n=k برقرار باشد، نشان می دهیم برای n=k+1 هم برقرار خواهد بود:


[ برای مشاهده لینک ، لطفا با نام کاربری خود وارد شوید یا ثبت نام کنید ]{120}%20%5Cdpi{120}%20%5C%5CL=%5Cl im_{x%5Crightarrow%20%5Cinfty%20}%5Cfrac{f(2^{k+1} x)}{f(x)}=%5Clim_{x%5Crightarrow%20%5Cinfty%20}%5C frac{f(2^{k+1}x)}{f(x)}%5Cfrac{f(2^{k}x)}{f(2^{k}x )}%5C%5C%5C%5C=%5Cleft%20(%20%5Clim_{x%5Crightarro w%20%5Cinfty%20}%5Cfrac{f(2^{k+1}x)}{f(2^kx)}%20%5 Cright%20)%5Cleft%20(%20%5Clim_{x%5Crightarrow%20% 5Cinfty%20}%5Cfrac{f(2^{k}x)}{f(x)}%20%5Cright%20) %5C%5C%5C%5C

حد سمت راست که بنا به فرض برابر با 1 است، اما برای حد سمت چپ، قرار می دهیم [ برای مشاهده لینک ، لطفا با نام کاربری خود وارد شوید یا ثبت نام کنید ]^kx:


[ برای مشاهده لینک ، لطفا با نام کاربری خود وارد شوید یا ثبت نام کنید ]{120}%20L=%5Clim_{u%5Crightarrow%2 0%5Cinfty%20}%5Cfrac{f(2u)}{f(u)}=1

همچنین به سادگی میتوان نشان داد که برای هر m صحیح مثبت، داریم:


[ برای مشاهده لینک ، لطفا با نام کاربری خود وارد شوید یا ثبت نام کنید ]{120}%20%5Clim_{x%5Crightarrow%20% 5Cinfty%20}%5Cfrac{f(2^{-m}x)}{f(x)}=1

برای c>0، اعداد صحیح m و n وجود دارند که داشته باشیم [ برای مشاهده لینک ، لطفا با نام کاربری خود وارد شوید یا ثبت نام کنید ]^{-m}%5Cleq%20c%5Cleq%202^n .

چون تابع f صعودی است، نتیجه می شود برای x مثبت داریم [ برای مشاهده لینک ، لطفا با نام کاربری خود وارد شوید یا ثبت نام کنید ](2^{-m}x)%5Cleq%20f(cx)%5Cleq%20f(2^nx) .

حال با تقسیم طرفین بر (f(x و استفاده از قضیه فشردگی، پاسخ حد برابر با 1 بدست می آید.

همچنین واضح است که برای c≤0، مقدار حد میتواند هر عددی باشد.

hossein_porru
28-08-2011, 18:48
داداش دکمه تشکرو پیدا کردم و همین طور نحوه تایپ فرمولهای ریاضی.
این سوال رو کی میتونه حل کنه؟؟؟؟
فاصله هر کانون از خط مجانب هذلولی به معادله [ برای مشاهده لینک ، لطفا با نام کاربری خود وارد شوید یا ثبت نام کنید ]^2-3y^2=12 کدام است.
1)2 2)3 3)[ برای مشاهده لینک ، لطفا با نام کاربری خود وارد شوید یا ثبت نام کنید ]{3} 4)4

رقم سمت راست [ برای مشاهده لینک ، لطفا با نام کاربری خود وارد شوید یا ثبت نام کنید ]^{1381} با رقم راست کدام عدد برابر است؟
1)[ برای مشاهده لینک ، لطفا با نام کاربری خود وارد شوید یا ثبت نام کنید ]^{1999} 2)[ برای مشاهده لینک ، لطفا با نام کاربری خود وارد شوید یا ثبت نام کنید ]^{2002} 3)[ برای مشاهده لینک ، لطفا با نام کاربری خود وارد شوید یا ثبت نام کنید ]^{2003} 4)[ برای مشاهده لینک ، لطفا با نام کاربری خود وارد شوید یا ثبت نام کنید ]^{2005}


اقا خسته نباشین کسی نیست بتونه اینهارو حل کنه؟

sb1370
28-08-2011, 20:40
ابا می تونی اگه می تونی اینو جواب بده!!!!!!!!!!!!!!!!
100X+220=?

---------- Post added at 09:40 PM ---------- Previous post was at 09:37 PM ----------

5555555555555555بابا جواب دادی

ms368
04-10-2011, 09:10
اثبات نامساوی مثلثی ( یا مثلثاتی )

سلام دوستان
یه تمرین دارم که باید 2 روز دیگه تحویل بدم
|a+b|≤|a|+|b|
فرمول بالا رو ممکنه اشتباه نوشته باشم ( آخه توی کافی نتم ) ممکنه به جای کوچکتر مساوی علامت بزرگتر مساوی باشه

استاد گفته که فقط نامساوی مثلثی رو اثبات کنید ( به جای عدد از حروف انگلیسی استفاده کتید )

لطفا راهنمایی کنید من که گیج گیج شدم ( همه فهمیدن جز من ) اونم جلسه اول

ALt3rnA
04-10-2011, 22:32
به توان 2 برسون
البته اگه خود نامساوی مثلثی رو میخوای اثبات کنی (این یه حالتشه فقط) باید با استقرا بری

Ship Storm
05-10-2011, 21:13
سلام بر اساتید بزرگ ریاضی
من سه تا سوال دارم که نیاز به راهنمایی دارم
اولی و دومی درمورد ریاضیات گسسته و سومی در مورد آمار و احتمالات
---
سوال اول : به ازای هر عدد صحیح مثبت n نشان دهید که[ برای مشاهده لینک ، لطفا با نام کاربری خود وارد شوید یا ثبت نام کنید ]

=====

سوال دوم : مطلوب است ظریب [ برای مشاهده لینک ، لطفا با نام کاربری خود وارد شوید یا ثبت نام کنید ] در بسط [ برای مشاهده لینک ، لطفا با نام کاربری خود وارد شوید یا ثبت نام کنید ]

=====

سوال سوم : در کشیدن 9 کارت به تصادف و با جایگزاری از یکدست کارت معمولی 52 تایی احتمال اینکه :
الف) 3A سه پیک
ب) 3 بی بی دل
ج) 3 شاه خاج
بیرون بیاید چقدر است ؟


==
پیشاپیش ممنون

ms368
06-10-2011, 10:13
به توان 2 برسون
البته اگه خود نامساوی مثلثی رو میخوای اثبات کنی (این یه حالتشه فقط) باید با استقرا بری


یعنی چی به توان 2 برسون
میتونی با حل یه تمرین به صورت ساده توضیح بدی ؟ :11:

ali_hp
07-10-2011, 13:48
سلام بر اساتید بزرگ ریاضی
من سه تا سوال دارم که نیاز به راهنمایی دارم
اولی و دومی درمورد ریاضیات گسسته و سومی در مورد آمار و احتمالات
---
سوال اول : به ازای هر عدد صحیح مثبت n نشان دهید که[ برای مشاهده لینک ، لطفا با نام کاربری خود وارد شوید یا ثبت نام کنید ]

=====

سوال دوم : مطلوب است ظریب [ برای مشاهده لینک ، لطفا با نام کاربری خود وارد شوید یا ثبت نام کنید ] در بسط [ برای مشاهده لینک ، لطفا با نام کاربری خود وارد شوید یا ثبت نام کنید ]

=====

سوال سوم : در کشیدن 9 کارت به تصادف و با جایگزاری از یکدست کارت معمولی 52 تایی احتمال اینکه :
الف) 3A سه پیک
ب) 3 بی بی دل
ج) 3 شاه خاج
بیرون بیاید چقدر است ؟


==
پیشاپیش ممنون
سلام
سوال اول نتیجه ای از بسط دو جمله ای نیوتونه.
بسط دو جمله ای نیوتون:

[ برای مشاهده لینک ، لطفا با نام کاربری خود وارد شوید یا ثبت نام کنید ](x&plus;y)^n=\sum_{k=0}^n&space;{n&space;\choose&space;k}x^ky^{ n-k}

اگه در رابطه بالا x=1 , y=-1 قرار بدیم حکم ثابت میشه!
سوال دوم هم با بسط چند جمله ای نیوتون حل میشه:

[ برای مشاهده لینک ، لطفا با نام کاربری خود وارد شوید یا ثبت نام کنید ](x_1&plus;x_2&plus;...&plus;x_k)^n=\sum_{d_1,d_2,...,d_ k}{n&space;\choose&space;d_1,d_2,...,d_k}x_1^{d_1}x_2^{d_2}... x_k^{d_k}

'که مجموع روی همه d_i های صحیح نامنفی است که مجموعشان n است.و:

[ برای مشاهده لینک ، لطفا با نام کاربری خود وارد شوید یا ثبت نام کنید ]{n&space;\choose&space;d_1,d_2,...,d_k}=\frac{n!}{d_ 1!d_2!...d_k!}

حال در این بسط قرار بدین k=4 , n=8 و x_1=2w , x_2=- x , x_3=3y , x_4= - 2z جمله ای که سوال ضریبشو خواسته به ازای d_1=3 , d_2=2 , d_3=1 , d_4=2 بوجود می اید.پس ضریب جمله خواسته شده در بسط برابر ضریب ان در تک جمله زیر است:

[ برای مشاهده لینک ، لطفا با نام کاربری خود وارد شوید یا ثبت نام کنید ]{8&space;\choose&space;3,2,1,2}(2w)^3(-x)^2(3y)(-2z)^2

نکته:هم فرمول بسط دو جمله ای هم چند جمله ای رو میشه با یک استدلال ترکیبیاتی ثابت کرد.
سوال سوم:
احتمال اینکه کارتی که از 52 کارت می کشیم Ace پیک باشه، یک پنجاهو دومه!چون با جایگذاری 9 بار این کارو انجام میدیم،پس یک ازمایش داریم با احتمال موفقیت یک پنجاهو دوم که نه بار تکرار شده و احتمال اینو می خواهیم که تعداد موفقیت ها دقیقا(؟) سه باشه.پس یک ازمایش دو جمله ای داریم با توزیع دو جمله ای با احتمال موفقیت p=1/52 و x=3 و n=9

[ برای مشاهده لینک ، لطفا با نام کاربری خود وارد شوید یا ثبت نام کنید ](X=x)={n&space;\choose&space;x&space;}p^x(1-p)^{n-x}={9&space;\choose&space;3}(\frac{1}{52})^3(\frac{51}{52})^6

البته فرمول بالا رو هم با یک استدلال ترکیبیاتی ساده میشه بدست اورد.
ب و ج هم دقیقا با الف فرقی ندارن!Ace پیکو بی بی دلو ساه خاجو هر کارت دیگه ای در این مساله با هم فرقی ندارن!چون از هر کدومشون دقیقا یکی هست و در کشیدن یک کارت از 52 تا با احتمال یکسان 1/52 رو میشن!

ali_hp
07-10-2011, 13:55
یعنی چی به توان 2 برسون
میتونی با حل یه تمرین به صورت ساده توضیح بدی ؟ :11:
سلام

[ برای مشاهده لینک ، لطفا با نام کاربری خود وارد شوید یا ثبت نام کنید ]|a|&plus;|b|\ge|a&plus;b|\Leftrightarrow&space;(|a|&plus;|b|) ^2\ge|a&plus;b|^2\\&space;\Leftrightarrow&space;|a|^2&plus;|b|^2&plus;2|a||b| \ge&space;a^2&plus;b^2&plus;2ab\\&space;\Leftrightarrow2|a||b|\ge&space;2ab&space;\L eftrightarrow&space;2|ab|\ge&space;2ab

که نامساوی اخر هم برقرار چون قدر مطلق هر عدد بزرگتر مساوی خودشه!

Farshid72
10-10-2011, 23:11
سلام من کتاب حساب دیفرانسیل دکتر نیکوکار جلد 1 رو میخوام...لطفا لینکش رو برام بزارید یا اگه تو فروم هست لینک پستشو برام بزارید خیلی خیلی ممنونتون میشم. با تشکر

avator20002000
11-10-2011, 06:00
کسی می تونه این مسئاله رو با راه حل برای من حل کنه؟
-در امارگیری از 100 خانواده.12 خانواده صاحب فرزند نبودند و 65 خانواده در بین فرزندانشان دختر داشتند.70 خانواده در بین فرزندانشان پسر داشتند.20 خانواده فقط یک فرزند پسر داشتند.
الف)چند خانواده هم فرزند دختر و هم فرزند پسر داشتند.
ب)چند خانواده فقط فرزند دختر داشته اند.
پ)چند خانواده فرزند پسر نداشته اند.
ت)چند خانواده فقط فرزند دختر و یا فقط یک فرزند پسر داشتند.
ه)چند خانواده بیش از یک فرزند پسر داشته اند.

Ship Storm
12-10-2011, 18:57
سلام بر اساتید بزرگ ریاضی
من یک سوال دارم در مورد آمار و احتمالات که نیاز به راهنمایی دارم
--
فرض کنید 5 فیوز سالم و 2 فیوز معیوب در هم شده اند
برای یافتن فیوزهای معیوب آن ها را یکی پس از دیگری و بدون جایگزاری امتحان میکنیم
احتمال اینکه هر دو فیوز معیوب پس از 3 امتحان پیدا شود چقدر است ؟

--
به نظر خودم گویا حالت های ممکن بطوری که بعد از 3 بار امتحان حتما معیوب ها پیدا بشن این ها هستند
سالم ، معیوب ، معیوب
معیوب ، سالم ، معیوب


ممنون از دوستان

ALt3rnA
13-10-2011, 18:28
سلام بر اساتید بزرگ ریاضی
من یک سوال دارم در مورد آمار و احتمالات که نیاز به راهنمایی دارم
--
فرض کنید 5 فیوز سالم و 2 فیوز معیوب در هم شده اند
برای یافتن فیوزهای معیوب آن ها را یکی پس از دیگری و بدون جایگزاری امتحان میکنیم
احتمال اینکه هر دو فیوز معیوب پس از 3 امتحان پیدا شود چقدر است ؟

--
به نظر خودم گویا حالت های ممکن بطوری که بعد از 3 بار امتحان حتما معیوب ها پیدا بشن این ها هستند
سالم ، معیوب ، معیوب
معیوب ، سالم ، معیوب

اول کل حالت هایی رو که میشه تک تک فیوز ها رو بیرون آورد رو حساب میکنی . ساده ترین راهش اینه که مثلا فرض کنی 7 تا فیوز توی یه ردیف به ترتیب چیده شدن چند حالت داریم که دو تاش خراب باشه
بعد باید تعداد حالت هایی رو که بعد 3 بار حتما فیوزای خراب پیدا میشن رو پیدا کنی . 3 مرحله یعنی 3 بار انتخاب که 2 تاش خرابن یعنی به چند حالت توی یه ردیف 3 تایی 2 تاش خرابن ؟

اگه راه حل رو نمینویسم برای اینه که ناظر انجمن یه بار گفت مستقیم جواب ندید . بعدم اینکه مطمئن نیستم فیوز ها متناسبن یا نه .
(من سوال دیدم خوک رو تو جایگشت دوری متمایز حساب کرده . هنوز نمیدونم دقیق چیا متمایزن چیا نیستن)

BOROKI1
14-10-2011, 13:48
سلام
شاید(شاید که نه حتما) سوالم براتون خنده دار باشه ولی واقعا نمیدونم
همیشه میخوایم درصد بگیریم از ماشین حساب استفاده میکنیم
ولی من موقع انجام اون محاسبه روی کاغذ نمیتونم درست انجام بدم بالاخره 30 ساله که از مدرسه اومدم بیرون :دی
مثلا سود بانک میخوام حساب کنم
مثلا من 6.5 ملیون پول دارم طریقه محاسبه کردنش با سود 17 درصد چقدر میشه؟
یا مقلا یه جنس 85 هزارتومنی 75 درصد تفیف میخوره چه جوری باسد حساب کرد؟
ببخشید اگه خیلی پیش پاافتادس

Ship Storm
14-10-2011, 13:57
اول کل حالت هایی رو که میشه تک تک فیوز ها رو بیرون آورد رو حساب میکنی . ساده ترین راهش اینه که مثلا فرض کنی 7 تا فیوز توی یه ردیف به ترتیب چیده شدن چند حالت داریم که دو تاش خراب باشه
بعد باید تعداد حالت هایی رو که بعد 3 بار حتما فیوزای خراب پیدا میشن رو پیدا کنی . 3 مرحله یعنی 3 بار انتخاب که 2 تاش خرابن یعنی به چند حالت توی یه ردیف 3 تایی 2 تاش خرابن ؟

اگه راه حل رو نمینویسم برای اینه که ناظر انجمن یه بار گفت مستقیم جواب ندید . بعدم اینکه مطمئن نیستم فیوز ها متناسبن یا نه .
(من سوال دیدم خوک رو تو جایگشت دوری متمایز حساب کرده . هنوز نمیدونم دقیق چیا متمایزن چیا نیستن)

سلام ممنون
من ولی جواب سوال رو میخوام چون با توضیحات کارم راه نمیوفته
فرض کن یک سوال امتحانیه و باید بر اساس فرمول به جواب برسی
همون مبحث احتمال شرطی و این مثال هم فکر کنم از قانون ضرب داره استفاده میکنه که میگه

[ برای مشاهده لینک ، لطفا با نام کاربری خود وارد شوید یا ثبت نام کنید ]

moh72
14-10-2011, 19:47
سلام میشه اینارو حل کنید؟ ([ ]جزصحیح)
حدالامکان تشریحی باشه :11:
[ برای مشاهده لینک ، لطفا با نام کاربری خود وارد شوید یا ثبت نام کنید ]

eh_mn
19-10-2011, 12:41
سلام میشه اینارو حل کنید؟ ([ ]جزصحیح)
حدالامکان تشریحی باشه :11:
[ برای مشاهده لینک ، لطفا با نام کاربری خود وارد شوید یا ثبت نام کنید ]


سلام

اولي: به دو نكته توجه كنيد: اول اين كه جزء صحيح، يك عدد صحيح است! و دوم اين كه عدد صحيح از جزء صحيح خارج مي‌شود يعني براي هر n طبيعي داريم


[ برای مشاهده لینک ، لطفا با نام کاربری خود وارد شوید یا ثبت نام کنید ][x&plus;n]=[x]&plus;n

دومي و پنجمي: بايد تغيير متغير مناسب انتخاب كنيد. مثلاً براي دومي [ برای مشاهده لینک ، لطفا با نام کاربری خود وارد شوید یا ثبت نام کنید ][x] و براي پنجمي [ برای مشاهده لینک ، لطفا با نام کاربری خود وارد شوید یا ثبت نام کنید ]^x. بعد معادله‌ي درجه‌ي دو را برحسب t حل كنيد. بعد از محاسبه‌ي مقدار t با استفاده از رابطه‌ي تغيير متغير مقادير x را محاسبه كنيد.


چهارمي: توجه كنيد كه


[ برای مشاهده لینک ، لطفا با نام کاربری خود وارد شوید یا ثبت نام کنید ][x]&plus;[-x]=\begin{cases}1&x\not\in\mathbb{Z}\cr0&x\in\mathbb{Z}\end{cases}

بنا بر اين دو حالت براي x در نظر بگيريد: اينكه صحيح باشد و نباشد!

Ship Storm
21-10-2011, 21:34
سلام سوالی داشتم ممنون میشم راهنمایی کنید و جواب رو به ما بدید
---
جعبه ای حاوی 18 توپ تنیس است که 8 توپ از ان ها کار نکرده اندفرض کنید 3 توپ به تصادف خارج کرده و بعد از انجام بازی ان ها را به دورن جعبه برگردانیم ، اگر در دور دوم بازی 3 توپ دیگر از این جعبه ها خارج کنیم ، احتمال اینکه همه این توپ ها کار نکرده باشند چقدر است ؟
(راهنمایی : پیشامدهای B4 , B3 , B2, B1 را به ترتیب به اینصورت تعریف میکنیم که از سه توپ مرحله اول 1 توپ کارکرده باشد ، 2 توپ کار کرده باشد ، 3 توپ کار کرده باشد . هیچ توپی کار نکرده باشد پیشامد A را به اینصورت تعریف میکنیم که در مرحله دوم هر سه توپ کار نکرده باشند)

Kaveh619
23-10-2011, 22:39
از کجا می تونم پاسخ سوالات فصل اول حسابان رو پیدا کنم؟اگه ممکنه یه لینکی بدید..........مرسی

hamed6672
24-10-2011, 13:12
سلام میشه اینارو حل کنید؟ ([ ]جزصحیح)
حدالامکان تشریحی باشه :11:
[ برای مشاهده لینک ، لطفا با نام کاربری خود وارد شوید یا ثبت نام کنید ]

حل مسأله شما: ( البته در توضیح دادن افتضاحم ، امیدوارم این بار خوب توضیح بدم )

[x +2 ] عددی صحیح است پس از براکت سومی (از سمت چپ بیرون می آید ) به همین ترتیپ [x ] +[x +2 ]

از براکت دوم بیرون می آیند و... در نهایت شکل براکت شما به صورت زیر می شود ( اگه خوب توضیح ندادم بگین تا بیشتر توضیح بدم )



6=[2+x]+[x]+[x]+[x]


بعد عدد 2 هم از براکت خارج میشه که درنتیجه :


[ برای مشاهده لینک ، لطفا با نام کاربری خود وارد شوید یا ثبت نام کنید ]

پس x بین 1 بسته تا 2 ی باز است.

اگر بد توضیح دادم شرمنده...

hamed6672
24-10-2011, 13:52
سلام میشه اینارو حل کنید؟ ([ ]جزصحیح)
حدالامکان تشریحی باشه :11:
[ برای مشاهده لینک ، لطفا با نام کاربری خود وارد شوید یا ثبت نام کنید ]

[ برای مشاهده لینک ، لطفا با نام کاربری خود وارد شوید یا ثبت نام کنید ]

hamed6672
24-10-2011, 14:21
سلام میشه اینارو حل کنید؟ ([ ]جزصحیح)
حدالامکان تشریحی باشه :11:
[ برای مشاهده لینک ، لطفا با نام کاربری خود وارد شوید یا ثبت نام کنید ]


سومی و چهارمی رو به زودی جواب میدم ولی پنجمی خیلی واضحه و هرگز نیاز به تغییر متغیر نداره .

فقط باید عدد 4 رو به طرف دیگر معادله ببرین تا 4- بشه .

این نکته هم که حتما میدونین که یه عدد وقتی به توان هر عددی میرسه هرگز حاصلش منفی نمیشه .

پس حاصل جمع دو عدد نامنفی هم هرگز منفی نمیشه ..

پس مسئله جواب ندارد . به همین راحتی..

hamed6672
24-10-2011, 14:40
از کجا می تونم پاسخ سوالات فصل اول حسابان رو پیدا کنم؟اگه ممکنه یه لینکی بدید..........مرسی


اینجا رو ببین به دردت میخوره :



برای مشاهده محتوا ، لطفا وارد شوید یا ثبت نام کنید

alim4
24-10-2011, 16:24
سلام
یک دنباله گویا همگرا به عدد اصم به چه شکل است؟مثال بزنید.

(برای مثال دنباله اصم همگرا به اصم مثل دنباله زیر که همگرا به رادیکال 2 است)
[ برای مشاهده لینک ، لطفا با نام کاربری خود وارد شوید یا ثبت نام کنید ]


ممنون می شم جواب بدید.

pouya_99999
24-10-2011, 17:52
سلام

یه سوال خیلی ساده ذهنمو در گیر کرده .. یادم رفته چجوری حلش کنم

اینکه دو تا عدد داده گفته a , b رو پیدا کنید نسبت به هم اول باشند a , b توان های دو تا عددن

hamed6672
24-10-2011, 18:27
سلام
یک دنباله گویا همگرا به عدد اصم به چه شکل است؟مثال بزنید.

(برای مثال دنباله اصم همگرا به اصم مثل دنباله زیر که همگرا به رادیکال 2 است)
[ برای مشاهده لینک ، لطفا با نام کاربری خود وارد شوید یا ثبت نام کنید ]


ممنون می شم جواب بدید.


دنباله معروف [ برای مشاهده لینک ، لطفا با نام کاربری خود وارد شوید یا ثبت نام کنید ] که با وجود داشتن جملات گویا به عدد اصم نپر (...e=2.7182) همگرا

است .


[ برای مشاهده لینک ، لطفا با نام کاربری خود وارد شوید یا ثبت نام کنید ]



---------- Post added at 08:27 PM ---------- Previous post was at 08:24 PM ----------


سلام

یه سوال خیلی ساده ذهنمو در گیر کرده .. یادم رفته چجوری حلش کنم

اینکه دو تا عدد داده گفته a , b رو پیدا کنید نسبت به هم اول باشند a , b توان های دو تا عددن


میشه واضح تر بگی ؟؟؟

من اصلا نمی فهمم سوالت چیه ؟؟؟؟

pouya_99999
24-10-2011, 18:45
سوال اینه

مقادیر a , b را چنان بیابید که دو عدد m=(18^4)*(125^a-5

و n- 9^(b-2)*25^2 نسبت به هم اول باشند

^ توانن .. با تشکر پیشاپیش ممنون سوال خیلی سادس

hamed6672
24-10-2011, 19:24
سوال اینه

مقادیر a , b را چنان بیابید که دو عدد m=(18^4)*(125^a-5

و n- 9^(b-2)*25^2 نسبت به هم اول باشند

^ توانن .. با تشکر پیشاپیش ممنون سوال خیلی سادس

پاسخ :


[ برای مشاهده لینک ، لطفا با نام کاربری خود وارد شوید یا ثبت نام کنید ]

skyzare
24-10-2011, 21:36
با سلام ....:20:

می خواستم بدونم جواب این انتگرال چی میشه ...من خودم یه بار تغییر متغیر رو زدم ولی بعدش رو بلد نبودم .:41:


[ برای مشاهده لینک ، لطفا با نام کاربری خود وارد شوید یا ثبت نام کنید ] %7B3%7D%7Ddx

alim4
24-10-2011, 21:44
دنباله معروف [ برای مشاهده لینک ، لطفا با نام کاربری خود وارد شوید یا ثبت نام کنید ] که با وجود داشتن جملات گویا به عدد اصم نپر (...e=2.7182) همگرا

است .


[ برای مشاهده لینک ، لطفا با نام کاربری خود وارد شوید یا ثبت نام کنید ]





اگر امکان داره اثباتش هم بنویسید.
باز هم ممنون.

pp8khat
24-10-2011, 22:26
با سلام ....:20:

می خواستم بدونم جواب این انتگرال چی میشه ...من خودم یه بار تغییر متغیر رو زدم ولی بعدش رو بلد نبودم .:41:


[ برای مشاهده لینک ، لطفا با نام کاربری خود وارد شوید یا ثبت نام کنید ] %7B3%7D%7Ddx


صورت رو با 2 جمع و کم کنید،میشه انتگرال 2 منهای انتگرال 2 روی x^3+1 که اینم تجزیه می شه به (x+1)(x^2-x+1) که اینم می نویسیم انتگرال A روی x+1 به علاوه ی Bx+c روی x^2-x+1 که همون روش تجزیه کسر هستش و ضرایب رو می یابی بعدشم اگه تغییر متغیری لازم بود می دی و دست آخر مینتگرالی.
این رو هم ببینی بد نیست:

برای مشاهده محتوا ، لطفا وارد شوید یا ثبت نام کنید

hamed6672
24-10-2011, 23:10
با سلام ....:20:

می خواستم بدونم جواب این انتگرال چی میشه ...من خودم یه بار تغییر متغیر رو زدم ولی بعدش رو بلد نبودم .:41:


[ برای مشاهده لینک ، لطفا با نام کاربری خود وارد شوید یا ثبت نام کنید ] %7B3%7D%7Ddx



لازم به تغییر متغیر نیست ، جواب رو کامل برات نوشتم ، البته چون یکم طولانی بود اسکن کردم ، امیدوارم بد نباشه:

[ برای مشاهده لینک ، لطفا با نام کاربری خود وارد شوید یا ثبت نام کنید ]

skyzare
25-10-2011, 00:16
با سلام ....

ممنون از پاسخ جامع و کامل شما ؛ گرفتم چی شد .....:20:

موفق باشید .......

hamed6672
25-10-2011, 00:20
اگر امکان داره اثباتش هم بنویسید.
باز هم ممنون.


[ برای مشاهده لینک ، لطفا با نام کاربری خود وارد شوید یا ثبت نام کنید ]

hamed6672
25-10-2011, 00:28
با سلام ....

ممنون از پاسخ جامع و کامل شما ؛ گرفتم چی شد .....:20:

موفق باشید .......


به نظرت اگه یه روز معلم بشم ، معلم خوبی میشم ؟؟؟!!!!!

آخه معلمی رو خیلی دوست دارم .

skyzare
25-10-2011, 00:44
به نظرت اگه یه روز معلم بشم ، معلم خوبی میشم ؟؟؟!!!!!

آخه معلمی رو خیلی دوست دارم .

با سلام ....

قطعا و بدون شک ....[ برای مشاهده لینک ، لطفا با نام کاربری خود وارد شوید یا ثبت نام کنید ] [ برای مشاهده لینک ، لطفا با نام کاربری خود وارد شوید یا ثبت نام کنید ]

موفق باشید .

alim4
25-10-2011, 08:36
دست شما درد نکنه ; من این سوال را از یک معلم پرسیدم اما نتونست جواب بده.;)

lebesgue
25-10-2011, 13:08
سلام
یک دنباله گویا همگرا به عدد اصم به چه شکل است؟مثال بزنید.

(برای مثال دنباله اصم همگرا به اصم مثل دنباله زیر که همگرا به رادیکال 2 است)
[ برای مشاهده لینک ، لطفا با نام کاربری خود وارد شوید یا ثبت نام کنید ]


ممنون می شم جواب بدید.
...3.141592=π
در نتیجه یک نمونه ساده (علاوه بر نمونه ای که دوستمان آوردند) به صورت زیر است:
3
3.1
3.14
3.141
3.1415
.
.
.

hamed6672
25-10-2011, 13:43
...3.141592=π
در نتیجه یک نمونه ساده (علاوه بر نمونه ای که دوستمان آوردند) به صورت زیر است:
3
3.1
3.14
3.141
3.1415
.
.
.

با عرض پوزش پاسخ شما درست نیست .

این نه یک دنباله به حساب می آید و نه این که با وجود گویا بودن به یک عدد اصم همگرا است و نه این که جمله عمومی دارد و نه...

در ضمن یکی از سخت ترین کارها توی دنباله ها نوشتن چنین دنباله هایی است که شرایط خاصی دارند .
به خصوص دنباله ای که با وجود گویا بودن به اصم میل کند .

hamed6672
25-10-2011, 15:03
دوستانی که توی سایت عضو نیستند میتونند سوالاشونو برام میل کنند ، اگه تونستم در اسرع وقت جواب میدم :



PRO1372@YAHOO.COM

زوار
25-10-2011, 15:20
با سلام این انتگرال را کسی میتونه حل کنه ممنونم
<img src="[ برای مشاهده لینک ، لطفا با نام کاربری خود وارد شوید یا ثبت نام کنید ] \frac{dx}{x^{3}(x^{3}+x+1)}" title="\int \frac{dx}{x^{3}(x^{3}+x+1)}" />
هر کاری میکنم نمیتونم نمایشش بدم فقط کدش میاد اگه کسی میدونه بگه چه جوری نمایشش بدم ممنونم
این عکسش
[ برای مشاهده لینک ، لطفا با نام کاربری خود وارد شوید یا ثبت نام کنید ]
[ برای مشاهده لینک ، لطفا با نام کاربری خود وارد شوید یا ثبت نام کنید ] %5E%7B3%7D&plus;x&plus;1%29%7D

hamed6672
25-10-2011, 15:34
با سلام این انتگرال را کسی میتونه حل کنه ممنونم
<img src="[ برای مشاهده لینک ، لطفا با نام کاربری خود وارد شوید یا ثبت نام کنید ] \frac{dx}{x^{3}(x^{3}+x+1)}" title="\int \frac{dx}{x^{3}(x^{3}+x+1)}" />
هر کاری میکنم نمیتونم نمایشش بدم فقط کدش میاد اگه کسی میدونه بگه چه جوری نمایشش بدم ممنونم
[ برای مشاهده لینک ، لطفا با نام کاربری خود وارد شوید یا ثبت نام کنید ] %5E%7B3%7D&plus;x&plus;1%29%7D


اگه تو word تایپش کردی printscr کن و عکستو آپلود کن تو یه سایت ( من خودم تو سایت up.--------.com آپلود میکنم ) بعد آدرس عکس رو از نوار بالای ویرایش متن تو متنت وارد کن .

اگه هم توی اینترنت تایپش کردی یا printscr کن یا اینکه عکستو save کن و خودت دوباره آپلودش کن .

اینجوری بهتره.

skyzare
25-10-2011, 15:44
با سلام این انتگرال را کسی میتونه حل کنه ممنونم
<img src="[ برای مشاهده لینک ، لطفا با نام کاربری خود وارد شوید یا ثبت نام کنید ] \frac{dx}{x^{3}(x^{3}+x+1)}" title="\int \frac{dx}{x^{3}(x^{3}+x+1)}" />
هر کاری میکنم نمیتونم نمایشش بدم فقط کدش میاد اگه کسی میدونه بگه چه جوری نمایشش بدم ممنونم
[ برای مشاهده لینک ، لطفا با نام کاربری خود وارد شوید یا ثبت نام کنید ] %5E%7B3%7D&plus;x&plus;1%29%7D

با سلام ....

منظورتون این هست دیگه ...؟؟



[ برای مشاهده لینک ، لطفا با نام کاربری خود وارد شوید یا ثبت نام کنید ] %5E%7B3%7D&plus;x&plus;1%29%7D
برای نمایش هم اول دکمه " وارد نمودن عکس " رو بزنید بعدش ادرس url رو توی کادری که باز میشه کپی کنید .



من این رو یه مقداریش حل کردم اما توی تکه اخرش میمونم :41: اگه کسی کمک کنه ممنون میشم .و اینکه ایا روش دیگه ای هست ؟



[ برای مشاهده لینک ، لطفا با نام کاربری خود وارد شوید یا ثبت نام کنید ] 7D&plus;x&plus;1%29%7D=%5Cfrac%7BAx%5E2&plus;Bx&plus;C%7D%7Bx%5E3%7D&plus;% 5Cfrac%7BDx%5E2&plus;Fx&plus;G%7D%7Bx%5E3&plus;x&plus;1%7D=


[ برای مشاهده لینک ، لطفا با نام کاربری خود وارد شوید یا ثبت نام کنید ] 7D&plus;x&plus;1%29%7D=%5Cfrac%7BAx%5E5&plus;Ax%5E3&plus;Ax%5E2&plus;Bx%5E4 &plus;Bx%5E2&plus;Bx&plus;Cx%5E3&plus;Cx&plus;C&plus;Dx%5E5&plus;Fx%5E4&plus;Gx%5E3%7D%7Bx %5E%7B3%7D%28x%5E%7B3%7D&plus;x&plus;1%29%7D=

چون دو کسر با هم برابر هستند پس مخرج و صورت هم باید با هم برابر باشند بعد از برابر قرار دادن صورت ها داریم :


[ برای مشاهده لینک ، لطفا با نام کاربری خود وارد شوید یا ثبت نام کنید ] C%5C&space;A&plus;C&plus;G=0%5C%5C&space;A&plus;B=0%5C%5C&space;B&plus;C=0%5C%5C&space;C=1&space;%5C end%7Bmatrix%7D

با حل معادله بالا هر یک از مجهولات به صورت زیر به دست میاد :


[ برای مشاهده لینک ، لطفا با نام کاربری خود وارد شوید یا ثبت نام کنید ]


پس میتونیم به جای کسر صورت سوال این جوری بنویسیم :


[ برای مشاهده لینک ، لطفا با نام کاربری خود وارد شوید یا ثبت نام کنید ] 7D&plus;x&plus;1%29%7D=%5Cfrac%7Bx%5E2-x&plus;1%7D%7Bx%5E3%7D&plus;%5Cfrac%7B-x%5E2&plus;x-2%7D%7Bx%5E3&plus;x&plus;1%7D

پس داریم :

[ برای مشاهده لینک ، لطفا با نام کاربری خود وارد شوید یا ثبت نام کنید ] 5E%7B3%7D&plus;x&plus;1%29%7Ddx=%5Cint&space;%5Cleft&space;%28&space;%5Cfrac%7 Bx%5E2-x&plus;1%7D%7Bx%5E3%7D&plus;%5Cfrac%7B-x%5E2&plus;x-2%7D%7Bx%5E3&plus;x&plus;1%7D&space;%5Cright&space;%29dx=

حالا انتگرال جمع ها رو جدا جدا مینویسیم که میشه :


[ برای مشاهده لینک ، لطفا با نام کاربری خود وارد شوید یا ثبت نام کنید ] %5Cfrac%7B1%7D%7Bx%7D-%5Cfrac%7B1%7D%7B2x%5E2%7D

و دومی که من بلد نیستم ! [ برای مشاهده لینک ، لطفا با نام کاربری خود وارد شوید یا ثبت نام کنید ]


[ برای مشاهده لینک ، لطفا با نام کاربری خود وارد شوید یا ثبت نام کنید ]

hamed6672
25-10-2011, 18:16
با سلام ....

منظورتون این هست دیگه ...؟؟


[ برای مشاهده لینک ، لطفا با نام کاربری خود وارد شوید یا ثبت نام کنید ] %5E%7B3%7D&plus;x&plus;1%29%7D
برای نمایش هم اول دکمه " وارد نمودن عکس " رو بزنید بعدش ادرس url رو توی کادری که باز میشه کپی کنید .



من این رو یه مقداریش حل کردم اما توی تکه اخرش میمونم :41: اگه کسی کمک کنه ممنون میشم .و اینکه ایا روش دیگه ای هست ؟



[ برای مشاهده لینک ، لطفا با نام کاربری خود وارد شوید یا ثبت نام کنید ] 7D&plus;x&plus;1%29%7D=%5Cfrac%7BAx%5E2&plus;Bx&plus;C%7D%7Bx%5E3%7D&plus;% 5Cfrac%7BDx%5E2&plus;Fx&plus;G%7D%7Bx%5E3&plus;x&plus;1%7D=


[ برای مشاهده لینک ، لطفا با نام کاربری خود وارد شوید یا ثبت نام کنید ] 7D&plus;x&plus;1%29%7D=%5Cfrac%7BAx%5E5&plus;Ax%5E3&plus;Ax%5E2&plus;Bx%5E4 &plus;Bx%5E2&plus;Bx&plus;Cx%5E3&plus;Cx&plus;C&plus;Dx%5E5&plus;Fx%5E4&plus;Gx%5E3%7D%7Bx %5E%7B3%7D%28x%5E%7B3%7D&plus;x&plus;1%29%7D=

چون دو کسر با هم برابر هستند پس مخرج و صورت هم باید با هم برابر باشند بعد از برابر قرار دادن صورت ها داریم :


[ برای مشاهده لینک ، لطفا با نام کاربری خود وارد شوید یا ثبت نام کنید ] C%5C&space;A&plus;C&plus;G=0%5C%5C&space;A&plus;B=0%5C%5C&space;B&plus;C=0%5C%5C&space;C=1&space;%5C end%7Bmatrix%7D

با حل معادله بالا هر یک از مجهولات به صورت زیر به دست میاد :


[ برای مشاهده لینک ، لطفا با نام کاربری خود وارد شوید یا ثبت نام کنید ]


پس میتونیم به جای کسر صورت سوال این جوری بنویسیم :


[ برای مشاهده لینک ، لطفا با نام کاربری خود وارد شوید یا ثبت نام کنید ] 7D&plus;x&plus;1%29%7D=%5Cfrac%7Bx%5E2-x&plus;1%7D%7Bx%5E3%7D&plus;%5Cfrac%7B-x%5E2&plus;x-2%7D%7Bx%5E3&plus;x&plus;1%7D

پس داریم :

[ برای مشاهده لینک ، لطفا با نام کاربری خود وارد شوید یا ثبت نام کنید ] 5E%7B3%7D&plus;x&plus;1%29%7Ddx=%5Cint&space;%5Cleft&space;%28&space;%5Cfrac%7 Bx%5E2-x&plus;1%7D%7Bx%5E3%7D&plus;%5Cfrac%7B-x%5E2&plus;x-2%7D%7Bx%5E3&plus;x&plus;1%7D&space;%5Cright&space;%29dx=

حالا انتگرال جمع ها رو جدا جدا مینویسیم که میشه :


[ برای مشاهده لینک ، لطفا با نام کاربری خود وارد شوید یا ثبت نام کنید ] %5Cfrac%7B1%7D%7Bx%7D-%5Cfrac%7B1%7D%7B2x%5E2%7D

و دومی که من بلد نیستم ! [ برای مشاهده لینک ، لطفا با نام کاربری خود وارد شوید یا ثبت نام کنید ]


[ برای مشاهده لینک ، لطفا با نام کاربری خود وارد شوید یا ثبت نام کنید ]



فکر نکنم قسمت دوم این انتگرال جواب داشته باشه ، حتی می تونین این انتگرال در سایت
برای مشاهده محتوا ، لطفا وارد شوید یا ثبت نام کنید
وارد کنین تا متوجه ارور بشین .

این سایت جواب آخر انتگرال ها رو محاسبه میکنه که اگر این انتگرال رو بهش بدین جوابی به شما نمیده به غیر از اون قسمت اول که دوستمون حساب کرده.




[ برای مشاهده لینک ، لطفا با نام کاربری خود وارد شوید یا ثبت نام کنید ]

lebesgue
25-10-2011, 18:30
با عرض پوزش پاسخ شما درست نیست .

این نه یک دنباله به حساب می آید و نه این که با وجود گویا بودن به یک عدد اصم همگرا است و نه این که جمله عمومی دارد و نه...

در ضمن یکی از سخت ترین کارها توی دنباله ها نوشتن چنین دنباله هایی است که شرایط خاصی دارند .
به خصوص دنباله ای که با وجود گویا بودن به اصم میل کند .

اتفاقاً دهها جمله عمومی میتوان برای آن نوشت. یکی از بدیهی ترین ها:

[ برای مشاهده لینک ، لطفا با نام کاربری خود وارد شوید یا ثبت نام کنید ]{120}%20a_n=10^{-n}%5Cleft%20%5Clfloor%2010^{n}%5Cpi%20%5Cright%20% 5Crfloor%5C:%20%5C:%20%5C:%20%5C:%20%5C:%20%5C:%20 %5C:%20n=1,2,...
[ برای مشاهده لینک ، لطفا با نام کاربری خود وارد شوید یا ثبت نام کنید ] یک تابع از [ برای مشاهده لینک ، لطفا با نام کاربری خود وارد شوید یا ثبت نام کنید ]{N} به [ برای مشاهده لینک ، لطفا با نام کاربری خود وارد شوید یا ثبت نام کنید ]{Q} بوده و همچنین همگرا به [ برای مشاهده لینک ، لطفا با نام کاربری خود وارد شوید یا ثبت نام کنید ] است، در نتیجه یک دنباله گویا می باشد که به یک عدد گنگ همگراست. شاید لازم باشد که مروری بر تعریف دنباله ها داشته باشید.

وجود چنین نمونه هایی در اعداد گویا، نشان دهنده ناتمامیت اعداد گویاست. برای اطلاعات بیشتر، لینک زیر را ببینید:

برای مشاهده محتوا ، لطفا وارد شوید یا ثبت نام کنید

hamed6672
25-10-2011, 18:44
اتفاقاً دهها جمله عمومی میتوان برای آن نوشت. یکی از بدیهی ترین ها:

[ برای مشاهده لینک ، لطفا با نام کاربری خود وارد شوید یا ثبت نام کنید ] ight%20%5Crfloor%5C:%20%5C:%20%5C:%20%5C:%20%5C:%2 0%5C:%20%5C:%20n=1,2,...
[ برای مشاهده لینک ، لطفا با نام کاربری خود وارد شوید یا ثبت نام کنید ] یک تابع از [ برای مشاهده لینک ، لطفا با نام کاربری خود وارد شوید یا ثبت نام کنید ] به [ برای مشاهده لینک ، لطفا با نام کاربری خود وارد شوید یا ثبت نام کنید ] بوده و همچنین همگرا به [ برای مشاهده لینک ، لطفا با نام کاربری خود وارد شوید یا ثبت نام کنید ] است، در نتیجه یک دنباله گویا می باشد که به یک عدد گنگ همگراست. شاید لازم باشد که مروری بر تعریف دنباله ها داشته باشید.

وجود چنین نمونه هایی در اعداد گویا، نشان دهنده ناتمامیت اعداد گویاست. برای اطلاعات بیشتر، لینک زیر را ببینید:

برای مشاهده محتوا ، لطفا وارد شوید یا ثبت نام کنید


دوست عزیز من از شما عذر می خوام ولی منظورم این جمله شما بود :

...3.141592=π
در نتیجه یک نمونه ساده (علاوه بر نمونه ای که دوستمان آوردند) به صورت زیر است:
3
3.1
3.14
3.141
3.1415
.
.
.

من گفتم که دنباله شما کدومه ؟؟

یعنی جمله عمومی این دنباله ای که بالا نوشتین .

از پاسختون هم ممنون.
موفق باشید...

lebesgue
25-10-2011, 19:35
[ برای مشاهده لینک ، لطفا با نام کاربری خود وارد شوید یا ثبت نام کنید ] %5E%7B3%7D&plus;x&plus;1%29%7D


تمام توابع گویا (توابع به فرم (P(x)/Q(x که (P(x و (Q(x چند جمله ای هستند)، تابع اولیه در توابع مقدماتی دارند. روند کلی برای حل این انتگرال ها، تجزیه (Q(x به عوامل درجه 1 و 2 (چنین امکانی را قضیه اساسی جبر تضمین می کند) و سپس تجزیه کسر می باشد. مشکلی که ممکن است در حل انتگرال بالا پیش بیاید، اینست که ریشه های عامل x^3+x+1 در مخرج، چندان سرراست نیستند:

برای مشاهده محتوا ، لطفا وارد شوید یا ثبت نام کنید
پس تجزیه آن به عوامل درجه 1 و 2، به صورت روبرو است: [ برای مشاهده لینک ، لطفا با نام کاربری خود وارد شوید یا ثبت نام کنید ](x-a)(x^2+b)
که a و b بر حسب ریشه های x^3+x+1 قابل بیان هستند. این در واقع همان کاری است که سایت Wolfram انجام می دهد.

hamed6672
25-10-2011, 20:15
[ برای مشاهده لینک ، لطفا با نام کاربری خود وارد شوید یا ثبت نام کنید ] %5E%7B3%7D&plus;x&plus;1%29%7D



تمام توابع گویا (توابع به فرم (P(x)/Q(x که (P(x و (Q(x چند جمله ای هستند)، تابع اولیه در توابع مقدماتی دارند. روند کلی برای حل این انتگرال ها، تجزیه (P(x و (Q(x به عوامل درجه 1 و 2 (چنین امکانی را قضیه اساسی جبر تضمین می کند) و سپس تجزیه کسر می باشد. مشکلی که ممکن است در حل انتگرال بالا پیش بیاید، اینست که ریشه های عامل x^3+x+1 در مخرج، چندان سرراست نیستند:

برای مشاهده محتوا ، لطفا وارد شوید یا ثبت نام کنید
پس تجزیه آن به عوامل درجه 1 و 2، به صورت روبرو است: [ برای مشاهده لینک ، لطفا با نام کاربری خود وارد شوید یا ثبت نام کنید ](x-a)(x^2+b)
که a و b بر حسب ریشه های x^3+x+1 قابل بیان هستند. این در واقع همان کاری است که سایت Wolfram انجام می دهد.

بله منظور من هم همین موضوع بود ، چون چه از روش تجزیه کسر ، چه از جزء به جزء ما در تجزیه چند جمله ای x^3+x+1 دچار مشکل میشیم .

پس این قضیه که دارای ریشه های گویا نباشه کاملا واضحه .

arshamis
25-10-2011, 21:04
سلام.ممنون از اینکه دنبال ریاضیات اول راهنمایی گشتین.پیداش کردم.بازم ممنون.[ برای مشاهده لینک ، لطفا با نام کاربری خود وارد شوید یا ثبت نام کنید ]

alim4
25-10-2011, 23:42
ببخشید باز هم یک سوال دیگه.
یک دنباله گویا همگرا به رادیکال 2 با اثباتش مثال بزنید.
ممنون میشم.

hamed6672
25-10-2011, 23:58
ببخشید باز هم یک سوال دیگه.
یک دنباله گویا همگرا به رادیکال 2 با اثباتش مثال بزنید.
ممنون میشم.

راستش اگه بخواهیم مثل دنباله ای که دوستمون نوشت یه دنباله بنویسیم میتونیم بنویسیم :



[ برای مشاهده لینک ، لطفا با نام کاربری خود وارد شوید یا ثبت نام کنید ]

برای اثبات هم حد این عبارت رو در بینهایت حساب می کنین که با توجه به این که حد براکت u در بینهایت خود u می شود ، دنباله به سمت رادیکال 2 همگرا میشه.

wxy4u
27-10-2011, 22:51
سلام
دوستان کسی لینک حل مسائل درفرانسیل و انتگرال هیت هولد جلد اول و دوم رو نداره؟!

ali_hp
28-10-2011, 19:18
کسی می تونه این مسئاله رو با راه حل برای من حل کنه؟
-در امارگیری از 100 خانواده.12 خانواده صاحب فرزند نبودند و 65 خانواده در بین فرزندانشان دختر داشتند.70 خانواده در بین فرزندانشان پسر داشتند.20 خانواده فقط یک فرزند پسر داشتند.
الف)چند خانواده هم فرزند دختر و هم فرزند پسر داشتند.
ب)چند خانواده فقط فرزند دختر داشته اند.
پ)چند خانواده فرزند پسر نداشته اند.
ت)چند خانواده فقط فرزند دختر و یا فقط یک فرزند پسر داشتند.
ه)چند خانواده بیش از یک فرزند پسر داشته اند.
سلام
A : خانواده هایی که فرزند دختر دارند.
B : خانواده هایی که فرزند پسر دارند.
پس طبق فرض های مساله داریم:

[ برای مشاهده لینک ، لطفا با نام کاربری خود وارد شوید یا ثبت نام کنید ](A\cup&space;B)=100-12=88&space;,&space;n(A)=65&space;,&space;n(B)=70

الف) باید تعداد اعضای اشتراک A و B را بدست اوریم:

[ برای مشاهده لینک ، لطفا با نام کاربری خود وارد شوید یا ثبت نام کنید ](A\cap&space;B)=n(A)&plus;n(B)-n(A\cup&space;B)=65&plus;70-88=47

ب)باید تعداد اونایی که دختر دارن منهای تعداد اونایی بشه که هم دختر دارن هم پسر پس جواب میشه شصت و پنج منهای چهل و هفت یعنی هیجده.
پ)میشه صد منهای هفتاد یعنی سی.
ت)اینم که واضحه،چون اونایی که دختر دارن با اونایی که پسر دارن هیچ اشتراکی ندارن،جواب میشه هیجده باضافه بیست یعنی سی و هشت.
ه) باید اونایی که پسر دارن منهای اونایی که دقیقا یک پسر دارن بشه.یعنی هفتاد منهای بیست که میشه 50.

Greedy
31-10-2011, 13:42
سلام ممنون میشم به این سوالات حد جواب بدین
چون اینجا نمیشد تایپ کرد تو paint نوشتم صورت سوالاتو

[ برای مشاهده لینک ، لطفا با نام کاربری خود وارد شوید یا ثبت نام کنید ]

taghi_ramzi
31-10-2011, 15:00
سلام
ظاهراً یه فرمولی هست که میشه فهمید فاکتوریل یه عدد چند رقمیه
بنده این فرمول رو میخوام.
با تشکر

hts1369
31-10-2011, 20:13
دوستان اگه این سوال رو جواب بدید خیلی ممنون میشم
این جور سوالها رو بلدم ولی در مورد اینکه در صفحه ی yz باشه نمیدونم باید برای مرکزش x رو صفر در نظر بگیرم
معادله ي کره اي را بيابيد که از نقاط (4و0و0) و (3و1و2)و (6و2و0) بگذرد و مرکزش در صفحه ي yz باشد

MasterGeek
01-11-2011, 09:39
دوستان اگه این سوال رو جواب بدید خیلی ممنون میشم
این جور سوالها رو بلدم ولی در مورد اینکه در صفحه ی yz باشه نمیدونم باید برای مرکزش x رو صفر در نظر بگیرم
معادله ي کره اي را بيابيد که از نقاط (4و0و0) و (3و1و2)و (6و2و0) بگذرد و مرکزش در صفحه ي yz باشد

فکر میکنم خودتون میتونین حلش کنین ولی به عنوان یه نکته:

1. مرکزش وقتی توی صفحه ی yz باشه، یعنی اینکه x=0 هست واسه نقطه ی مرکز (یعنی توی اون معادله ای که مینویسن اینطوریه:

x^2+(y-b)^2+(z-c)^2=r

حالا دیگه حله، سه معادله و سه مجهول که توسط جاگذاری اون سه نقطه به دست میاد


2. الان یکی از نکات خودش روی صفحه ی yz هست (6و2و0) پس مرکز هم روی همین صفحه هست و در نتیجه ی یه نقطه ی قرینه دقیقا روی همین صفحه است. از روی دو نطقه ی دیگه میشه شعاع رو بدست آورد اگر کمی روش کار بشه به نظرم میشه یه راه جایگزین رو (عوض راه کلاسیک) برای حل مسئله در پیش گرفت

davy jones
01-11-2011, 10:26
سلام به همه ی دوستان!!



سلام ممنون میشم به این سوالات حد جواب بدین
چون اینجا نمیشد تایپ کرد تو paint نوشتم صورت سوالاتو

[ برای مشاهده لینک ، لطفا با نام کاربری خود وارد شوید یا ثبت نام کنید ]
سلام.

جواب سوال اول:



حاصل نهایی برابر با صفره چون صفر در یک عبارت کراندار داره ضرب میشه. (سینوس حتی اگه داخل آرگومانش به سمت بی نهایت بره همواره بین 1 و 1- خواهد بود)




------------------------------------
جواب سوال دوم:


[ برای مشاهده لینک ، لطفا با نام کاربری خود وارد شوید یا ثبت نام کنید ] 5Ccos&space;x.%5Csin&space;x-%5Ctan&space;x%7D%7Bx%5E%7B2%7D%5Csin&space;x%7D=%5Clim_%7Bx%5 Crightarrow&space;0%7D%5Cfrac%7B%5Ccos%5E%7B2%7Dx.%5Csin &space;x-%5Csin&space;x%7D%7Bx%5E%7B2%7D%5Csin&space;x.%5Ccos&space;x%7D=%5Cl im_%7Bx%5Crightarrow&space;0%7D%5Cfrac%7B%5Csin&space;x%28%5Cc os&space;%5E%7B2%7Dx-1%29%7D%7Bx%5E%7B2%7D%5Csin&space;x.%5Ccos&space;x%7D=%5Clim_% 7Bx%5Crightarrow&space;0%7D%5Cfrac%7B-%5Csin&space;%5E%7B3%7Dx%7D%7Bx%5E%7B2%7D%5Csin&space;x.%5Ccos &space;x%7D=%5Clim_%7Bx%5Crightarrow&space;0%7D%5Cfrac%7B-x%5E%7B3%7D%7D%7Bx%5E%7B2%7D.x.%5Ccos&space;x%7D=%5Clim_ %7Bx%5Crightarrow&space;0%7D%5Cfrac%7B-1%7D%7B%5Ccos&space;x%7D=-1


----------------------------------------------
جواب سوال سوم با استفاده از قائده هوپیتال:


[ برای مشاهده لینک ، لطفا با نام کاربری خود وارد شوید یا ثبت نام کنید ] &space;%7D%7B4%7D%7D%5Cfrac%7B%5Csin&space;%28x-%5Cfrac%7B%5Cpi&space;%7D%7B4%7D%29%7D%7B2%5Csin&space;%5E%7B2 %7Dx-1%7D%5Coverset%7BHop%7D%7B%5Crightarrow%7D=%5Clim_ %7Bx%5Crightarrow&space;%5Cfrac%7B%5Cpi&space;%7D%7B4%7D%7D%5C frac%7B%5Ccos&space;%28x-%5Cfrac%7B%5Cpi&space;%7D%7B4%7D%29%7D%7B2%5Csin&space;2x%7D=% 5Cfrac%7B1%7D%7B2%7D


------------------------------------------
جواب سوال چهارم: وقتی در چند جمله ایها، x به سمت بی نهایت میل کند، مقدار چند جمله ای هم ارز با جمله ی دارای بیشترین توان x خواهد بود. بنابراین داریم:


[ برای مشاهده لینک ، لطفا با نام کاربری خود وارد شوید یا ثبت نام کنید ] Cfrac%7B3x%5E%7B2%7D&plus;5x-%5Csqrt%7B4x%5E%7B2%7D-x&plus;1%7D%7D%7Bx&plus;2&plus;%5Csqrt%7Bx&plus;3%7D%7D%5Cequiv&space;%5Clim _%7Bx%5Crightarrow&space;&plus;%5Cinfty&space;%7D%5Cfrac%7B3x%5E%7B 2%7D%7D%7Bx%7D=%5Clim_%7Bx%5Crightarrow&space;&plus;%5Cinfty&space; %7D3x%5Crightarrow&space;&plus;%5Cinfty


==========================================


سلام
ظاهراً یه فرمولی هست که میشه فهمید فاکتوریل یه عدد چند رقمیه
بنده این فرمول رو میخوام.
با تشکر

سلام.
من تا حالا چنین چیزی نشنیده بودم. ولی به نظرم ابتدا باید این سوال رو پرسید که: آیا مقدار !n (منظورم n فاکتوریل هستش) تقریبا برابر با [ برای مشاهده لینک ، لطفا با نام کاربری خود وارد شوید یا ثبت نام کنید ] 7D%7B2%7D%29%5E%7Bn%7D هست یا نه؟ اگه نیست بزرگتر از اون عبارت میشه یا کوچکتر؟ اگه بشه تو این زمینه کارایی کرد به نظرم میشه تقریبی گفت که !n به ازای n های دلخواه تقریبا چند رقمیه. (با توجه به تخمین تعداد ارقام n به توان n)

================================================== ======


دوستان اگه این سوال رو جواب بدید خیلی ممنون میشم
این جور سوالها رو بلدم ولی در مورد اینکه در صفحه ی yz باشه نمیدونم باید برای مرکزش x رو صفر در نظر بگیرم
معادله ي کره اي را بيابيد که از نقاط (4و0و0) و (3و1و2)و (6و2و0) بگذرد و مرکزش در صفحه ي yz باشد
سلام.
جواب این سوال همونیه که جناب MasterGeek اشاره کردند. (استفاده از سه معادله و سه مجهول) البته به نظر حقیر، تنا راه هم همینه و نمیشه راه جایگزینی بدست آورد چون نقطه ی قرینه ی نقطه ی (0,2,6) رو بدون معلوم بودن مرکز نمیشه بدست آورد. علاوه بر این نقطه نقطه ی (0,0,4) هم روی صفحه ی yz هست ولی باز هم نتیجه ی خاصی رو حاصل نمیکنه.



موفق باشین.
90/8/10

hts1369
01-11-2011, 10:39
سلام ممنون میشم به این سوالات حد جواب بدین
چون اینجا نمیشد تایپ کرد تو paint نوشتم صورت سوالاتو

[ برای مشاهده لینک ، لطفا با نام کاربری خود وارد شوید یا ثبت نام کنید ]
این سوالها رو از کتاب حساب دیفرانسیل انتگرال لیتهلد نیستن؟؟
من هم با اجازه ی اساتید سوالهاتو حل کردم ولی یه دونه جواب من با جواب اقا حمید فرق میکنه (که مطمئن هستم ماله من غلطه اگه اساتید ایراد کاره منو بگن ممنون میشم)
سوال اولت با قضیه ی فشردگی حل کردم
دومی واضحه
سومی رو با تغیر متغیر
چهارمی هم واضحه
[ برای مشاهده لینک ، لطفا با نام کاربری خود وارد شوید یا ثبت نام کنید ]

---------- Post added at 10:39 AM ---------- Previous post was at 10:36 AM ----------


فکر میکنم خودتون میتونین حلش کنین ولی به عنوان یه نکته:

1. مرکزش وقتی توی صفحه ی yz باشه، یعنی اینکه x=0 هست واسه نقطه ی مرکز (یعنی توی اون معادله ای که مینویسن اینطوریه:

x^2+(y-b)^2+(z-c)^2=r

حالا دیگه حله، سه معادله و سه مجهول که توسط جاگذاری اون سه نقطه به دست میاد


2. الان یکی از نکات خودش روی صفحه ی yz هست (6و2و0) پس مرکز هم روی همین صفحه هست و در نتیجه ی یه نقطه ی قرینه دقیقا روی همین صفحه است. از روی دو نطقه ی دیگه میشه شعاع رو بدست آورد اگر کمی روش کار بشه به نظرم میشه یه راه جایگزین رو (عوض راه کلاسیک) برای حل مسئله در پیش گرفت
x^2+(y-b)^2+(z-c)^2=r^2
فک کنم یه توان 2 یاد رفته:20:

davy jones
01-11-2011, 10:50
من هم با اجازه ی اساتید سوالهاتو حل کردم ولی یه دونه جواب من با جواب اقا حمید فرق میکنه (که مطمئن هستم ماله من غلطه اگه اساتید ایراد کاره منو بگن ممنون میشم)سلام.
هر چی نگاه میکنم، ایرادی در روش شما در حل مساله سوم نمیبینم. شاید روش حل من غلطه. اما آخه کجاش؟:41:
======================
ویرایش:
همین الان سوال رو تو والفام آلفا دادم و جواب زیر رو بهم داد:


برای مشاهده محتوا ، لطفا وارد شوید یا ثبت نام کنید

گزینه ی show steps رو بزنین تا مراحل کار رو نشون بده.

موفق باشین.
90/8/10

skyzare
01-11-2011, 11:23
همین الان سوال رو تو والفام آلفا دادم و جواب زیر رو بهم داد:


برای مشاهده محتوا ، لطفا وارد شوید یا ثبت نام کنیدگزینه ی show steps رو بزنین تا مراحل کار رو نشون بده.



با سلام ...

ببخشید یه سوال خیلی ساده ...من نمی دونم باید چه جوری داخل این سایت مسائل ریاضی رو بنویسیم :41: :41:

lebesgue
01-11-2011, 11:50
سلام
ظاهراً یه فرمولی هست که میشه فهمید فاکتوریل یه عدد چند رقمیه
بنده این فرمول رو میخوام.
با تشکر

تعداد ارقام عدد صحیح مثبت k در مبنای 10 برابر است با [ برای مشاهده لینک ، لطفا با نام کاربری خود وارد شوید یا ثبت نام کنید ](k)=%5Cleft%20%5Clfloor%20%5 Clog_{10}(k)%20%5Cright%20%5Crfloor+1.

بنابراین تعداد دقیق ارقام !n در مبنای 10، به صورت زیر قابل محاسبه است:

[ برای مشاهده لینک ، لطفا با نام کاربری خود وارد شوید یا ثبت نام کنید ]{10}(n!)=%5Clog_{10}(n (n-1)%5Ccdots%201)%5C%5C%5C%5C=%5Clog_{10}(n)+%5Clog_ {10}(n-1)+%5Ccdots%20%5Clog_{10}(1)%5C%5C%5C%5C%20=%5Csum _{i=1}^{n}%5Clog_{10}(i)%5CRightarrow%20D(n!)=%5Cl eft%20%5Clfloor%20%5Csum_{i=1}^{n}%5Clog_{10}(i)%2 0%5Cright%20%5Crfloor+1

برای n های به اندازه کافی بزرگ، میتوانید از تقریب استیرلینگ استفاده کنید:



[ برای مشاهده لینک ، لطفا با نام کاربری خود وارد شوید یا ثبت نام کنید ]!%5Csim%20%5Csqrt{2%5Cpi%20n }%5Cleft%20(%5Cfrac{n}{e}%20%5Cright%20)^{n}


برای مشاهده محتوا ، لطفا وارد شوید یا ثبت نام کنید

davy jones
01-11-2011, 11:58
با سلام ...

ببخشید یه سوال خیلی ساده ...من نمی دونم باید چه جوری داخل این سایت مسائل ریاضی رو بنویسیم :41: :41:
سلام.
روش خاصی وجود نداره. کافیه از امکانات و دکمه های اولیه ی تعبیه شده روی کیبورد کامپیوترتون برای تایپ عبارت مورد نظر استفاده کنین. مثل:
+ (جمع)
- (تفریق یا منها)
* (ضرب)
/ (تقسیم)
^ (توان)
! (فاکتوریل)
(....) پرانتزها برای جدا کردن و ساده کردن درک عبارت
pi (عدد پی)

مثلا من برای نوشتن صورت سوالی که لینکشو براتون گذاشتم این کارو کردم (اینطوری نوشتم):


lim x to pi/4 (sin(x-pi/4)/(2(sin(x))^2-1)a

(اون حرف a کوچک که رنگش رو هم سفید کردم جزء متن تایپی اصلی من در والفرام نیست و فقط اینجا صرفا به خاطر اینکه پرانتز آخر درست نمایش داده بشه نوشتم)

خودتون یکم باهاش ور برین دستتون میاد که باید چطوری بنویسین.

موفق باشین.
90/8/10

lebesgue
01-11-2011, 12:04
دوستان اگه این سوال رو جواب بدید خیلی ممنون میشم
این جور سوالها رو بلدم ولی در مورد اینکه در صفحه ی yz باشه نمیدونم باید برای مرکزش x رو صفر در نظر بگیرم
معادله ي کره اي را بيابيد که از نقاط (4و0و0) و (3و1و2)و (6و2و0) بگذرد و مرکزش در صفحه ي yz باشد

برای حل سریع دستگاه، میتوانید اینگونه عمل کنید:
مختصات مرکز را [ برای مشاهده لینک ، لطفا با نام کاربری خود وارد شوید یا ثبت نام کنید ](0,b,c) گرفته و از این فرض استفاده کنید که فاصله هر کدام از این سه نقطه تا مرکز، با هم برابر است. در نتیجه:


[ برای مشاهده لینک ، لطفا با نام کاربری خود وارد شوید یا ثبت نام کنید ]^2+(c-4)^2=2^2+(b-1)^2+(c-3)^2=(b-2)^2+(c-6)^2


با حذف b² و c² از طرفین، دستگاه ساده زیر بدست می آید:


[ برای مشاهده لینک ، لطفا با نام کاربری خود وارد شوید یا ثبت نام کنید ]

---------- Post added at 12:04 PM ---------- Previous post was at 12:01 PM ----------



[ برای مشاهده لینک ، لطفا با نام کاربری خود وارد شوید یا ثبت نام کنید ]

دقت کنید که [ برای مشاهده لینک ، لطفا با نام کاربری خود وارد شوید یا ثبت نام کنید ](%5Calpha%20+%5Cpi%2 0/2)=-%5Csin%20%5Calpha.

szh_1367
01-11-2011, 13:35
ببخشید دوستان بنده قراره که در چندین ازمون شرکت کنم و این ازمون ها مربوط هست به رشته کامپیوتر در بعضی از مسائل این رشته نیاز هست که محاسبات عددی انجام بدهیم که این محاسبات اگر به صورت معمولی حل بشوند زمان نسبتا طولانی رو به خوداختصاص میدهند

حالا سوال من اینکه راه میانبری برای اعمال چهارگانه ریاضی + - * / وجود داره که سریعتر به جواب برسیم

من توقع ندارم که بله با این فرمول شما دیگه نیاز نیست دوتا عدد 5 رقمی رو درهم ضرب کنید نه نه اصلا این توقع رو ندارم اما خب همیشه راهی وجود داره که راه طولانی رو کوتاه تر بکند

ممنون

emgjey
01-11-2011, 13:51
با سلام محضر اساتید
اثبات رابطه ی زیر را به کمک قضیه ی مانده ها میخواستم:

[ برای مشاهده لینک ، لطفا با نام کاربری خود وارد شوید یا ثبت نام کنید ] 5E%7B2n%7D%5Ctheta&space;d%5Ctheta&space;=%5Cpi&space;%5Cfrac%7B%282 n%29%21%7D%7B2%5E%7B2n-1%7D%28n%21%29%5E%7B2%7D%29%7D

maryam_72
01-11-2011, 20:25
سلام.
میشه جواب این سوالو بدین؟
اگر تابع fبر بازه [a,b] پیوسته باشد و[f:[a,b]->[a,b(یعنی fبازه[a,b]به بازه [a,b]تصویر کند)ثابت کنید:
به ازای هرcعضو[f(c)= c;[a,b

Greedy
01-11-2011, 22:01
سلام به همه ی دوستان!!



سلام.

جواب سوال اول:

[/CENTER]

واقعا ممنون عزیز از جوابا




این سوالها رو از کتاب حساب دیفرانسیل انتگرال لیتهلد نیستن؟؟



نمیدونم اینا رو استاد ریاضی پیشمون داد گفت حل کنید

MasterGeek
02-11-2011, 16:01
سلام
ظاهراً یه فرمولی هست که میشه فهمید فاکتوریل یه عدد چند رقمیه
بنده این فرمول رو میخوام.
با تشکر

به نظرم با توجه به اینکه رابطه ی زیر برقرار هست:
[ برای مشاهده لینک ، لطفا با نام کاربری خود وارد شوید یا ثبت نام کنید ]{num}%20(n)=%20\log_{10}^{n}

خیلی راحت میشه به رابطه ی زیر برای فاکتوریل رسید:
[ برای مشاهده لینک ، لطفا با نام کاربری خود وارد شوید یا ثبت نام کنید ]{num}%20(n!)=%20\sum_{i=1}^{n}\log _{10}^{i}

---------- Post added at 03:56 PM ---------- Previous post was at 03:56 PM ----------

البته یه جزء صحیح رو به بالا (ceil) باید بگیرید

---------- Post added at 03:59 PM ---------- Previous post was at 03:56 PM ----------


ببخشید دوستان بنده قراره که در چندین ازمون شرکت کنم و این ازمون ها مربوط هست به رشته کامپیوتر در بعضی از مسائل این رشته نیاز هست که محاسبات عددی انجام بدهیم که این محاسبات اگر به صورت معمولی حل بشوند زمان نسبتا طولانی رو به خوداختصاص میدهند

حالا سوال من اینکه راه میانبری برای اعمال چهارگانه ریاضی + - * / وجود داره که سریعتر به جواب برسیم

من توقع ندارم که بله با این فرمول شما دیگه نیاز نیست دوتا عدد 5 رقمی رو درهم ضرب کنید نه نه اصلا این توقع رو ندارم اما خب همیشه راهی وجود داره که راه طولانی رو کوتاه تر بکند

ممنون

راه حل هست ولی اکثرا ابتکاری هستند، توی همین بخش از فروم یه تاپیک راجع به محاسبات ذهنی هست که اکثرا این تیپ راه حل ها رو داره
ولی رفتن دنبال اینکه اینا رو یاد بگیری واسه آزمون به نظر من درست نیست و خیلی هم موثر نیست. بهتره با حل و تمرین کم کم خودت راههای ابتکاری مخصوص بخودت رو پیدا کنی اینجوری مطمئنا بازدهی بیشتری داری سر امتحان

---------- Post added at 04:01 PM ---------- Previous post was at 03:59 PM ----------


سلام.
میشه جواب این سوالو بدین؟
اگر تابع fبر بازه [a,b] پیوسته باشد و[f:[a,b]->[a,b(یعنی fبازه[a,b]به بازه [a,b]تصویر کند)ثابت کنید:
به ازای هرcعضو[f(c)= c;[a,b

سوالتون غلط هست و چاره ش یه مثال نقضه:

مثلا f=sin(x) از 0 تا 1 به عنوان دامنه، هر عددی رو از دامنه به 0 و 1 می نگارد.

اما sin(1/3) برابر 1/3 نیست. پس این قضیه که شما میگین غلطه

davy jones
03-11-2011, 12:24
سلام.
میشه جواب این سوالو بدین؟
اگر تابع fبر بازه [a,b] پیوسته باشد و[f:[a,b]->[a,b(یعنی fبازه[a,b]به بازه [a,b]تصویر کند)ثابت کنید:
به ازای هرcعضو[f(c)= c;[a,b



سوالتون غلط هست و چاره ش یه مثال نقضه:

مثلا f=sin(x) از 0 تا 1 به عنوان دامنه، هر عددی رو از دامنه به 0 و 1 می نگارد.

اما sin(1/3) برابر 1/3 نیست. پس این قضیه که شما میگین غلطه

سلام.
سوال ایشون غلط نیست و در واقع یه عبارت وجود دارد رو تو متن باید به جای به ازای هر می نوشتند. در واقع اصل سوال اینه که اگه تابع پیوسته ی f ، بازه [a,b] رو در دامنه ی خودش بر روی بازه ی [a,b] تصویر کنه، ثابت کنید وجود دارد حداقل یک عضو بازه ی دامنه مانند c که: f(c)= c .

دوستانی که تمایل به حل این سوال با شرایط جدیدی که عرض کردم رو دارند این نکته رو هم بررسی کنند که در این مساله حداقل یک d عضو بازه ی مذکور وجود دارد به طوری که مشتق تابع f در نقطه ی d برابر با یک میشود.

اگه کسی جواب نداد خودم در آینده جوابش رو میذارم به شرط حیات!!


با سلام محضر اساتید
اثبات رابطه ی زیر را به کمک قضیه ی مانده ها میخواستم:

[ برای مشاهده لینک ، لطفا با نام کاربری خود وارد شوید یا ثبت نام کنید ] 5E%7B2n%7D%5Ctheta&space;d%5Ctheta&space;=%5Cpi&space;%5Cfrac%7B%282 n%29%21%7D%7B2%5E%7B2n-1%7D%28n%21%29%5E%7B2%7D%29%7D
سلام.

برای مشاهده محتوا ، لطفا وارد شوید یا ثبت نام کنید
:23::23:



موفق باشین.
90/8/12

emgjey
03-11-2011, 14:24
سلام.
سوال ایشون غلط نیست و در واقع یه عبارت وجود دارد رو تو متن باید به جای به ازای هر می نوشتند. در واقع اصل سوال اینه که اگه تابع پیوسته ی f ، بازه [a,b] رو در دامنه ی خودش بر روی بازه ی [a,b] تصویر کنه، ثابت کنید وجود دارد حداقل یک عضو بازه ی دامنه مانند c که: f(c)= c .

دوستانی که تمایل به حل این سوال با شرایط جدیدی که عرض کردم رو دارند این نکته رو هم بررسی کنند که در این مساله حداقل یک d عضو بازه ی مذکور وجود دارد به طوری که مشتق تابع f در نقطه ی d برابر با یک میشود.

اگه کسی جواب نداد خودم در آینده جوابش رو میذارم به شرط حیات!!


سلام.

برای مشاهده محتوا ، لطفا وارد شوید یا ثبت نام کنید:23::23:



موفق باشین.
90/8/12
با تشکر
ولی بنده روش اثبات رو میخواستم

lebesgue
03-11-2011, 17:14
با سلام محضر اساتید
اثبات رابطه ی زیر را به کمک قضیه ی مانده ها میخواستم:


[ برای مشاهده لینک ، لطفا با نام کاربری خود وارد شوید یا ثبت نام کنید ] 5E%7B2n%7D%5Ctheta&space;d%5Ctheta&space;=%5Cpi&space;%5Cfrac%7B%282 n%29%21%7D%7B2%5E%7B2n-1%7D%28n%21%29%5E%7B2%7D%29%7D

[ برای مشاهده لینک ، لطفا با نام کاربری خود وارد شوید یا ثبت نام کنید ]{120}%20z=e^{ix}%5CRightarrow%20dx =%5Cfrac{dz}{iz}%5C%5C%5C%5C%20%5Csin%20x=%5Cfrac{ e^{ix}-e^{-ix}}{2i}=%5Cfrac{z-z^{-1}}{2i}

[ برای مشاهده لینک ، لطفا با نام کاربری خود وارد شوید یا ثبت نام کنید ]{120}%20%5CRightarrow%20%5Cint_{0} ^{%5Cpi}%5Csin^{2n}%20(x)dx=%5Cfrac{1}{2}%5Cint_{-%5Cpi}^{%5Cpi}%5Csin^{2n}%20(x)dx=%5Coint_{%5Cleft %20|%20z%20%5Cright%20|=1}%5Cleft%20(%20%5Cfrac{z-z^{-1}}{2i}%20%5Cright%20)^{2n}%5Cfrac{dz}{2iz}

حال کافی است که مانده اینتگرند در 0 محاسبه شود.

Ship Storm
03-11-2011, 18:07
سلام بر متفکران و ریاضی دانان عزیز
2 تا سوال دارم ممنون میشم جوابش رو بدید
===

1-اگر 8 تاس متمایز را بریزیم احتمال اینکه تمام اعداد از 1 تا 6 ظاهر شود چقدر است ؟
2-از جعبه ای که دارای 5 مهره قرمز و 5 مهره سفید و 5 مهره آبی است 2 مهره به تصادف بیرون میکشیم ، به ازای هر مهره آّی 1 دلار و هر مهره سفید 2 دلار برنده میشویم ، اما به ازای هر مهره قرمز 3 دلار می بازیم ، اگر X میزان برد و باخت ما باشد مقادیر ممکن X و احتمال های متناظر با آن ها چقدر است ؟ تابع توزیع X را بدست بیاورید

emgjey
04-11-2011, 01:53
ضمن تشکر از 1233445566 عزیز
بنده مشکل اصلیم محاسبه ی مقادیر مانده ها بود. به مشتق زیر میرسم که نمیدونم اونو چطور ساده کنم که بشه طرف دوم تساوی
[ برای مشاهده لینک ، لطفا با نام کاربری خود وارد شوید یا ثبت نام کنید ] n%7D%7D%28-%28z%29%5E%7B4%7D+2z%5E%7B2%7D+1%29%5E%7Bn%7D

lebesgue
04-11-2011, 19:22
راهنمایی: از قضیه دو جمله ای استفاده کنید:

[ برای مشاهده لینک ، لطفا با نام کاربری خود وارد شوید یا ثبت نام کنید ]{120}%20(z-z^{-1})^{2n}=z^{2n}+%5Ccdots%20+%5Cbegin{pmatrix}2n%20 %5C%5C%20n%20%5Cend{pmatrix}(-1)^{n}z^0+%5Ccdots%20+z^{-2n}

در ضمن، جواب صحیح، یک ضریب 2 با آنچه که شما نوشتید، تفاوت دارد.

MasterGeek
05-11-2011, 23:51
سلام.
سوال ایشون غلط نیست و در واقع یه عبارت وجود دارد رو تو متن باید به جای به ازای هر می نوشتند. در واقع اصل سوال اینه که اگه تابع پیوسته ی f ، بازه [a,b] رو در دامنه ی خودش بر روی بازه ی [a,b] تصویر کنه، ثابت کنید وجود دارد حداقل یک عضو بازه ی دامنه مانند c که: f(c)= c .

دوستانی که تمایل به حل این سوال با شرایط جدیدی که عرض کردم رو دارند این نکته رو هم بررسی کنند که در این مساله حداقل یک d عضو بازه ی مذکور وجود دارد به طوری که مشتق تابع f در نقطه ی d برابر با یک میشود.

اگه کسی جواب نداد خودم در آینده جوابش رو میذارم به شرط حیات!!


سلام.

برای مشاهده محتوا ، لطفا وارد شوید یا ثبت نام کنید
:23::23:



موفق باشین.
90/8/12

بازم به نظرم ممکن نیست هنوز مگر اینکه شرط اینکه یکنوا بودن رو هم اضافه کنید....
چون مثلا در بازه.ی 0 تا 1 تابع زیر رو در نظر بگیرید:

[ برای مشاهده لینک ، لطفا با نام کاربری خود وارد شوید یا ثبت نام کنید ](x)=%20\left\{\begin{matrix}%20-4x+2%20\;\;\;\;%20x%3C0.5%20\\%200%20\;\;\;\;%20x\ ge%200.5%20\end{matrix}\right.

پس من شرط یکنوا بودن رو اضافه میکنم:
این قضیه
[ برای مشاهده لینک ، لطفا با نام کاربری خود وارد شوید یا ثبت نام کنید ][a\;\;%20b]%20\to%20[a\;\;%20b]%20\;\;%20\&%20f%20\;\;is%20\;%20monolitic%20\Rightarrow%20\ex ists%20\;%20c%20\in%20[a\;\;b]%20:%20\;%20f(c)=c

پس باید در این صورت معادله.ی f(c)=c باید جوابی در بازه.ی a,b داشته باشد. چون تابع f یکنواست پس حتما خط y=c تابع f رو قطع میکنه...
اثبات رسمی تر این قضیه فکر کنم همون قضیه رول میشه یا همچین چیزی.....

---------- Post added at 11:51 PM ---------- Previous post was at 11:50 PM ----------

البته شرط پیوسته بودن هم به طور ضمنی میشه در یکنوائی جاش داد....

SuperSt@r
06-11-2011, 14:51
سلام به همه دوستان عزیز
چنتا سوال داشتم که الان دوتاش رو میپرسم امیدوارم و ازتون خواهش میکنم جوابم رو بدید خیلی فوریه

1-فرض کنید دو تابع f و g از R به R هستند و پیوسته نیز هستند به طوری که برای هر x عضو اعداد گویا f(x=g(x ثابت کنید برای هر x عضو R نیز رابطه f(x=g(x برقرار است

2-ثابت کنید هیچ تابع پیوسته ای وجود ندارد که هر مقدار از بردش را دقیقا دو بار اختیار کند. و ایا تابع پیوسته ای وجود دارد که هر مقدار از بردش را دقیقا سه بار اختیار کند؟

ممنون میشم جوابم رو بدید خیلی فوریه:11:

ali_hp
07-11-2011, 02:25
بازم به نظرم ممکن نیست هنوز مگر اینکه شرط اینکه یکنوا بودن رو هم اضافه کنید....
چون مثلا در بازه.ی 0 تا 1 تابع زیر رو در نظر بگیرید:

[ برای مشاهده لینک ، لطفا با نام کاربری خود وارد شوید یا ثبت نام کنید ](x)=%20\left\{\begin{matrix}%20-4x+2%20\;\;\;\;%20x%3C0.5%20\\%200%20\;\;\;\;%20x\ ge%200.5%20\end{matrix}\right.

پس من شرط یکنوا بودن رو اضافه میکنم:
این قضیه
[ برای مشاهده لینک ، لطفا با نام کاربری خود وارد شوید یا ثبت نام کنید ][a\;\;%20b]%20\to%20[a\;\;%20b]%20\;\;%20\&%20f%20\;\;is%20\;%20monolitic%20\Rightarrow%20\ex ists%20\;%20c%20\in%20[a\;\;b]%20:%20\;%20f(c)=c

پس باید در این صورت معادله.ی f(c)=c باید جوابی در بازه.ی a,b داشته باشد. چون تابع f یکنواست پس حتما خط y=c تابع f رو قطع میکنه...
اثبات رسمی تر این قضیه فکر کنم همون قضیه رول میشه یا همچین چیزی.....

---------- Post added at 11:51 PM ---------- Previous post was at 11:50 PM ----------

البته شرط پیوسته بودن هم به طور ضمنی میشه در یکنوائی جاش داد....
سلام
مثالي كه شما زدي بازه صفر و يك به خودش تصوير نمي كنه، مثلا f(0)=2
و ٢ در بازه صفر و يك نيست.
نيازي به شرط يكنوايي هم نيست.
اين قضيه رو با به كار بردن قضيه مقدار مياني براي تابع f(x)-x روي بازه [a,b] ميشه ثابت كرد.

ali_hp
07-11-2011, 03:26
1-اگر 8 تاس متمایز را بریزیم احتمال اینکه تمام اعداد از 1 تا 6 ظاهر شود چقدر است ؟

سلام
فرض کنید A پیشامد ظاهر نشدن حداقل یکی از اعداد یک تا شش در ریختن 8 تاس باشد.(که مکمل پیشامد مطلوب مساله است!)
فرض کنید A_i پیشامد ظاهر نشدن عدد i در ریختن این 8 تاس باشد.پس داریم:

[ برای مشاهده لینک ، لطفا با نام کاربری خود وارد شوید یا ثبت نام کنید ] up&space;A_4%5Ccup&space;A_5%5Ccup&space;A_6%29=%5C%5C%5C%5C&space;%5Csum&space; P%28A_i%29-&space;%5Csum_%7Bi%5Cneq&space;j%7D&space;P%28A_i%5Ccap&space;A_j%29&space;&plus;&space;... &space;-P%28A_1%5Ccap&space;A_2%5Ccap&space;A_3%5Ccap&space;A_4%5Ccap&space;A_5%5C cap&space;A_6%29%5C%5C

[ برای مشاهده لینک ، لطفا با نام کاربری خود وارد شوید یا ثبت نام کنید ]


پس احتمال اینکه همه اعداد یک تا شش ظاهر شوند می شود:

[ برای مشاهده لینک ، لطفا با نام کاربری خود وارد شوید یا ثبت نام کنید ] 2%7D

ali_hp
07-11-2011, 04:19
سلام به همه دوستان عزیز
چنتا سوال داشتم که الان دوتاش رو میپرسم امیدوارم و ازتون خواهش میکنم جوابم رو بدید خیلی فوریه

1-فرض کنید دو تابع f و g از R به R هستند و پیوسته نیز هستند به طوری که برای هر x عضو اعداد گویا f(x=g(x ثابت کنید برای هر x عضو R نیز رابطه f(x=g(x برقرار است

2-ثابت کنید هیچ تابع پیوسته ای وجود ندارد که هر مقدار از بردش را دقیقا دو بار اختیار کند. و ایا تابع پیوسته ای وجود دارد که هر مقدار از بردش را دقیقا سه بار اختیار کند؟

ممنون میشم جوابم رو بدید خیلی فوریه:11:
1-برای هر x حقیقی یک دنباله از اعداد گویا مثل q_n در نظر بگیرید که به x میل کند(چنین دنباله ای وجود دارد،چرا؟) داریم:

[ برای مشاهده لینک ، لطفا با نام کاربری خود وارد شوید یا ثبت نام کنید ] m_%7Bn%5Crightarrow&space;%5Cinfty%7D&space;f%28q_n%29=%5Clim_ %7Bn%5Crightarrow&space;%5Cinfty%7D&space;g%28q_n%29%5C%5C%5CR ightarrow&space;f%28x%29=g%28x%29

که در نتیجه گیری اخر از پیوستگی f , g استفاده شده است.

2-فرض می کنیم دامنه توابع ما بازه باشن یا کل R ...
فرض کنید چنین تابعی وجود دارد x را یک مقدار در برد این تابع در نظر بگیرید،پس a,b وجود دارند که f(a)=f(b)=x و a<b . چون f پیوسته است،پس ماکسیمم مقدار خود را روی [a,b] در نقطه ای مثل c میگیرد.و f(c)>x .
حال باید یک نقطه دیگر مثل d موجود باشد که f(d)=f(c)l.(اینجا اگه شکل تابع رو در نظر بگیرین میبینین که d چه خارج [a,b]باشه چه داخلش با توجه به پیوسته بودن نمودار باعث میشه بعضی از مقادیر بیشتر از دوبار گرفته بشن!)
اگر d خارج از [a,b] باشه و مثلا d>b:
با استفاده از قضیه مقدار میانی روی بازه [b,d] برای هر t که f(b) < t < f(c)=f(d)l عدد حقیقی s وجود دارد که f(s)=t همچنین با بکاربردن مقدار میانگین روی بازه های [a,c] , [c,b] دو مقدار دیگر مثل s بدست می اید که f(s)=t باشد پس مقدار t را تابع f حداقل سه بار میگیرد که تناقض است.
حالت d داخل بازه [a,b] باشه هم تقریبا مشابه حل میشه.
برای قسمت دوم مساله هم نمودار چنین تابعی رو معرفی میکنیم!
حرف انگلیسی N رو در صفحه مختصات بنویسید!(البته N ای که یکم مایل باشه،نه قاِئم!)
از نقطه انتهایی شمال شرقی این N یک N دیگه بنویسید و این روندو ادامه بدین...همینطور انتهای جنوب غربی این N یک N دیگه بزارین و ادامه بدین...این نمودار تابعی رو نشون میده که پیوستست،با دامنه R وهر مقدار حقیقی رو دقیقا سه بار می گیره!البته میشه ضابطه هم براش بدست اورد...

davy jones
07-11-2011, 13:07
بازم به نظرم ممکن نیست هنوز مگر اینکه شرط اینکه یکنوا بودن رو هم اضافه کنید....
چون مثلا در بازه.ی 0 تا 1 تابع زیر رو در نظر بگیرید:

[ برای مشاهده لینک ، لطفا با نام کاربری خود وارد شوید یا ثبت نام کنید ] ix%7D%20-4x+2%20%5C;%5C;%5C;%5C;%20x%3C0.5%20%5C%5C%200%20% 5C;%5C;%5C;%5C;%20x%5Cge%200.5%20%5Cend%7Bmatrix%7 D%5Cright.

پس من شرط یکنوا بودن رو اضافه میکنم:
این قضیه
[ برای مشاهده لینک ، لطفا با نام کاربری خود وارد شوید یا ثبت نام کنید ][a%5C;%5C;%20b]%20%5Cto%20[a%5C;%5C;%20b]%20%5C;%5C;%20%5C&%20f%20%5C;%5C;is%20%5C;%20monolitic%20%5CRightarr ow%20%5Cexists%20%5C;%20c%20%5Cin%20[a%5C;%5C;b]%20:%20%5C;%20f%28c%29=c

پس باید در این صورت معادله.ی f(c)=c باید جوابی در بازه.ی a,b داشته باشد. چون تابع f یکنواست پس حتما خط y=c تابع f رو قطع میکنه...
اثبات رسمی تر این قضیه فکر کنم همون قضیه رول میشه یا همچین چیزی.....

البته شرط پیوسته بودن هم به طور ضمنی میشه در یکنوائی جاش داد....


سلام
مثالي كه شما زدي بازه صفر و يك به خودش تصوير نمي كنه، مثلا f(0)=2
و ٢ در بازه صفر و يك نيست.
نيازي به شرط يكنوايي هم نيست.
اين قضيه رو با به كار بردن قضيه مقدار مياني براي تابع f(x)-x روي بازه [a,b] ميشه ثابت كرد.

سلام. با تشکر از دوستان بابت بحث و بررسی روی این مساله.
جواب نهایی رو جناب ali_hp دادند.



دوستانی که تمایل به حل این سوال با شرایط جدیدی که عرض کردم رو دارند این نکته رو هم بررسی کنند که در این مساله حداقل یک d عضو بازه ی مذکور وجود دارد به طوری که مشتق تابع f در نقطه ی d برابر با یک میشود.


برای اثبات این قسمت هم این شرط رو باید اضافه کرد که ابتدا فراموش کرده بودم و اون اینکه علاوه بر شروط قبلی بایستی f(a)=a و f(b)=b فرض شود.


موفق باشین.
90/8/16 مصادف با عید سعید قربان. عید همگی مبارک!!

SuperSt@r
07-11-2011, 14:23
1-برای هر x حقیقی یک دنباله از اعداد گویا مثل q_n در نظر بگیرید که به x میل کند(چنین دنباله ای وجود دارد،چرا؟) داریم:

[ برای مشاهده لینک ، لطفا با نام کاربری خود وارد شوید یا ثبت نام کنید ] m_%7Bn%5Crightarrow&space;%5Cinfty%7D&space;f%28q_n%29=%5Clim_ %7Bn%5Crightarrow&space;%5Cinfty%7D&space;g%28q_n%29%5C%5C%5CR ightarrow&space;f%28x%29=g%28x%29

که در نتیجه گیری اخر از پیوستگی f , g استفاده شده است.

2-فرض می کنیم دامنه توابع ما بازه باشن یا کل R ...
فرض کنید چنین تابعی وجود دارد x را یک مقدار در برد این تابع در نظر بگیرید،پس a,b وجود دارند که f(a)=f(b)=x و a<b . چون f پیوسته است،پس ماکسیمم مقدار خود را روی [a,b] در نقطه ای مثل c میگیرد.و f(c)>x .
حال باید یک نقطه دیگر مثل d موجود باشد که f(d)=f(c)l.(اینجا اگه شکل تابع رو در نظر بگیرین میبینین که d چه خارج [a,b]باشه چه داخلش با توجه به پیوسته بودن نمودار باعث میشه بعضی از مقادیر بیشتر از دوبار گرفته بشن!)
اگر d خارج از [a,b] باشه و مثلا d>b:
با استفاده از قضیه مقدار میانی روی بازه [b,d] برای هر t که f(b) < t < f(c)=f(d)l عدد حقیقی s وجود دارد که f(s)=t همچنین با بکاربردن مقدار میانگین روی بازه های [a,c] , [c,b] دو مقدار دیگر مثل s بدست می اید که f(s)=t باشد پس مقدار t را تابع f حداقل سه بار میگیرد که تناقض است.
حالت d داخل بازه [a,b] باشه هم تقریبا مشابه حل میشه.
برای قسمت دوم مساله هم نمودار چنین تابعی رو معرفی میکنیم!
حرف انگلیسی N رو در صفحه مختصات بنویسید!(البته N ای که یکم مایل باشه،نه قاِئم!)
از نقطه انتهایی شمال شرقی این N یک N دیگه بنویسید و این روندو ادامه بدین...همینطور انتهای جنوب غربی این N یک N دیگه بزارین و ادامه بدین...این نمودار تابعی رو نشون میده که پیوستست،با دامنه R وهر مقدار حقیقی رو دقیقا سه بار می گیره!البته میشه ضابطه هم براش بدست اورد...

سلام دوست عزیز خیلی خیلی ازت ممنونم به خاطر اینکه وقت گذاشتی و جوابم رو دادی:10:
اگه میشه سوال اول رو بیشتر توضیح بده آخه قشنگ متوجه نشدم
ولی سوال دوم رو گرفتم دستت درد نکنه ولی قسمت دوم که گفته هر مقدار از بردش رو سه بار اختیار کنه که ما اگه تابع N رو بکشیم و این روند رو ادامه بدیم نقاطی هستند که بیش از سه بار و حتی بیشتر هم تکرار میشن حتی شما اگه همون خود N رو به تنهایی در نظر بگیریم نقاطی هستن که سه بار بردشون یکیه چه برسه که این روند رو ادامه بدیم

در ضمن میشه نرم افزاری رو که خودتون برا نوشتن توابع و مسائل ریاضی استفاده میکنید رو معرفی کنید که اگه بازم سوالی داشتم با اون نرم افزار به راحتی بنویسمش ممنون میشم:11:

lebesgue
07-11-2011, 15:15
برای قسمت دوم مساله هم نمودار چنین تابعی رو معرفی میکنیم!
حرف انگلیسی N رو در صفحه مختصات بنویسید!(البته N ای که یکم مایل باشه،نه قاِئم!)
از نقطه انتهایی شمال شرقی این N یک N دیگه بنویسید و این روندو ادامه بدین...همینطور انتهای جنوب غربی این N یک N دیگه بزارین و ادامه بدین...این نمودار تابعی رو نشون میده که پیوستست،با دامنه R وهر مقدار حقیقی رو دقیقا سه بار می گیره!البته میشه ضابطه هم براش بدست اورد...

یک نمونه برای ضابطه چنین تابعی، [ برای مشاهده لینک ، لطفا با نام کاربری خود وارد شوید یا ثبت نام کنید ](x/a) هست و نمودارش به صورت زیر:


[ برای مشاهده لینک ، لطفا با نام کاربری خود وارد شوید یا ثبت نام کنید ]

a تقریباً 0.2172 بوده و به طور دقیق، ریشه معادله زیر است:

[ برای مشاهده لینک ، لطفا با نام کاربری خود وارد شوید یا ثبت نام کنید ]^{-1}(x)=%5Cfrac{%5Csqrt{1-x^2}}{x}-%5Cpi

همچنین نسخه فشرده (کراندار) آن:


[ برای مشاهده لینک ، لطفا با نام کاربری خود وارد شوید یا ثبت نام کنید ]

کسی میتواند بگوید نمودار بالا چگونه بدست آمده و ضابطه تابع آن چیست؟

ali_hp
07-11-2011, 16:22
سلام دوست عزیز خیلی خیلی ازت ممنونم به خاطر اینکه وقت گذاشتی و جوابم رو دادی:10:
اگه میشه سوال اول رو بیشتر توضیح بده آخه قشنگ متوجه نشدم
ولی سوال دوم رو گرفتم دستت درد نکنه ولی قسمت دوم که گفته هر مقدار از بردش رو سه بار اختیار کنه که ما اگه تابع N رو بکشیم و این روند رو ادامه بدیم نقاطی هستند که بیش از سه بار و حتی بیشتر هم تکرار میشن حتی شما اگه همون خود N رو به تنهایی در نظر بگیریم نقاطی هستن که سه بار بردشون یکیه چه برسه که این روند رو ادامه بدیم

در ضمن میشه نرم افزاری رو که خودتون برا نوشتن توابع و مسائل ریاضی استفاده میکنید رو معرفی کنید که اگه بازم سوالی داشتم با اون نرم افزار به راحتی بنویسمش ممنون میشم:11:
سلام،خواهش می کنم.
خوب کجاشو متوجه نشدی شما؟اول از این استفاده شد که به هر عددی با اعداد گویا میشه میل کرد،بعد هم با توجه به پیوستگی، حد f یک دنباله برابر f حد دنباله میشه...شاید اینجوری واضح تر باشه.

[ برای مشاهده لینک ، لطفا با نام کاربری خود وارد شوید یا ثبت نام کنید ] m_%7Bn%5Crightarrow%5Cinfty%7Df%28q_n%29=%5Clim_%7 Bn%5Crightarrow%5Cinfty%7Dg%28q_n%29%5C%5C&space;%5C%5C% 5CRightarrow&space;f%28%5Clim_%7Bn%5Crightarrow%5Cinfty% 7Dq_n%29=g%28%5Clim_%7Bn%5Crightarrow%5Cinfty%7Dq_ n%29%5CRightarrow&space;f%28x%29=g%28x%29
سوال دوم هم فکر کنم شما متوجه ساختار نمودار نشدین، شبيه نموداری که دوستمون 1233445566
گفتن ، که البته نمودار ایشون هم ضابطه داره هم برای حالت کراندارش جوابو گفتن.
برای فرمول نویسی هم میتونین اینجا رو ببینین:

برای مشاهده محتوا ، لطفا وارد شوید یا ثبت نام کنید

SuperSt@r
07-11-2011, 21:18
سلام،خواهش می کنم.
خوب کجاشو متوجه نشدی شما؟اول از این استفاده شد که به هر عددی با اعداد گویا میشه میل کرد،بعد هم با توجه به پیوستگی، حد f یک دنباله برابر f حد دنباله میشه...شاید اینجوری واضح تر باشه.

[ برای مشاهده لینک ، لطفا با نام کاربری خود وارد شوید یا ثبت نام کنید ] m_%7Bn%5Crightarrow%5Cinfty%7Df%28q_n%29=%5Clim_%7 Bn%5Crightarrow%5Cinfty%7Dg%28q_n%29%5C%5C&space;%5C%5C% 5CRightarrow&space;f%28%5Clim_%7Bn%5Crightarrow%5Cinfty% 7Dq_n%29=g%28%5Clim_%7Bn%5Crightarrow%5Cinfty%7Dq_ n%29%5CRightarrow&space;f%28x%29=g%28x%29
سوال دوم هم فکر کنم شما متوجه ساختار نمودار نشدین، شبيه نموداری که دوستمون 1233445566
گفتن ، که البته نمودار ایشون هم ضابطه داره هم برای حالت کراندارش جوابو گفتن.
برای فرمول نویسی هم میتونین اینجا رو ببینین:

برای مشاهده محتوا ، لطفا وارد شوید یا ثبت نام کنید


بازم ازت ممنونم دوست عزیز
دقیقا همین دنباله ها رو متوجه نمیشم که گفتید

برای هر x حقیقی یک دنباله از اعداد گویا مثل q_n در نظر بگیرید که به x میل کند(چنین دنباله ای وجود دارد،چرا؟)
و

اول از این استفاده شد که به هر عددی با اعداد گویا میشه میل کرد

در مورد اون تابع هم آره شکلش رو درست متوجه نشده بودم که بعدش که فک کردم متوجه شدم الانم که اون تابع با ضابطه اش رو دیدم دیگه بهتر شد

MasterGeek
08-11-2011, 03:08
نوشته شده توسط MasterGeek
بازم به نظرم ممکن نیست هنوز مگر اینکه شرط اینکه یکنوا بودن رو هم اضافه کنید....
چون مثلا در بازه.ی 0 تا 1 تابع زیر رو در نظر بگیرید:



پس من شرط یکنوا بودن رو اضافه میکنم:
این قضیه


پس باید در این صورت معادله.ی f(c)=c باید جوابی در بازه.ی a,b داشته باشد. چون تابع f یکنواست پس حتما خط y=c تابع f رو قطع میکنه...
اثبات رسمی تر این قضیه فکر کنم همون قضیه رول میشه یا همچین چیزی.....

البته شرط پیوسته بودن هم به طور ضمنی میشه در یکنوائی جاش داد....




سلام
مثالي كه شما زدي بازه صفر و يك به خودش تصوير نمي كنه، مثلا f(0)=2
و ٢ در بازه صفر و يك نيست.
نيازي به شرط يكنوايي هم نيست.
اين قضيه رو با به كار بردن قضيه مقدار مياني براي تابع f(x)-x روي بازه [a,b] ميشه ثابت كرد.



درسته
اما اون تابعی که نوشتم کلیتش مصداق داره (تعریفش رو اشتباه کرده بودم ولی میشه از 1 شروع بشه تغییری در کلیت مثال نقض نمیده)

صورت سوال هم از اول اشکال داشت مثلا شرط یکنوائی ولو لازم نباشد ولی لازم هست که تابع در کل بازه مشتق پذیر باشه...(که یه درجه بالاتر از پیوستگی هست)

اما درست تر همون فرضهائی بود که شما کردین....

ali_hp
08-11-2011, 19:17
بازم ازت ممنونم دوست عزیز
دقیقا همین دنباله ها رو متوجه نمیشم که گفتید

برای هر x حقیقی یک دنباله از اعداد گویا مثل q_n در نظر بگیرید که به x میل کند(چنین دنباله ای وجود دارد،چرا؟)
دنباله های زیادی با این خاصیت وجود دارد،مثلا می توانید بسط اعشاری x در مبنای 10 را در نظر بگیرید ، q_0 را برابر با ارقام قبل از ممیز x بگیرید ، q_1 را برابر تقریب x تا یک رقم اعشار بگیرید، q_2 را برابر تقریب x تا دو رقم اعشار بگیرید و ....مثلا اگر x=π باشد دنباله q_n بصورت زیر می شود:

برای مشاهده محتوا ، لطفا وارد شوید یا ثبت نام کنیدکه یک جمله عمومیش میشه:

برای مشاهده محتوا ، لطفا وارد شوید یا ثبت نام کنید

mofidy1
10-11-2011, 13:13
با سلام

دوستان عزیز روی حل معادله ی زیبای زیر فکر کنید. هدف به دست آوردن تمام ریشه ها ی حقیقی و مختلط آن است ( البته بدون استفاده از نرم افزارهای ریاضی!!). روی روش های مختلف حل آن هم بحث کنید. آیا می توان از این مساله، یک روش کلی برای حل معادلات مشابه دیگر به دست آورد؟


[ برای مشاهده لینک ، لطفا با نام کاربری خود وارد شوید یا ثبت نام کنید ]^9&plus;5x^8-7x^7-5x^6-7x^5&plus;x^4&plus;7x^3-2x&plus;10=0

با تشکر

19 آبان 1390

MasterGeek
11-11-2011, 01:03
با سلام

دوستان عزیز روی حل معادله ی زیبای زیر فکر کنید. هدف به دست آوردن تمام ریشه ها ی حقیقی و مختلط آن است ( البته بدون استفاده از نرم افزارهای ریاضی!!). روی روش های مختلف حل آن هم بحث کنید. آیا می توان از این مساله، یک روش کلی برای حل معادلات مشابه دیگر به دست آورد؟


[ برای مشاهده لینک ، لطفا با نام کاربری خود وارد شوید یا ثبت نام کنید ]^9&plus;5x^8-7x^7-5x^6-7x^5&plus;x^4&plus;7x^3-2x&plus;10=0

با تشکر

19 آبان 1390

من فقط یه استراتژی واسه حلش دارم و اون از طریق پارامترهای مجهول و معادل گرفتن ضرب فاکتورهای ممکن هست. فقط مشکل اینکه باید اینا رو از روی خود معادله بفهمیم (یادمه یه روشی برای این کار وجود داشت):

1. تعداد ریشه های حقیقی معادله
2. تعداد ریشه های مکرر معادله (به خصوص اگه مختلط مکرر باشند)

اگه اینا رو بفهیم اونوقت با روش ضرائب مجهول شاید حل بشه...

SuperSt@r
13-11-2011, 15:43
سلام دوستان من این سوال رو روش فکری نکردم فقط حالا که دیدم اینجام گفتم که اینجا وارد کنم ببینم شما چه جوری حلش میکنید که اگه خودم حلش کردم با راه شما چک کنم البته فک میکنم ساده باشه
گفته نشان دهید

[ برای مشاهده لینک ، لطفا با نام کاربری خود وارد شوید یا ثبت نام کنید ] rac%7B1%7D%7Bn%7D%5Cleft&space;%5Cright&space;%29%5En=e

hts1369
13-11-2011, 16:30
سلام دوستان من این سوال رو روش فکری نکردم فقط حالا که دیدم اینجام گفتم که اینجا وارد کنم ببینم شما چه جوری حلش میکنید که اگه خودم حلش کردم با راه شما چک کنم البته فک میکنم ساده باشه
گفته نشان دهید

[ برای مشاهده لینک ، لطفا با نام کاربری خود وارد شوید یا ثبت نام کنید ] rac%7B1%7D%7Bn%7D%5Cleft&space;%5Cright&space;%29%5En=e


حلش رو برات گذاشتم
اولش بینهایت در صفر رو به صفر بر صفر تبدیل میکنیم و بعدش از hop استفاده میکنیم
[ برای مشاهده لینک ، لطفا با نام کاربری خود وارد شوید یا ثبت نام کنید ]
این یکی رو هم بعنوان تمرین خودت حل کن (مثله قبلی هست)
[ برای مشاهده لینک ، لطفا با نام کاربری خود وارد شوید یا ثبت نام کنید ]

tasnim68
16-11-2011, 19:08
سلام دوستان. آیا میشه این نامساوی رو از طریق قضیه لاگرانژ (مقدار میانگین) اثبات کرد:

[ برای مشاهده لینک ، لطفا با نام کاربری خود وارد شوید یا ثبت نام کنید ]


اگر میشه لطفا منو راهنمایی کنید که چطور حلش کنم.

sepehr_x50
16-11-2011, 19:26
سلام. دوستان لینک زیر رو لطفا چک کنید. میخوام ببینم چطور از از معادله اول به معادله دوم رسید:


برای مشاهده محتوا ، لطفا وارد شوید یا ثبت نام کنید

متن کامل :


برای مشاهده محتوا ، لطفا وارد شوید یا ثبت نام کنید

ممنون

lebesgue
16-11-2011, 22:46
سلام دوستان. آیا میشه این نامساوی رو از طریق قضیه لاگرانژ (مقدار میانگین) اثبات کرد:

[ برای مشاهده لینک ، لطفا با نام کاربری خود وارد شوید یا ثبت نام کنید ]


اگر میشه لطفا منو راهنمایی کنید که چطور حلش کنم.

میتوانید به طور جداگانه، هر کدام از نامساوی های زیر را با استفاده از قضیه مقدار میانگین اثبات کنید (برای x در بازه مورد نظر):

[ برای مشاهده لینک ، لطفا با نام کاربری خود وارد شوید یا ثبت نام کنید ]{1}{1+x}%3C%20%5 Cfrac{%5Cln%20(1+x)}{x}%3C%201%5C%5C%5C%5C%5C%5C%2 01%3C%20%5Cfrac{%5Csin^{-1}x}{x}%3C%20%5Cfrac{1}{%5Csqrt{1-x^2}}

حال از ترکیب ایندو می توان نامساوی مطلوب را نتیجه گرفت.
برای اثبات هر کدام از آن نامساوی ها نیز، مثلاً نامساوی اول، توجه کنید که:

[ برای مشاهده لینک ، لطفا با نام کاربری خود وارد شوید یا ثبت نام کنید ]{%5Cln%20(1+x)}{x}=%5C frac{%5Cln%20(1+x)-%5Cln%20(1+0)}{x-0}

lebesgue
16-11-2011, 23:40
سلام. دوستان لینک زیر رو لطفا چک کنید. میخوام ببینم چطور از از معادله اول به معادله دوم رسید:


برای مشاهده محتوا ، لطفا وارد شوید یا ثبت نام کنید

متن کامل :


برای مشاهده محتوا ، لطفا وارد شوید یا ثبت نام کنید

ممنون

خب اولی یک معادله دیفرانسیل جدا شدنی است و دومی هم (ظاهراً) پاسخ آن. اگر با شیوه حل این نوع معادلات آشنا نیستید، میتوانید لینکی که در همان صفحه معرفی شده (Separation of Variables) را ببینید! مثال های مشابهی در آنجا حل شده است.

tasnim68
17-11-2011, 11:57
میتوانید به طور جداگانه، هر کدام از نامساوی های زیر را با استفاده از قضیه مقدار میانگین اثبات کنید (برای x در بازه مورد نظر):

[ برای مشاهده لینک ، لطفا با نام کاربری خود وارد شوید یا ثبت نام کنید ] %3C%20%5Cfrac%7B%5Cln%20%281+x%29%7D%7Bx%7D%3C%201 %5C%5C%5C%5C%5C%5C%201%3C%20%5Cfrac%7B%5Csin%5E%7B-1%7Dx%7D%7Bx%7D%3C%20%5Cfrac%7B1%7D%7B%5Csqrt%7B1-x%5E2%7D%7D

حال از ترکیب ایندو می توان نامساوی مطلوب را نتیجه گرفت.
برای اثبات هر کدام از آن نامساوی ها نیز، مثلاً نامساوی اول، توجه کنید که:

[ برای مشاهده لینک ، لطفا با نام کاربری خود وارد شوید یا ثبت نام کنید ] D%7Bx%7D=%5Cfrac%7B%5Cln%20%281+x%29-%5Cln%20%281+0%29%7D%7Bx-0%7D


خب من متوجه نشدم شما این نامساوی هارو چجوری بدست آوردید؛ ولی استاد ما وقتی این سوالو طرح کرد گفت می تونید طرفین رو به arcsinx ضرب کنید که تقریبا میشه:


[ برای مشاهده لینک ، لطفا با نام کاربری خود وارد شوید یا ثبت نام کنید ]


حالا فکر کنم اگه ضابطه (f(t رو بدست بیاریم مسئله حل بشه. من خودم این 2 تا ضابطه رو حدس زدم ولی از هیچ کدوم به نتیجه نرسیدم:


f(t)= t‍ ln(1+x) .1
f(t)= ln(t+x^t) .2

lebesgue
17-11-2011, 12:12
خب من متوجه نشدم شما این نامساوی هارو چجوری بدست آوردید؛


ساده است، مثلاً برای اولی، بنا به قضیه مقدار میانگین داریم:

[ برای مشاهده لینک ، لطفا با نام کاربری خود وارد شوید یا ثبت نام کنید ](x)=%5Cln(1+x)%20%5C%5C%5C%5 C%5CRightarrow%20%5Cfrac{%5Cln(1+x)}{x}=%5Cfrac{f( x)-f(0)}{x-0}=f'(c)=%5Cfrac{1}{1+c}%5C;%20%5C;%20%5C;%20%5C:% 20%5C:%5C;%20%5C;%20%5C;%20%5C;%20%5C;%20%5C;%200% 3Cc%3Cx

حالا چون تابع [ برای مشاهده لینک ، لطفا با نام کاربری خود وارد شوید یا ثبت نام کنید ](1+c) در [ برای مشاهده لینک ، لطفا با نام کاربری خود وارد شوید یا ثبت نام کنید ] نزولی است، داریم:

[ برای مشاهده لینک ، لطفا با نام کاربری خود وارد شوید یا ثبت نام کنید ]{1}{1+x}%3C%5Cfrac{1}{ 1+c}%3C%20%5Cfrac{1}{1+0}%20%5CRightarrow%20%5Cfra c{1}{1+x}%3C%5Cfrac{%5Cln(1+x)}{x}%3C1

tasnim68
17-11-2011, 14:18
خب آخه مگه نگفته تو بازه (0,1) ؟ شما بجای 1 گذاشتید x ؟

lebesgue
17-11-2011, 14:57
خب آخه مگه نگفته تو بازه (0,1) ؟ شما بجای 1 گذاشتید x ؟
خیر، x در بازه [ برای مشاهده لینک ، لطفا با نام کاربری خود وارد شوید یا ثبت نام کنید ](0,1) است و c در بازه [ برای مشاهده لینک ، لطفا با نام کاربری خود وارد شوید یا ثبت نام کنید ](0,x).

---------- Post added at 02:57 PM ---------- Previous post was at 02:55 PM ----------

توضیج بیشتر: خط دوم در اثبات بالا، از قضیه مقدار میانگین بدست آمده است؛ برای هر x در بازه [ برای مشاهده لینک ، لطفا با نام کاربری خود وارد شوید یا ثبت نام کنید ](0,1)، تابع f (برای سادگی فرض کنید (f(t) شرایط قضیه مقدار میانگین را در بازه [ برای مشاهده لینک ، لطفا با نام کاربری خود وارد شوید یا ثبت نام کنید ](0,x) داراست، در نتیجه c ای در این بازه موجود است که:


[ برای مشاهده لینک ، لطفا با نام کاربری خود وارد شوید یا ثبت نام کنید ]{f(x)-f(0)}{x-0}=f'(c)

tasnim68
17-11-2011, 15:42
آهان؛ مرسی متوجه شدم.

می تونم خواهش کنم چگونگی بدست آوردن نامساوی دوم و نهایتا ترکیب اون دو نامساوی و نتیجه گیری رو هم بنویسید؟

راستش من تو دبیرستان رشتم تجربی بود واسه همین به ریاضی زیاد تسلط ندارم و الان حتی اصلا نمی دونم arcsinx یعنی چی! اینه که یه مقدار تو حل این مسئله به مشکل برخوردم!

davy jones
17-11-2011, 19:02
سلام. دوستان لینک زیر رو لطفا چک کنید. میخوام ببینم چطور از از معادله اول به معادله دوم رسید:


برای مشاهده محتوا ، لطفا وارد شوید یا ثبت نام کنیدمتن کامل :


برای مشاهده محتوا ، لطفا وارد شوید یا ثبت نام کنیدممنون
سلام.
این معادله ی ریکاتی هستش اگه اشتباه نکنم و با یک تغییر متغیر به معادله ی همگن اویلر قابل تبدیل هست:


[ برای مشاهده لینک ، لطفا با نام کاربری خود وارد شوید یا ثبت نام کنید ] hrm%7Bd%7D&space;t%7D=pcA-cA%5E%7B2%7D


از تغییر متغیر زیر استفاده میکنیم:


[ برای مشاهده لینک ، لطفا با نام کاربری خود وارد شوید یا ثبت نام کنید ]

که در اصل تغییر متغیر به صورت [ برای مشاهده لینک ، لطفا با نام کاربری خود وارد شوید یا ثبت نام کنید ] هستش که q در اینجا ضریب عبارت A به توان 2 در معادله بالا هستش.

در نتیجه داریم:


[ برای مشاهده لینک ، لطفا با نام کاربری خود وارد شوید یا ثبت نام کنید ]

که این همون معادله ی همگن اویلر هستش که تازه جمله ی R(t).y رو هم نداره و حلش خیلی آسونتر میشه. کافیه که 'y رو با z معادل بگیرید و در نتیجه یک درجه از معادله کم میشه و ادامه ی مراحل که خودتون بهش بیشتر از من واقفید!

برای مطالعه ی بیشتر در زمینه ی معادلات دیفرانسیل ریکاتی به پیوندهای زیر مراجعه کنید:


برای مشاهده محتوا ، لطفا وارد شوید یا ثبت نام کنید

برای مشاهده محتوا ، لطفا وارد شوید یا ثبت نام کنید

موفق باشین.
90/8/26

lebesgue
18-11-2011, 12:05
davy jones عزیز، این معادله ریکاتی نیست، در همون لینک ویکیپدیا نوشته:



...
it is an equation of the form
[ برای مشاهده لینک ، لطفا با نام کاربری خود وارد شوید یا ثبت نام کنید ]
where
[ برای مشاهده لینک ، لطفا با نام کاربری خود وارد شوید یا ثبت نام کنید ]
...


این یک معادله جدا شدنی هست که به سادگی قابل حل می باشد:

[ برای مشاهده لینک ، لطفا با نام کاربری خود وارد شوید یا ثبت نام کنید ]{%5Cmathrm{d}A}{%5Cmat hrm{d}%20t}=50000cA-cA^2%5C%5C%5C%5C%20%5CRightarrow%20%5Cfrac{%5Ctext up{d}A}{50000cA-cA^2}=dt%5C%5C%5C%5C%20%5CRightarrow%20%5Cint%20%5 Cfrac{%5Ctextup{d}A}{50000cA-cA^2}=%5Cint%20dt%5C%5C%5C%5C
بقیه مسئله هم انتگرال گیری و ساده سازی است.

lebesgue
18-11-2011, 12:14
آهان؛ مرسی متوجه شدم.

می تونم خواهش کنم چگونگی بدست آوردن نامساوی دوم و نهایتا ترکیب اون دو نامساوی و نتیجه گیری رو هم بنویسید؟

راستش من تو دبیرستان رشتم تجربی بود واسه همین به ریاضی زیاد تسلط ندارم و الان حتی اصلا نمی دونم arcsinx یعنی چی! اینه که یه مقدار تو حل این مسئله به مشکل برخوردم!

ایشون تابع معکوس سینوس هستند، البته در بازه [ برای مشاهده لینک ، لطفا با نام کاربری خود وارد شوید یا ثبت نام کنید ][-%5Cpi/2,%5Cpi/2]:

[ برای مشاهده لینک ، لطفا با نام کاربری خود وارد شوید یا ثبت نام کنید ] row%20x=%5Csin%20y%5C%5C%5Ctextup{Domain}:x%5Cin[-1,1]%5C;%20,%5C;%20%5Ctextup{Range}:y%5Cin%20[-%5Cfrac{%5Cpi%20}{2},%5Cfrac{%5Cpi%20}{2}]

نمودار این تابع:

[ برای مشاهده لینک ، لطفا با نام کاربری خود وارد شوید یا ثبت نام کنید ]
و همچنین مشتقش:

[ برای مشاهده لینک ، لطفا با نام کاربری خود وارد شوید یا ثبت نام کنید ]{%5Cmathrm{d}%20}{%5Cm athrm{d}%20x}%5Carcsin%20x=%5Cfrac{1}{%5Csqrt{1-x^2}}
([ برای مشاهده لینک ، لطفا با نام کاربری خود وارد شوید یا ثبت نام کنید ] رو با [ برای مشاهده لینک ، لطفا با نام کاربری خود وارد شوید یا ثبت نام کنید ]^{-1}x هم نشون میدن.)

بقیه مسئله رو سعی کنید خودتون حل کنید دیگه!

davy jones
18-11-2011, 12:18
davy jones عزیز، این معادله ریکاتی نیست، در همون لینک ویکیپدیا نوشته:

این یک معادله جدا شدنی هست که به سادگی قابل حل می باشد:

[ برای مشاهده لینک ، لطفا با نام کاربری خود وارد شوید یا ثبت نام کنید ] D%7B%5Cmathrm%7Bd%7D%20t%7D=50000cA-cA%5E2%5C%5C%5C%5C%20%5CRightarrow%20%5Cfrac%7B%5C textup%7Bd%7DA%7D%7B50000cA-cA%5E2%7D=dt%5C%5C%5C%5C%20%5CRightarrow%20%5Cint% 20%5Cfrac%7B%5Ctextup%7Bd%7DA%7D%7B50000cA-cA%5E2%7D=%5Cint%20dt%5C%5C%5C%5C
بقیه مسئله هم انتگرال گیری و ساده سازی است.
سلام دوست عزیزم.
شما از بنده تو این زمینه ها استاد ترید.:11:
یه سوال داشتم: حالا یعنی روش حل بنده کلا غلطه یا اینکه جواب آخر هر دو راه یکی میشه و فقط من لقمه رو دور سر خودم پیچوندم؟:31:

skyzare
18-11-2011, 13:32
با سلام ....

پست ها رو نگاه میکردم ماشاالله چه قدر بعضی ها ریاضی شون خوبه ....:20: من هم فعلا در حال یادگیری ...:41:
1- ببخشید من میخوام دو تابع f و g که چند ضابطه ای هست رو داخل هم ضرب کنم ( حالا تقسیم جمع و تفریق ) . ایا یاید چیز خاصی رو در نظر بگیرم ؟ اخه دامنه های ضابطه های مختلف با هم یکی نیستند باید چی کار کنم ؟ باید حتما نمودار رسم بشه ؟ یا بدون نمودار هم میشه ؟

sepehr_x50
18-11-2011, 14:23
davy jones عزیز، این معادله ریکاتی نیست، در همون لینک ویکیپدیا نوشته:

این یک معادله جدا شدنی هست که به سادگی قابل حل می باشد:

[ برای مشاهده لینک ، لطفا با نام کاربری خود وارد شوید یا ثبت نام کنید ]{%5Cmathrm{d}A}{%5Cmat hrm{d}%20t}=50000cA-cA^2%5C%5C%5C%5C%20%5CRightarrow%20%5Cfrac{%5Ctext up{d}A}{50000cA-cA^2}=dt%5C%5C%5C%5C%20%5CRightarrow%20%5Cint%20%5 Cfrac{%5Ctextup{d}A}{50000cA-cA^2}=%5Cint%20dt%5C%5C%5C%5C
بقیه مسئله هم انتگرال گیری و ساده سازی است.

مرسی از زحماتتون. ممنون میشم انتگرال گیری و ساده سازی هم بفرمایید چطور انجام شده... :20:

davy jones
18-11-2011, 14:31
با سلام ....

پست ها رو نگاه میکردم ماشاالله چه قدر بعضی ها ریاضی شون خوبه ....:20: من هم فعلا در حال یادگیری ...:41:
1- ببخشید من میخوام دو تابع f و g که چند ضابطه ای هست رو داخل هم ضرب کنم ( حالا تقسیم جمع و تفریق ) . ایا یاید چیز خاصی رو در نظر بگیرم ؟ اخه دامنه های ضابطه های مختلف با هم یکی نیستند باید چی کار کنم ؟ باید حتما نمودار رسم بشه ؟ یا بدون نمودار هم میشه ؟
سلام.
در توابع چند ضابطه ای باید دقت داشت که هر عملی باید در دامنه های مشترک صورت بگیره. یعنی مثلا اگه دو تابع چند ضابطه ای f و g رو میخوایم در هم ضرب کنیم، ضابطه هایی از این دو تابع در هم ضرب میشن که دامنه های مشترک داشته باشند.
اگه منظورمو متوجه نشدین، بگین تا یه مثال بزنم.

===================

مرسی از زحماتتون. ممنون میشم انتگرال گیری و ساده سازی هم بفرمایید چطور انجام شده... [ برای مشاهده لینک ، لطفا با نام کاربری خود وارد شوید یا ثبت نام کنید ]

سلام.
از تبدیل ضرب کسرها به جمع استفاده کردم:


[ برای مشاهده لینک ، لطفا با نام کاربری خود وارد شوید یا ثبت نام کنید ] t&space;%5Cfrac%7BdA%7D%7B50000cA-cA%5E%7B2%7D%7D%5CRightarrow&space;t=%5Cfrac%7B1%7D%7Bc% 7D%5Cint&space;%5Cfrac%7BdA%7D%7BA%2850000-A%29%7D=%5Cfrac%7B1%7D%7Bc%7D%5Cint%28%5Cfrac%7BX% 7D%7BA%7D&plus;%5Cfrac%7BY%7D%7B50000-A%7D%29dA=%5Cfrac%7B1%7D%7Bc%7D%5Cint&space;%5Cfrac%7B%2 8AY-AX&plus;50000X%29dA%7D%7BA%2850000-A%29%7D%5CRightarrow&space;%5Cleft%5C%7B%5Cbegin%7Bmatri x%7D&space;AY-AX=0%5C%5C&space;50000X=1&space;%5Cend%7Bmatrix%7D%5Cright.%5C Rightarrow&space;X=Y=%5Cfrac%7B1%7D%7B50000%7D%5CRightar row&space;t=%5Cfrac%7B1%7D%7B50000c%7D%5Cint&space;%28%5Cfrac% 7B1%7D%7BA%7D&plus;%5Cfrac%7B1%7D%7B%2850000-A%29%7D%29dA=%5Cfrac%7B1%7D%7B50000c%7D%5Cint&space;%5Cf rac%7BdA%7D%7BA%7D&plus;%5Cint&space;%5Cfrac%7BdA%7D%7B%28500 00-A%29%7D=%5Cfrac%7B1%7D%7B50000c%7D%28%5Cln&space;A%5C;&space;-&space;%5Cln&space;%2850000-A%29%29=%5Cfrac%7B1%7D%7B50000c%7D%5C;&space;%5Cln&space;%28%5 Cfrac%7BA%7D%7B50000-A%7D%29%5CRightarrow&space;%5Cfrac%7BA%7D%7B50000-A%7D=e%5E%7B50000ct%7D%5CRightarrow&space;A=%5Cfrac%7B50 000e%5E%7B50000ct%7D%7D%7B1&plus;e%5E%7B50000ct%7D%7D


موفق باشین.
90/8/27

sepehr_x50
18-11-2011, 16:58
سلام.
در توابع چند ضابطه ای باید دقت داشت که هر عملی باید در دامنه های مشترک صورت بگیره. یعنی مثلا اگه دو تابع چند ضابطه ای f و g رو میخوایم در هم ضرب کنیم، ضابطه هایی از این دو تابع در هم ضرب میشن که دامنه های مشترک داشته باشند.
اگه منظورمو متوجه نشدین، بگین تا یه مثال بزنم.

===================


سلام.
از تبدیل ضرب کسرها به جمع استفاده کردم:


[ برای مشاهده لینک ، لطفا با نام کاربری خود وارد شوید یا ثبت نام کنید ] t&space;%5Cfrac%7BdA%7D%7B50000cA-cA%5E%7B2%7D%7D%5CRightarrow&space;t=%5Cfrac%7B1%7D%7Bc% 7D%5Cint&space;%5Cfrac%7BdA%7D%7BA%2850000-A%29%7D=%5Cfrac%7B1%7D%7Bc%7D%5Cint%28%5Cfrac%7BX% 7D%7BA%7D&plus;%5Cfrac%7BY%7D%7B50000-A%7D%29dA=%5Cfrac%7B1%7D%7Bc%7D%5Cint&space;%5Cfrac%7B%2 8AY-AX&plus;50000X%29dA%7D%7BA%2850000-A%29%7D%5CRightarrow&space;%5Cleft%5C%7B%5Cbegin%7Bmatri x%7D&space;AY-AX=0%5C%5C&space;50000X=1&space;%5Cend%7Bmatrix%7D%5Cright.%5C Rightarrow&space;X=Y=%5Cfrac%7B1%7D%7B50000%7D%5CRightar row&space;t=%5Cfrac%7B1%7D%7B50000c%7D%5Cint&space;%28%5Cfrac% 7B1%7D%7BA%7D&plus;%5Cfrac%7B1%7D%7B%2850000-A%29%7D%29dA=%5Cfrac%7B1%7D%7B50000c%7D%5Cint&space;%5Cf rac%7BdA%7D%7BA%7D&plus;%5Cint&space;%5Cfrac%7BdA%7D%7B%28500 00-A%29%7D=%5Cfrac%7B1%7D%7B50000c%7D%28%5Cln&space;A%5C;&space;-&space;%5Cln&space;%2850000-A%29%29=%5Cfrac%7B1%7D%7B50000c%7D%5C;&space;%5Cln&space;%28%5 Cfrac%7BA%7D%7B50000-A%7D%29%5CRightarrow&space;%5Cfrac%7BA%7D%7B50000-A%7D=e%5E%7B50000ct%7D%5CRightarrow&space;A=%5Cfrac%7B50 000e%5E%7B50000ct%7D%7D%7B1&plus;e%5E%7B50000ct%7D%7D


موفق باشین.
90/8/27



مرسی آقا. لطف کردی... ما هنوز چون روش های انتگرال گیریو نخوندیم خب نمی دونم...
فقط اینا جواب های آخرشون یکیه؟!

ضمنا برای تایپ معادلات و فرمول های ریاضی از چی استفاده میکنید؟ با ورد 2003 میشه کامل این علائم رو تایپ کرد؟

tasnim68
18-11-2011, 17:50
بقیه مسئله رو سعی کنید خودتون حل کنید دیگه!

خب من الان همونطور که نوشته بودید به همون نامساوی ها رسیدم:


[ برای مشاهده لینک ، لطفا با نام کاربری خود وارد شوید یا ثبت نام کنید ]


ولی مسئله اینجاست که نمی دونم چطور باید از ترکیب این دو به نتیجه برسم!؟ به هم تقسیمشون کنم بشه همون نامساوی اول؟

skyzare
18-11-2011, 19:09
سلام.
در توابع چند ضابطه ای باید دقت داشت که هر عملی باید در دامنه های مشترک صورت بگیره. یعنی مثلا اگه دو تابع چند ضابطه ای f و g رو میخوایم در هم ضرب کنیم، ضابطه هایی از این دو تابع در هم ضرب میشن که دامنه های مشترک داشته باشند.
اگه منظورمو متوجه نشدین، بگین تا یه مثال بزنم.


با سلام ....

با تشکر از پاسختون .....

متوجه منظورتون شدم ...ولی اگه لطف کنید یه مثال بزتید . مثلا این یکی که خودم نوشتم :





[ برای مشاهده لینک ، لطفا با نام کاربری خود وارد شوید یا ثبت نام کنید ] 7D&space;-x&plus;1%5C:&space;%5C:&space;%5C:&space;%5C:&space;%5C:&space;%5C:&space;%5C:&space;x%5Cgeqslant &space;2&space;&&space;%5C%5C&space;x-2%5C:&space;%5C:&space;%5C:&space;%5C:&space;%5C:&space;%5C:&space;x%3C2&space;&&space;%5Cend%7Bmatrix%7D%5Cright.

[ برای مشاهده لینک ، لطفا با نام کاربری خود وارد شوید یا ثبت نام کنید ] 7D&space;-x%5E2&plus;1%5C:&space;%5C:&space;%5C:&space;%5C:&space;%5C:&space;%5C:&space;%5C:&space;x%5Cgeqs lant1&space;&&space;%5C%5C&space;2x-1%5C:&space;%5C:&space;%5C:&space;%5C:&space;%5C:&space;%5C:&space;x%3C1&space;&&space;%5Cend%7Bmatrix%7D%5Cright.


================================================== ==============


من یه سوال دیگه هم داشتم میخواستم بدونم این معادله زیر چه جوری حل میشه البته خودم یه مقداریش رو حل کردم :


[ برای مشاهده لینک ، لطفا با نام کاربری خود وارد شوید یا ثبت نام کنید ] Bx&plus;y%7D=81&space;&&space;%5C%5C&space;3%5E%7Bx-y%7D=8&space;&&space;%5Cend%7Bmatrix%7D%5Cright.



خودم این جوریش کردم که بعد توان های طرفین رو با هم برابر قرار بدم ولی پایه های دومی با هم برابر نیستند نمیدونم چی کارش کنم ؟



[ برای مشاهده لینک ، لطفا با نام کاربری خود وارد شوید یا ثبت نام کنید ] Bx&plus;y%7D=3%5E6&space;&&space;%5C%5C&space;3%5E%7Bx-y%7D=2%5E3&space;&&space;%5Cend%7Bmatrix%7D%5Cright.

skyzare
18-11-2011, 22:58
با سلام ...

میخواستم بدون این روابط چه جوری اثبات میشه ؟؟


[ برای مشاهده لینک ، لطفا با نام کاربری خود وارد شوید یا ثبت نام کنید ]



[ برای مشاهده لینک ، لطفا با نام کاربری خود وارد شوید یا ثبت نام کنید ]

hamed6672
18-11-2011, 23:02
[ برای مشاهده لینک ، لطفا با نام کاربری خود وارد شوید یا ثبت نام کنید ] Bx&plus;y%7D=81&space;&&space;%5C%5C&space;3%5E%7Bx-y%7D=8&space;&&space;%5Cend%7Bmatrix%7D%5Cright.







با سلام ...

من سوال شما رو این جوری تونستم حل کنم ، امیدوارم درست باشه :


[ برای مشاهده لینک ، لطفا با نام کاربری خود وارد شوید یا ثبت نام کنید ]

hamed6672
18-11-2011, 23:09
با سلام ...

میخواستم بدون این روابط چه جوری اثبات میشه ؟؟


[ برای مشاهده لینک ، لطفا با نام کاربری خود وارد شوید یا ثبت نام کنید ]



[ برای مشاهده لینک ، لطفا با نام کاربری خود وارد شوید یا ثبت نام کنید ]





[ برای مشاهده لینک ، لطفا با نام کاربری خود وارد شوید یا ثبت نام کنید ]
__________________________________________________ _________________


[ برای مشاهده لینک ، لطفا با نام کاربری خود وارد شوید یا ثبت نام کنید ]

lebesgue
19-11-2011, 13:19
یه سوال داشتم: حالا یعنی روش حل بنده کلا غلطه یا اینکه جواب آخر هر دو راه یکی میشه و فقط من لقمه رو دور سر خودم پیچوندم؟:31:
گزینه دوم!:31:


فقط اینا جواب های آخرشون یکیه؟!

در واقع davy jones عزیز ثابت انتگرال نامعین را فراموش کرده اند. با در نظر گرفتن این ثابت، تغییری کوچک در جواب نهایی ایشان ایجاد می شود:


[ برای مشاهده لینک ، لطفا با نام کاربری خود وارد شوید یا ثبت نام کنید ]{120}%20A(t)=%5Cfrac{50000e^{50000 ct}}{e^{50000ct}+k}

که k در اینجا یک ثابت مثبت است. با ضرب صورت و مخرج در [ برای مشاهده لینک ، لطفا با نام کاربری خود وارد شوید یا ثبت نام کنید ]^{-50000ct} بدست می آید:


[ برای مشاهده لینک ، لطفا با نام کاربری خود وارد شوید یا ثبت نام کنید ]{120}%20A(t)=%5Cfrac{50000}{1+ke^{-50000ct}}
حال باید ثابت k با استفاده از اطلاعات مسئله محاسبه شود.

lebesgue
19-11-2011, 13:42
خب من الان همونطور که نوشته بودید به همون نامساوی ها رسیدم:


[ برای مشاهده لینک ، لطفا با نام کاربری خود وارد شوید یا ثبت نام کنید ]


ولی مسئله اینجاست که نمی دونم چطور باید از ترکیب این دو به نتیجه برسم!؟ به هم تقسیمشون کنم بشه همون نامساوی اول؟

باید از قوانین نامساوی ها استفاده کنید، مثلا از دو نامساوی بالا فوراً نتیجه می شود:

[ برای مشاهده لینک ، لطفا با نام کاربری خود وارد شوید یا ثبت نام کنید ](1+x)%5Cln(1+x)%5C% 5C%5C%5C%20%5Csqrt{1-x^2}%5Carcsin%20x%3Cx%5C%5C%5C%5C%20%5CRightarrow% 20%5Csqrt{1-x^2}%5Carcsin%20x%3C(1+x)%5Cln(1+x)%5C%5C%5C%5C%20 %5CRightarrow%20%5Cfrac{%5Csqrt{1-x^2}}{(1+x)}%3C%20%5Cfrac{%5Cln(1+x)}{%5Carcsin%20 x}%5C%5C%5C%5C%5C%5C%20%5Cfrac{%5Csqrt{1-x^2}}{(1+x)}=%5Cfrac{%5Csqrt{(1-x)}%5Csqrt{(1+x)}}{%5Csqrt{(1+x)}%5Csqrt{(1+x)}}=% 5Csqrt{%5Cfrac{1-x}{1+x}}

البته واضح است که تمام اینها برای x در بازه مورد نظر می باشد که طرفین نامساوی و همچنین زیر رادیکال ها مثبت هستند. اثبات طرف دیگر نامساوی هم که ساده تر است.

tasnim68
19-11-2011, 14:36
باید از قوانین نامساوی ها استفاده کنید، مثلا از دو نامساوی بالا فوراً نتیجه می شود:

[ برای مشاهده لینک ، لطفا با نام کاربری خود وارد شوید یا ثبت نام کنید ] x%29%5C%5C%5C%5C%20%5Csqrt%7B1-x%5E2%7D%5Carcsin%20x%3Cx%5C%5C%5C%5C%20%5CRightar row%20%5Csqrt%7B1-x%5E2%7D%5Carcsin%20x%3C%281+x%29%5Cln%281+x%29%5C %5C%5C%5C%20%5CRightarrow%20%5Cfrac%7B%5Csqrt%7B1-x%5E2%7D%7D%7B%281+x%29%7D%3C%20%5Cfrac%7B%5Cln%28 1+x%29%7D%7B%5Carcsin%20x%7D%5C%5C%5C%5C%5C%5C%20% 5Cfrac%7B%5Csqrt%7B1-x%5E2%7D%7D%7B%281+x%29%7D=%5Cfrac%7B%5Csqrt%7B%28 1-x%29%7D%5Csqrt%7B%281+x%29%7D%7D%7B%5Csqrt%7B%281+ x%29%7D%5Csqrt%7B%281+x%29%7D%7D=%5Csqrt%7B%5Cfrac %7B1-x%7D%7B1+x%7D%7D

البته واضح است که تمام اینها برای x در بازه مورد نظر می باشد که طرفین نامساوی و همچنین زیر رادیکال ها مثبت هستند. اثبات طرف دیگر نامساوی هم که ساده تر است.


ممنونم. حل شد.

davy jones
19-11-2011, 18:20
ضمنا برای تایپ معادلات و فرمول های ریاضی از چی استفاده میکنید؟ با ورد 2003 میشه کامل این علائم رو تایپ کرد؟
سلام.
برای تایپ معادلات هم از سایت زیر استفاده کنید:

برای مشاهده محتوا ، لطفا وارد شوید یا ثبت نام کنیدبرای توضیحات بیشتر در مورد نحوه ی استفاده از سایت بالا به این تاپیک مراجعه کنید:

برای مشاهده محتوا ، لطفا وارد شوید یا ثبت نام کنید

=================


با سلام ....

با تشکر از پاسختون .....

متوجه منظورتون شدم ...ولی اگه لطف کنید یه مثال بزتید . مثلا این یکی که خودم نوشتم :





[ برای مشاهده لینک ، لطفا با نام کاربری خود وارد شوید یا ثبت نام کنید ] 7D&space;-x&plus;1%5C:&space;%5C:&space;%5C:&space;%5C:&space;%5C:&space;%5C:&space;%5C:&space;x%5Cgeqslant &space;2&space;&&space;%5C%5C&space;x-2%5C:&space;%5C:&space;%5C:&space;%5C:&space;%5C:&space;%5C:&space;x%3C2&space;&&space;%5Cend%7Bmatrix%7D%5Cright.

[ برای مشاهده لینک ، لطفا با نام کاربری خود وارد شوید یا ثبت نام کنید ] 7D&space;-x%5E2&plus;1%5C:&space;%5C:&space;%5C:&space;%5C:&space;%5C:&space;%5C:&space;%5C:&space;x%5Cgeqs lant1&space;&&space;%5C%5C&space;2x-1%5C:&space;%5C:&space;%5C:&space;%5C:&space;%5C:&space;%5C:&space;x%3C1&space;&&space;%5Cend%7Bmatrix%7D%5Cright.





سلام.
بفرمایین:


[ برای مشاهده لینک ، لطفا با نام کاربری خود وارد شوید یا ثبت نام کنید ] 5Cbegin%7Bmatrix%7D&space;%282x-1%29%28x-2%29&space;&&space;,%5C;&space;%5C;&space;x%3C1%5C%5C&space;%28x-2%29%281-x%5E%7B2%7D%29&space;&&space;,%5C;&space;%5C;&space;1%5Cleq&space;x%3C2%5C%5C&space;%281-x%29%281-x%5E%7B2%7D%29&&space;,%5C;&space;%5C;&space;x%5Cgeq&space;2&space;%5Cend%7Bmatrix%7D%5Cri ght.


موفق باشین.
90/8/28

hts1369
19-11-2011, 20:07
سلام بر دوستان عزیز
میخواستم ببینم جواب این حد چند میشه
[ برای مشاهده لینک ، لطفا با نام کاربری خود وارد شوید یا ثبت نام کنید ]
و اینکه این جور حدها رو چطور باید با Mathematica حل کرد.

hamed6672
19-11-2011, 21:17
سلام بر دوستان عزیز
میخواستم ببینم جواب این حد چند میشه
[ برای مشاهده لینک ، لطفا با نام کاربری خود وارد شوید یا ثبت نام کنید ]
و اینکه این جور حدها رو چطور باید با Mathematica حل کرد.

با سلام ...

اگه اشتباه نکنم این حد هم مثل حدی که تو این صفحه


برای مشاهده محتوا ، لطفا وارد شوید یا ثبت نام کنیدحل کردم ، حل میشه و اگه مراحل رو به همون ترتیب پیش برین در نهایت جواب بشه e^a .

hts1369
19-11-2011, 21:35
با سلام ...

اگه اشتباه نکنم این حد هم مثل حدی که تو این صفحه


برای مشاهده محتوا ، لطفا وارد شوید یا ثبت نام کنیدحل کردم ، حل میشه و اگه مراحل رو به همون ترتیب پیش برین در نهایت جواب بشه e^a .
ممنون این چیزی که شما حل کردی رو من هم تو این پست حل کردم

برای مشاهده محتوا ، لطفا وارد شوید یا ثبت نام کنیدولی این حدی که من نوشتم یک حد دو متغیره هست و در زمان استفاده از hop باید از یک تابع دو متغیره مشتق گرفت که مشتقش با این چیز فرق میکنه و جواب اخر هم تفاوت داره
البته این کتاب که من این سوال رو توش دیدم همین جواب e به توان a رو نوشته (البته بدون راه حل) ولی مطمئن نیستم درست باشه.
بنظر من باید جواب بشه صفر

davy jones
20-11-2011, 08:42
ممنون این چیزی که شما حل کردی رو من هم تو این پست حل کردم

برای مشاهده محتوا ، لطفا وارد شوید یا ثبت نام کنیدولی این حدی که من نوشتم یک حد دو متغیره هست و در زمان استفاده از hop باید از یک تابع دو متغیره مشتق گرفت که مشتقش با این چیز فرق میکنه و جواب اخر هم تفاوت داره
البته این کتاب که من این سوال رو توش دیدم همین جواب e به توان a رو نوشته (البته بدون راه حل) ولی مطمئن نیستم درست باشه.
بنظر من باید جواب بشه صفر
اگه در سوال شما y به سمت صفر میل کنه جواب میشه صفر. برای حل سوال شما ابتدا اگه y رو میل بدیم به سمت a هیچ اتفاقی نمیفته و حد به همون سوال استاندارد تبدیل میشه. در واقع دو تا حد که یکیش هم هیچ ابهامی نداره رو در هم ادغام کرده تو سوال.

موفق باشین.
90/8/29